You are on page 1of 358

Civil Law Q&As (2007-2013) hectorchristopher@yahoo.com dbaratbateladot@gmail.

com

A Compilation of the

Questions and Suggested Answers

In the

PHILIPPINE BAR EXAMINATIONS 2007-2013

In

CIVIL LAW
Compiled and Arranged By:

Baratbate-Ladot, Delight

Salise, Hector Christopher Jay-Arh Jr. M.

(University of San Jose-Recoletos School of Law)

ANSWERS TO BAR EXAMINATION QUESTIONS by

the UP LAW COMPLEX (2007-2013)

&

PHILIPPINE ASSOCIATION OF LAW SCHOOLS (2008)


Never Let The Odds Keep You From Pursuing What You Know In Your Heart You Were Meant To Do.-
Leroy Satchel Paige
Page 1 of 180
Civil Law Q&As (2007-2013) hectorchristopher@yahoo.com dbaratbateladot@gmail.com

FOREWORD

This work is a compilation of the ANSWERS TO BAR


EXAMINATION QUESTIONS by the UP LAW COMPLEX ,
Philippine Association of Law Schools from 2007-2010 and
local law students and lawyers forum sites from 2011-2013
and not an original creation or formulation of the author.

The authors were inspired by the work of Silliman Universitys


College of Law and its students of producing a very good
material to everyone involved in the legal field particularly the
students and the reviewees for free. Hence, this work is a
freeware.

Everyone is free to distribute and mass produce copies of this


work, however, the author accepts no liability for the content of
this reviewer, or for the consequences of the usage, abuse, or
any actions taken by the user on the basis of the information
given.

The answers (views or opinions) presented in this reviewer are


solely those of the authors in the given references and do not
necessarily represent those of the authors of this work.

The Authors.
Never Let The Odds Keep You From Pursuing What You Know In Your Heart You Were Meant To Do.-
Leroy Satchel Paige
Page 2 of 180
Civil Law Q&As (2007-2013) hectorchristopher@yahoo.com dbaratbateladot@gmail.com

TABLE OF CONTENTS

(Titles are based on Sillimans Compilation [Arranged by Topic])

Persons

Capacity: Juridical Capacity (2008).......................................................................................................... 12

Capacity; Juridical Capacity of Donee; Requisites for Acceptance (2012)...............................12

Capacity: Legal Capacity; Lex Rei Sitae (2007).....................................................................................13

Correction of Entries; Clerical Error Act (2008)..................................................................................14

Nationality Principle (2009)...........................................................................................................................14

Nationality Principle; Change of Name not Covered (2009).............................................................15

Conflict of Laws

Processual Presumption (2009).................................................................................................................... 16

Jurisdiction; Courts may Assume Jurisdiction over Conflict of Laws Cases (2010)...........17

Adoption

Adoption; Termination; Death of Adopter (2009).................................................................................17

Adoption; Illegitimate Child (2010)............................................................................................................18

Adoption; Illegitimate Child; Use of Mothers Surname as Middle Name (2012)..................19

Consent; Consent of the Adopters Heirs (2008)..................................................................................19

Qualifications of Adopter (2010).................................................................................................................20


Never Let The Odds Keep You From Pursuing What You Know In Your Heart You Were Meant To Do.-
Leroy Satchel Paige
Page 3 of 180
Civil Law Q&As (2007-2013) hectorchristopher@yahoo.com dbaratbateladot@gmail.com

Family Code

Marriage; Annulment; Grounds (2009).....................................................................................................20

Marriage; Annulment; Grounds (2007).....................................................................................................21

Marriage; Annulment; Parties (2012).........................................................................................................22

Marriage; Annulment; Support Pendente Lite (2010).........................................................................22

Marriage; Divorce Decrees; Filipino Spouse Becoming Alien (2009)..........................................23

Marriage; Divorce Decrees; Foreign Spouse Divorces Filipino Spouse (2012)........................24

Marriage; Divorce Decrees; Foreign Spouse Divorces Filipino Spouse (2010)........................25

Marriage; Legal Separation; Prescription (2012)..................................................................................25

Marriage; Legal Separation; Prescription (2007)..................................................................................26

Marriage; Psychological Incapacity (2013).............................................................................................26

Marriage; Psychological Incapacity (2012).............................................................................................28

Marriage; Requisites (2008)...........................................................................................................................28

Marriage; Subsequent Marriage (2008).....................................................................................................29

Marriage; Void Marriages; By Reason of Public Policy (2008)........................................................30

Marriage; Void Marriages; By Reason of Public Policy (2007)........................................................30

Marriage; Void Marriages; Property Relations (2009).........................................................................30

Marriage; Void Marriages; Status of Children (2009)..........................................................................31

Parental Authority; Illegitimate Minor Child (2009)..........................................................................32

Parental Authority; In Vitro Fertilization (2010).................................................................................32


Never Let The Odds Keep You From Pursuing What You Know In Your Heart You Were Meant To Do.-
Leroy Satchel Paige
Page 4 of 180
Civil Law Q&As (2007-2013) hectorchristopher@yahoo.com dbaratbateladot@gmail.com

Paternity & Filiation; Child Born Under a Void Marriage (2010)................................................33

Paternity & Filiation; Impugning Legitimacy (2010)..........................................................................34

Paternity & Filiation; In Vitro Fertilization;

Surrogate Mothers Remedy to Regain Custody (2010).........................34

Paternity & Filiation; Legitimacy; Presumption (2008)................................................................... 35

Paternity & Filiation; Legitimation of a Child from a Previous Valid Marriage (2008)......36

Paternity & Filiation; Legitimation of a Dead Child (2009)............................................................37

Paternity & Filiation; Support: Ascendants & Descendants;

Collateral Blood Relatives (2008).................................................................... 37

Paternity & Filiation; Use of Surname; Illegitimate Child (2009)................................................38

Paternity & Filiation; Who May Impugn Legitimacy (2009)...........................................................39

Property Relations; Adulterous Relationship (2009)..........................................................................39

Property Relations; Accession (2012)........................................................................................................40

Property Relations; Ante-Nuptial Debt (2007)...................................................................................... 40

Property Relations; Unions Without Marriage (2012)........................................................................41

Property Relations; Unions Without Marriage (2012)........................................................................42

Property Relations; Void Marriages (2010).............................................................................................43

Property Relations; Void Marriages (2010).............................................................................................45


Never Let The Odds Keep You From Pursuing What You Know In Your Heart You Were Meant To Do.-
Leroy Satchel Paige
Page 5 of 180
Civil Law Q&As (2007-2013) hectorchristopher@yahoo.com dbaratbateladot@gmail.com

Succession

Disposition; Mortis Causa vs. Intervivos; Corpse (2009)................................................................. 46

Heirs; Fideicommissary Substitution (2008).........................................................................................46

Heirs; Intestate Succession; Legitime; Computation (2010)..........................................................47

Heirs; Representation; Iron-Curtain Rule (2012).................................................................................49

Heirs; Reserva Troncal (2009)......................................................................................................................49

Intestate Succession (2008)...........................................................................................................................50

Intestate Succession (2008)...........................................................................................................................51

Intestate Succession; Rights of Representation:

Illegitimate, Adopted Child; Iron Curtain Rule (2007).......................... 51

Legitimes; Compulsory Heirs (2012).........................................................................................................53

Legitime; Compulsory Heirs (2008)............................................................................................................53

Preterition; Disinheritance (2008).............................................................................................................54

Succession; Proof of Death between persons called to succeed each other (2008).............55

Succession; Rule on Survivorship (2009)...............................................................................................56

Wills; Holographic Wills; Insertions & Cancellations (2012)...........................................................57

Wills; Holographic Wills; Probate (2009).................................................................................................. 57

Wills; Joint Wills (2008)...................................................................................................................................59

Wills; Joint Wills; Probate (2012)................................................................................................................59


Never Let The Odds Keep You From Pursuing What You Know In Your Heart You Were Meant To Do.-
Leroy Satchel Paige
Page 6 of 180
Civil Law Q&As (2007-2013) hectorchristopher@yahoo.com dbaratbateladot@gmail.com

Wills; Prohibition to Partition of a Co-Owned Property (2010).....................................................60

Wills; Notarial Wills; Blind Testator; Requisites (2008).....................................................................61

Wills; Testamentary Disposition; Period to Prohibit Partition (2008)........................................61

Wills; Witnesses to a Will, Presence required; Thumbmark as Signature (2007)..................62

Donation

Donations; Formalities; In Writing (2007).............................................................................................. 63

Donations; Illegal & Impossible Conditions (2007).............................................................................64

Donation; Inter Vivos (2013)..........................................................................................................................64

Property

Accretion; Alluvium (2008)............................................................................................................................65

Accretion; Rights of the Riparian Owner (2009)..................................................................................67

Builder; Good Faith; Requisites (2013).....................................................................................................68

Easement; Prescription; Acquisitive Prescription (2009)................................................................70

Easement; Right of Way (2013)....................................................................................................................70

Easement; Right of Way (2010)....................................................................................................................72

Hidden Treasure (2008)...................................................................................................................................73

Mortgage; Public or Private Instrument (2013).....................................................................................74

Occupation vs. Possession (2007)...............................................................................................................76

Ownership; Co-Ownership (2009)................................................................................................................76

Ownership; Co-Ownership (2008)................................................................................................................77


Never Let The Odds Keep You From Pursuing What You Know In Your Heart You Were Meant To Do.-
Leroy Satchel Paige
Page 7 of 180
Civil Law Q&As (2007-2013) hectorchristopher@yahoo.com dbaratbateladot@gmail.com

Property; Movable or Immovable (2007)...................................................................................................78

Land Titles and Deeds

Acquisition of Lands; Sale of Real Property to an Alien (2009)....................................................79

Non-Registrable Properties (2007)...............................................................................................................80

Prescription; Acquisitive Prescription (2008)....................................................................................... 81

Prescription; Judicially Foreclosed Real Property Mortgage (2012)...........................................82

Purchaser in Good Faith; Mortgaged Property (2008)....................................................................... 83

Registration; Governing Law (2007)...........................................................................................................84

Registration; Party Who First took Possession (2013)......................................................................85

Registration; Requisites; Proof (2013)...................................................................................................... 86

Remedies; Fraud; Rights of Innocent Purchaser (2009)...................................................................89

Contracts

Contract to Sell vs. Conditional Contract of Sale (2012)................................................................90

Rescission of Contract; Fortuitous Event (2008).................................................................................90

Stipulation; Arbitration Clause (2009).......................................................................................................91

Obligations

Extinguishment; Compensation (2009)....................................................................................................91

Extinguishment; Compensation (2008)....................................................................................................92

Extinguishment; Novation (2008)...............................................................................................................93

Extinguishment; Payment of Check (2013)............................................................................................94


Never Let The Odds Keep You From Pursuing What You Know In Your Heart You Were Meant To Do.-
Leroy Satchel Paige
Page 8 of 180
Civil Law Q&As (2007-2013) hectorchristopher@yahoo.com dbaratbateladot@gmail.com

Extinguishment; Payment of Check; Legal Tender (2008)..............................................................95

Liability; Solidary Liability (2008).............................................................................................................. 96

Obligations; Without Agreement (2007)..................................................................................................97

Trust

Trust De Son Tort (2007)................................................................................................................................98

Sales

Condominium Act; Partition of a Condominium (2009)...................................................................99

Mortgage; Equitable Mortgage (2012)........................................................................................................99

Option Contract; Liquor & Pulutan as Consideration (2013)...................................................100

Right of First Refusal; Lessee; Effect (2008).......................................................................................101

Lease

Builder; Good Faith; Useful Improvements (2013)...........................................................................103

Lease; Caveat Emptor (2009)......................................................................................................................104

Agency

Agency; Sale of a Real Property through an Agent (2010)............................................................104

Partnership

Liability; Liability of a Partner (2010)....................................................................................................105

Oral Partnership (2009)................................................................................................................................106

Share; Demand during the Existence of Partnership (2012)........................................................107


Never Let The Odds Keep You From Pursuing What You Know In Your Heart You Were Meant To Do.-
Leroy Satchel Paige
Page 9 of 180
Civil Law Q&As (2007-2013) hectorchristopher@yahoo.com dbaratbateladot@gmail.com

Commodatum & Mutuum

Mutuum; Interest; Solutio Indebiti (2012)............................................................................................107

Guaranty

Guaranty (2009).................................................................................................................................................108

Surety

Surety (2010)......................................................................................................................................................108

Pledge

Pledge; Pactum Commissorium (2009)..................................................................................................109

Torts and Damages

Damages (2012).................................................................................................................................................109

Damages; Moral & Exemplary (2009)......................................................................................................110

Damages; Public Officers acting in the Performance of their Duties (2012).......................111

Death Indemnity (2009)................................................................................................................................ 112

Doctrine of Discovered Peril (Last Clear Chance) (2007)...............................................................112

Liability; Owner of a Pet; Fortuitous Event (2010)..........................................................................113

Liability; Special Parental Authority (2010).......................................................................................115

Quasi-Delict; Claims; Requisites (2013).................................................................................................115

Quasi Tort (2010)..............................................................................................................................................116


Never Let The Odds Keep You From Pursuing What You Know In Your Heart You Were Meant To Do.-
Leroy Satchel Paige
Page 10 of 180
Civil Law Q&As (2007-2013) hectorchristopher@yahoo.com dbaratbateladot@gmail.com

MULTIPLE CHOICE QUESTIONS

2013 Civil Law Exam MCQ (October 13, 2013)..............118

2012 Civil Law Exam MCQ (October 14, 2012).................130

2011 Civil Law Exam MCQ (November 13, 2011)..............149

2010 Civil Law Exam MCQ (September 12, 2010).............176

2007 Civil Law Exam MCQ (September 09, 2007).............179

Never Let The Odds Keep You From Pursuing What You Know In Your Heart You Were Meant To Do.-
Leroy Satchel Paige
Page 11 of 180
Civil Law Q&As (2007-2013) hectorchristopher@yahoo.com dbaratbateladot@gmail.com

that "for civil purposes, the fetus shall

be considered born if it is alive at the


Persons
time it is completely delivered from the

Capacity: Juridical Capacity (2008)

No. II. At age 18, Marian found out that she

was pregnant. She insured her own life and


named her unborn child as her sole

beneficiary. When she was already due to


give birth, she and her boyfriend Pietro, the

father of her unboarn child, were


kidnapped in a resort in Bataan where they

were vacationing. The military gave chase


and after one week, they were found in an

abandoned hut in Cavite. Marian and Pietro


were hacked with bolos. Marian and the

baby delivered were both found dead, with


the baby's umbilical cord already cut. Pietro

survived.

(A). Can Marian's baby be the beneficiary of


the insurance taken on the life of the

mother? (2%)

SUGGESTED ANSWER:

Yes, the baby can be the beneficiary of

the life insurance of Marian. Art. 40 NCC

provides that "birth determines

personality; but the conceived child

shall be considered born for all purposes

that are favorable to it, provided that it

be born later with the conditions

specified in Art. 41. Article 41 states


mother's womb. However, if the fetus
Capacity; Juridical Capacity of Donee;
had an intra-uterine life of less than
Requisites for Acceptance (2012)
seven months, it is not deemed born if it

dies within twenty-four (24) hours after


No.I. b) Ricky donated P 1 Million to the
its complete delivery from the maternal
unborn child of his pregnant girlfriend,
womb. The act of naming the unborn
which she accepted. After six (6) months of
child as sole beneficiary in the insurance pregnancy, the fetus was born and baptized
is favorable to the conceived child and as Angela. However, Angela died 20 hours

therefore the fetus acquires presumptive after birth. Ricky sought to recover the P 1

or provisional personality. However, said Million. Is Ricky entitled to recover?

presumptive personality only becomes Explain. (5%)

conclusive if the child is born alive. The


SUGGESTED ANSWER:
child need not survive for twenty-four

(24) hours as required under Art. 41 of Yes, Ricky is entitled to recover the

the Code because "Marian was already P1,000,000.00. The NCC considers a

due to give birth," indicating that the

child was more than seven months old.

Never Let The Odds Keep You From Pursuing What You Know In Your Heart You Were Meant To Do.-
Leroy Satchel Paige
Page 12 of 180
Civil Law Q&As (2007-2013) hectorchristopher@yahoo.com dbaratbateladot@gmail.com

the statement is FALSE, state the reason.


(2% each).
fetus a person for purposes favorable to

it provided it is born later in accordance


(1). Roberta, a Filipino, 17 years of age,
with the provision of the NCC. While the without the knowledge of his parents, can
donation is favorable to the fetus, the

donation did not take effect because the

fetus was not born in accordance with

the NCC.

To be considered born, the fetus that

had an intrauterine life of less than

seven (7) months should live for 24

hours from its complete delivery from

the mothers womb. Since Angela had an

intrauterine life of less than seven (7)

months but did not live for 24 hours, she

was not considered born and, therefore,

did not become a person. Not being a

person, she has no juridical capacity to

be a donee, hence, the donation to her

did not take effect. The donation not

being effective, the amount donated may

be recovered. To retain it will be unjust

enrichment.

Capacity: Legal Capacity; Lex Rei Sitae

(2007)

No.VII. Write "TRUE" if the statement is


true or "FALSE" if the statement is false. If
contract in Australia is governed by

acquire a house in Australia because Philippine Law, she will acquire

Australian Laws allow aliens to acquire ownership over the property bought

property from the age of 16. until the contract is annulled.

SUGGESTED ANSWER: ALTERNATIVE ANSWER:

TRUE. Since Australian Law allows alien FALSE. Laws relating to family rights

to acquire property from the age of 16, and duties, or to the status, condition or

Roberta may validly own a house in legal capacity of persons are binding

Australia, following the principle of lex upon the citizens of the Philippines,

rei sitae enshrined in Art. 16, NCC, even though living abroad (Art. 15, NCC).

which states "Real property as well as The age of majority under Philippine law

personal property is subject to the law of is 18 years (R.A. No. 6809); hence,

the country where it is situated." Roberta, being only 17 years old, has no

Moreover, even assuming that legal legal capacity to acquire and own land.

capacity of Roberta in entering the

Never Let The Odds Keep You From Pursuing What You Know In Your Heart You Were Meant To Do.-
Leroy Satchel Paige
Page 13 of 180
Civil Law Q&As (2007-2013) hectorchristopher@yahoo.com dbaratbateladot@gmail.com

that of Andy's to Aimee's maiden

Correction of Entries; Clerical Error Act surname in accordance with Rule 108 of

(2008)

No. IV. Gianna was born to Andy and

Aimee, who at the time Gianna's birth were


not married to each other. While Andy was

single at the time, Aimee was still in the


process of securing a judicial declaration of

nullity on her marriage to her ex-husband.


Gianna's birth certificate, which was signed

by both Andy and Aimee, registered the


status of Gianna as "legitimate", her

surname carrying that of Andy's and that


her parents were married to each other.

(A). Can a judicial action for correction of


entries in Gianna's birth certificate be
successfully maintained to:

a). Change her status from "legitimate" to


"illegitimate" (1%);

and

b). Change her surname from that of Andy's


to Aimee's maiden surname? (1%)

SUGGESTED ANSWER:

Yes, a judicial action for correction of

entries in Gianna's birth certificate can

be successfully maintained to change (a)

her status from "legitimate" to

"illegitimate," and (b) her surname from


nickname in the civil register without

the Rules of Court because said changes need of a judicial order. Errors that

are substantive corrections. involve the change of nationality, age,

status, surname or sex of petitioner are


(B). Instead of a judicial action, can not included from the coverage of the
administrative proceedings be brought for
said Act (Silverio v. Republic, G.R. No.
the purpose of making the above
174689, 22 Oct., 2007).
corrections? (2%)

SUGGESTED ANSWER:
Nationality Principle (2009)

No. An administrative proceeding cannot


No.XII. Emmanuel and Margarita, American
be brought for the purpose of making the
citizens and employees of the U.S. State
above corrections. R.A. 9048, otherwise
Department, got married in the African
known as the Clerical Error Act, which state of Kenya where sterility is a ground for
authorizes the city or municipal civil annulment of marriage. Thereafter, the
registrar or the consul general to correct spouses were assigned to the U.S. Embassy

a clerical or typographical error in an in Manila. On the first year of the spouses

entry and/or change the first name or

Never Let The Odds Keep You From Pursuing What You Know In Your Heart You Were Meant To Do.-
Leroy Satchel Paige
Page 14 of 180
Civil Law Q&As (2007-2013) hectorchristopher@yahoo.com dbaratbateladot@gmail.com

Recio, 366 SCRA 437 [2001], Llorente v.

tour of duty in the Philippines, Margarita Court of Appeals 345 SCRA 92 [2000],

filed an annulment case against Emmanuel and Bayot v. Court of Appeals 570 SCRA

before a Philippine court on the ground of 472 [2008]).

her husbands sterility at the time of the


celebration of the marriage.

(A). Will the suit prosper? Explain your


answer. (3%)

SUGGESTED ANSWER:

No, the suits will not prosper. As applied

to foreign nationals with the respect to

family relations and status of persons,

the nationality principle set forth in

Article 15 of the Civil Code will govern

the relations of Emmanuel and

Margarita. Since they are American

citizens, the governing law as to the

ground for annulment is not Kenyan Law

which Magarita invokes in support of

sterility as such ground; but should be

U.S. Law, which is the national Law of

both Emmanuel and Margarita as

recognized under Philippine Law. Hence,

the Philippine court will not give due

course to the case based on Kenyan Law.

The nationality principle as expressed in

the application of national law of foreign

nationals by Philippine courts is

established by precedents (Pilapil v. Ibay-

Somera, 174 SCRA 653[1989], Garcia v.


No.XX. (A). If Ligaya, a Filipino citizen

residing in the United States, files a petition


ALTERNATIVE ANSWER:
for change of name before the District Court
The forum has jurisdiction over an
of New York, what law shall apply? Explain.
action for the annulment of marriage
(2%)
solemnized elsewhere but only when the

party bringing the actions is domiciled SUGGESTED ANSWER:

in the forum. In this case, none of the New York law shall apply. The petition of

parties to the marriage is domiciled in change of name file din New York does

the Philippines. They are here as not concern the legal capacity or status

officials of the US Embassy whose stay in of the petitioner. Moreover, it does nto

the country is merely temporary, lasting affect the registry of any other country

only during their fixed tour of duty. including the country of birth of the

Hence, the Philippine courts have no petitioner. Whatever judgment is

jurisdiction over the action. rendered in that petition will have effect

only in New York. The New York court

Nationality Principle; Change of Name

not Covered (2009)

Never Let The Odds Keep You From Pursuing What You Know In Your Heart You Were Meant To Do.-
Leroy Satchel Paige
Page 15 of 180
Civil Law Q&As (2007-2013) hectorchristopher@yahoo.com dbaratbateladot@gmail.com

SUGGESTED ANSWER:

Philippine law will apply. The petition for


cannot, for instance, order the Civil
change of name in the Philippines will
Registrar in the Philippines to change its
affect only the records of the
records. The judgment of the New York

court allowing a change in the name of

the petitioner will be limited to the

records of the petitioner in New York and

the use of her new name in all

transactions in New York. Since the

records and processes in New York are

the only ones affected, the New York

court will apply New YorK law in

resolving the petition.

ALTERNATIVE ANSWER:

Philippine law shall apply (Art 15, NCC).

Status, conditions, family rights and

duties are governed by Philippine laws as

to Filipinos even though sojourning

abroad.

ALTENATIVE ANSWER:

If Ligaya, a Filipino, files a petition for

change of name with the District Court

of New YoRk, the laws of New York will

govern since change of name is not one

of those covered by the principles of

nationality.

(B). If Henry, an American citizen residing


in the Philippines, files a petition for change

of name before a Philippine court, what law


shall apply? Explain. (2%)
principle, by which his legal status is

petitioner and his transactions in the governed by national law, the matter of

Philippines. The Philippine court can change of name being included in the

never acquire jurisdiction over the legal status. The Supreme Court has

custodian in the US of the records of the reiterate in several cases, that the lex

petitioner. Moreover, change of name has patriae as provided in Article 15 of the

nothing to do with the legal capacity or Civil Code is applicable to foreign

status of the alien. Since Philippine nationals in determining their legal

records and transactions are the only status (supra).

ones affected, the Philippine court may

effect the change only in accordance Conflict of Laws


with the laws governing those records
Processual Presumption (2009)
and transactions that law cannot be but

Philippine law. No.I. TRUE or FALSE. Answer TRUE if the


ALTERNATIVE ANSWER: statement is true, or FALSE if the
U.S. law shall apply as it is his national statement is false. Explain your answer in

law. This is pursuant to the application not more than two (2) sentences.

of lex patriae or the nationality

Never Let The Odds Keep You From Pursuing What You Know In Your Heart You Were Meant To Do.-
Leroy Satchel Paige
Page 16 of 180
Civil Law Q&As (2007-2013) hectorchristopher@yahoo.com dbaratbateladot@gmail.com

ALTERNATIVE ANSWER:
(A). The doctrine of "processual

presumption" allows the court of the forum


to presume that the foreign law applicable

to the case is the same as the local or


domestic law. (1%)

SUGGESTED ANSWER:

TRUE. If the foreign law necessary to the

resolve an issue is not proven as a fact,

the court of the forum may presume that

the foreign law is the same as the law of

the forum.

Jurisdiction; Courts may Assume

Jurisdiction over Conflict of Laws Cases

(2010)

No.III. Define, Enumerate or Explain. (2%

each)

(C) Give at least two reasons why a court

may assume jurisdiction over a conflict of


laws case.

SUGGESTED ANSWER:

(1) Statute theory. There is a domestic

law authorizing the local court to

assume jurisdiction.

(2) Comity theory. The local court

assumes jurisdiction based on the

principle of comity or courtesy.


No.XIII. Rafael, a wealthy bachelor, filed a
(1) Public Order. To maintain peace and
petition for the adoption of Dolly, a one-year
order, disputes that disturb the peace of old foundling who had a severe heart ailment.
the forum should be settled by the court During the pendency of the adoption
of the forum even though the application proceedings, Rafael died of natural causes.

of the foreign law is necessary for the The Office of the Solicitor General files a

purpose. motion to dismiss the petition on the ground

that the case can no longer proceed because


(2) Humanitarian Principle. An aggrieved
of the petitioners death.
party should not be left without remedy

in a forum even though the application (A). Should the case be dismissed? Explain.
(2%)
of the foreign law by the courts of the

forum is unavoidable in order to extend


SUGGESTED ANSWER:
relief.
It depends on the stage of the

proceedings when Rafael died. If he died


Adoption
after all the requirements under the law

Adoption; Termination; Death of Adopter

(2009)

Never Let The Odds Keep You From Pursuing What You Know In Your Heart You Were Meant To Do.-
Leroy Satchel Paige
Page 17 of 180
Civil Law Q&As (2007-2013) hectorchristopher@yahoo.com dbaratbateladot@gmail.com

decree should be allowed in case it is the

have been complied with and the case is adoptee who dies because adoption is

already submitted for resolution, the primarily for his benefit.

court may grant the petition and issue a

decree of adoption despite the death of

the adopter (Section 13, RA 8552).

Otherwise, the death of the petitioner

shall have the effect terminating the

proceedings.

(B). Will your answer be the same if it was


Dolly who died during the pendency of the
adoption proceedings? Explain. (2%)

SUGGESTED ANSWER:

No, if it was Dolly who died, the case

should be dismissed. Her death

terminates the proceedings (Art. 13,

Domestic Adoption Law).

ALTERNATIVE ANSWER:

It depends. If all the requirements under

the law have already been complied with

and the case is already submitted for

resolution, the death of the adoptee

should not abate the proceedings. The

court should issue the decree of

adoption if will be for the best interest of

the adoptee. While RA8552 provides only

for the case where it is the petitioner

who dies before the decree is issued, it is

with more compelling reason that the


SUGGESTED ANSWER:

Adoption; Illegitimate Child (2010) The consent of the 14-year-old

legitimate child, of the 10- year -old


No.VIII. Spouses Rex and Lea bore two
illegitimate child and of the biological
children now aged 14 and 8. During the
subsistence of their marriage, Rex begot a mother of the illegitimate child are

child by another woman. He is now 10 needed for the adoption (Section 7 and

years of age. 9, RA 8552). The consent of Lea is no

longer required because there was


On Leas discovery of Rexs fathering a child
already a final decree of legal separation.
by another woman, she filed a petition for
legal separation which was granted. (B) If there was no legal separation, can Rex
still adopt his illegitimate child? Explain.
Rex now wants to adopt his illegitimate
(2.5%)
child.

SUGGESTED ANSWER:
(A) Whose consent is needed for Rexs
adoption of his illegitimate child? (2.5%) Yes, he can still adopt his illegitimate

child but with the consent of his spouse,

Never Let The Odds Keep You From Pursuing What You Know In Your Heart You Were Meant To Do.-
Leroy Satchel Paige
Page 18 of 180
Civil Law Q&As (2007-2013) hectorchristopher@yahoo.com dbaratbateladot@gmail.com

law is silent as to what middle name an

adoptee may use. In case of In re:


of his 14-year-old legitimate child, of the

illegitimate child, and of the biological

mother of the illegitimate child (Section

7 and 9, RA 8552).

Adoption; Illegitimate Child; Use of

Mothers Surname as Middle Name

(2012)

No.IV.b) Honorato filed a petition to adopt


his minor illegitimate child Stephanie,
alleging that Stephanies mother is Gemma

Astorga Garcia; that Stephanie has been


using her mothers middle name and

surname; and that he is now a widower and


qualified to be her adopting parent. He

prayed that Stephanies middle name be


changed from "Astorga" to "Garcia," which is

her mothers surname and that her


surname "Garcia" be changed to "Catindig,"

which is his surname. This the trial court


denied. Was the trial court correct in

denying Hororatos request for Stephanies


use of her mothers surname as her middle

name? Explain. (5%)

SUGGESTED ANSWER:

No, the trial court was not correct. There

is no law prohibiting an illegitimate child

adopted by his natural father to use as

middle name his mothers surname. The


Consent; Consent of the Adopters Heirs
Adoption of Stephanie Nathy Astorga
(2008)
Garcia, G.R. No. 148311, March 31,

2005, the Supreme Court ruled that the No.V. Despite several relationships with
adopted child may use the surname of different women, Andrew remained

the natural mother as his middle name unmarried. His first relationship with

because there is no prohibition in the Brenda produced a daughter, Amy, now 30

law against it. Moreover, it will also be years old. His second, with Carla, produced
two sons: Jon and Ryan. His third, with
for the benefit of the adopted child who
Donna, bore him no children although
shall preserve his lineage on his
Elena has a daughter Jane, from a previous
mothers side and reinforce his right to
relationship. His last, with Fe, produced no
inherit from his mother and her family.
biological children but they informally
Lastly, it will make the adopted child
adopted without court proceedings, Sandy's
conform with the time-honored Filipino
now 13 years old, whom they consider as
tradition of carrying the mothers their own. Sandy was orphaned as a baby
surname as the persons middle name. and was entrusted to them by the midwife

who attended to Sandy's birth. All the

Never Let The Odds Keep You From Pursuing What You Know In Your Heart You Were Meant To Do.-
Leroy Satchel Paige
Page 19 of 180
Civil Law Q&As (2007-2013) hectorchristopher@yahoo.com dbaratbateladot@gmail.com

including Amy and Elena obtains the

children, including Amy, now live with written consent of Jane, if she is over

andrew in his house. ten years old (Sec. 9(d), RA 8552).

(A). Is there any legal obstacle to the legal


adoption of Amy by Andrew? To the legal

adoption of Sandy by Andrew and Elena?


(2%)

SUGGESTED ANSWER:

Yes, there is a legal obstacle to the legal

adoption of Amy by Andrew. Under Sec.

9(d) of RA 8552, the New Domestic

Adoption Act of 1998, the written

consent of the illegitimate

sons/daughters, ten (10) years of age or

over, of the adopter, if living with said

adopter and the latter's spouse, if any, is

necessary to the adoption. All the

children of Andrew are living with him.

Andrew needs to get the written consent

of Jon, Ryan, Vina and Wilma, who are

all ten (10) years old or more. Sandy's

consent to Amy's adoption is not

necessary because she was not legally

adopted by Andrew. Jane's consent is

likewise not necessary because she is

not a child of Andrew. Sandy, an orphan

since birth, is eligible for adoption under

Sec. 8(f) of RA 8552, provided that

Andrew obtains the written consent of

the other children mentioned above,


on certain situations enumerated in the

Qualifications of Adopter (2010) law. The case of John does not fall in any

of the exceptions (R.A. 8552).


No.IX. Eighteen-year old Filipina Patrice
Family Code
had a daughter out of wedlock whom she

named Laurie. At 26, Patrice married


Marriage; Annulment; Grounds (2009)
American citizen John who brought her to

live with him in the United States of


No.XII. Emmanuel and Margarita, American
America. John at once signified his
citizens and employees of the U.S. State
willingness to adopt Laurie.
Department, got married in the African
state of Kenya where sterility is a ground for
Can John file the petition for adoption? If
annulment of marriage. Thereafter, the
yes, what are the requirements? If no, why?
spouses were assigned to the U.S. Embassy
(5%)
in Manila. On the first year of the spouses

SUGGESTED ANSWER: tour of duty in the Philippines, Margarita

filed an annulment case against Emmanuel


No, John cannot file the petition to
before a Philippine court on the ground of
adopt alone. Philippine law requires

husband and wife to adopt jointly except

Never Let The Odds Keep You From Pursuing What You Know In Your Heart You Were Meant To Do.-
Leroy Satchel Paige
Page 20 of 180
Civil Law Q&As (2007-2013) hectorchristopher@yahoo.com dbaratbateladot@gmail.com

the formal validity of such marriage is

her husbands sterility at the time of the governed by Kenyan law and any issue as

celebration of the marriage. to the formal validity of that marriage

(B). Assume Emmanuel and Margarita are


both Filipinos. After their wedding in

Kenya, they come back and take up


residence in the Philippines. Can their

marriage be annulled on the ground of

Emmanuels sterility? Explain. (3%)

SUGGESTED ANSWER:

No, the marriage cannot be annulled

under the Philippine law. Sterility is not

a ground for annulment of marriage

under Article 45 of the Family Code.

ALTERNATIVE ANSWER:

No, the marriage cannot be annulled in

the Philippines.

The Philippine court shall have

jurisdiction over the action to annul the

marriage not only because the parties

are residents of the Philippines but

because they are Filipino citizens. The

Philippine court, however, shall apply

the law of the place where the marriage

was celebrated in determining its formal

validity (Article 26, FC; Article 17, NCC).

Since the marriage was celebrated in

Kenya in accordance with Kenyan law,


Since sterility does not constitute

shall be determined by applying Kenyan absence or defect in the legal capacity of

law and not Philippine law. the parties under Philippine law, there is

no ground to avoid or annul the

However, while Kenyan law governs the marriage. Hence, the Philippine court

formal validity of the marriage, the legal has to deny the petition.

capacity of the Filipino parties to the

marriage is governed not by Kenyan law

but by Philippine law (Article 15, NCC).


Marriage; Annulment; Grounds (2007)
Sterility of a party as a ground for the

annulment of the marriage is not a No. VII. Write "TRUE" if the statement is

matter of form but a matter of legal true or "FALSE" if the statement is false. If

capacity. Hence, the Philippine court the statement is FALSE, state the reason.
(2% each).
must apply Phillippine law in

determining the status of the marriage


(4). The day after John and Marsha got
on the ground of absence or defect in the
married, John told her that he was
legal capacity of the Filipino parties. impotent. Marsha continued to live with

Never Let The Odds Keep You From Pursuing What You Know In Your Heart You Were Meant To Do.-
Leroy Satchel Paige
Page 21 of 180
Civil Law Q&As (2007-2013) hectorchristopher@yahoo.com dbaratbateladot@gmail.com

declaration of nullity may only be filed

John for 2 years. Marsha is now estopped by any of the spouses.

from filing an annulment case against


ALTERNATIVE SUGGESTED ANSWER:
John.

SUGGESTED ANSWER:

FALSE. Marsha is not estopped from

filing an annulment case against John on

the ground of his impotence, because

she learned of his impotence after the

celebration of the marriage and not

before. Physical incapacity to

consummate is a valid ground for the

annulment of marriage if such incapacity

was existing at the time of the marriage,

continues and appears to be incurable.

The marriage may be annulled on this

ground within five years from its

celebration.

Marriage; Annulment; Parties (2012)

No.IX.b) A petition for declaration of nullity

of a void marriage can only be filed by


either the husband or the wife? Do you

agree? Explain your answer. (5%)

SUGGESTED ANSWER:

Yes, I agree. Under the rules promulgated

by the Supreme Court, a direct action for


two business firms to G and their two
children, aged 18 and 19.
No, I do not agree. There are others who

may file a petition for declaration of


B also opened a bank account in the
nullity such as the other spouse in amount of P3 million in the name of the two
bigamous marriages. children to answer for their educational

expenses until they finish their college


degrees.

Marriage; Annulment; Support Pendente


For her part, G undertook to shoulder the
Lite (2010)
day-to-day living expenses and upkeep of
the children. The Court approved the
No.V. G filed on July 8, 2000 a petition for
spouses agreement on September 8, 2000.
declaration of nullity of her marriage to B.
During the pendency of the case, the
(A) Suppose the business firms suffered
couple entered into a compromise
reverses, rendering G unable to support
agreement to dissolve their absolute
herself and the children. Can G still ask for
community of property. B ceded his right to
support pendente lite from B? Explain. (3%)
their house and lot and all his shares in

Never Let The Odds Keep You From Pursuing What You Know In Your Heart You Were Meant To Do.-
Leroy Satchel Paige
Page 22 of 180
Civil Law Q&As (2007-2013) hectorchristopher@yahoo.com dbaratbateladot@gmail.com

SUGGESTED ANSWER: Marriage; Divorce Decrees; Filipino

Spouse Becoming Alien (2009)


If B acquiesces and does not file the

action to impugn the legitimacy of the No.IV. Harry married Wilma, a very wealthy
child within the prescriptive period for woman. Barely five (5) years into the

doing so in Article 170 of the Family marriage, Wilma fell in love with Joseph.

Code, G's daughter by another man shall Thus, Wilma went to a small country in

Europe, became a naturalized citizen of


be conclusively presumed as the
that country, divorced Harry, and married
legitimate daughter of B by G.
Joseph. A year thereafter, Wilma and

(B) Suppose in late 2004 the two children Joseph returned and established

had squandered the P3 million fund for permanent residence in the Philippines.

their education before they could obtain


(A). Is the divorce obtained by Wilma from
their college degrees, can they ask for more
Harry recognized in the Philippines?
support from B? Explain. (3%)
Explain your answer. (3%)

SUGGESTED ANSWER:
SUGGESTED ANSRWER :
Yes, the two children can still ask for

support for schooling or training for As to Wilma, the divorced obtained by

some professions, trade or vocation, her is recognized as valid in the

even beyond the age of majority until Philippines because she is now a

they shall have finished or completed foreigner. Philippine personal laws do

their education (Article 194, Paragraph not apply to a foreigner. However,

2, Family Code; Javier v. Lucero, 94 Phil. recognition of the divorce as regards

634 {1954}].Their having squandered the Harry will depend on the applicability to

money given to them for their education his case of the second paragraph of

will not deprive them of their right to Article 26 of the Family Code. If it is

complete an education, or to extinguish applicable, divorce is recognized as to

the obligation of the parents to ensure him and, therefore, he can remarry.

the future of their children. However, if it is not applicable, divorce is

not recognized as to him and,

consequently, he cannot remarry.


ALTERNATIVE ANSWER:

Never Let The Odds Keep You From Pursuing What You Know In Your Heart You Were Meant To Do.-
Leroy Satchel Paige
Page 23 of 180
Civil Law Q&As (2007-2013) hectorchristopher@yahoo.com dbaratbateladot@gmail.com

Wilma is already married to Joseph. Can


Harry legally marry Elizabeth? Explain.
Yes , the divorce obtained by Wilma is
(2%)
recognized as valid in the Philippines. At

the time she got the divorce, she was

already a foreign national having been

naturalized as a citizen of that small

country in Europe. Based on precedents

established by the Supreme Court

( Bayot v. CA, 570 SCRA 472 [2008]),

divorce obtained by a foreigner is

recognized in the Philippines if validly

obtained in accordance with his or her

national law .

(B). If Harry hires you as his lawyer, what


legal recourse would you advise him to
take? Why? (2%)

SUGGESTED ANSWER:

I will advice Harry to:

(1) Dissolve and liquidate his property

relations with Wilma ; and

(2) If he will remarry, file a petition for

the recognition and enforcement of the

foreign judgment of divorced (Rule

39,Rules of Court ).

(C). Harry tells you that he has fallen in

love with another woman, Elizabeth, and


wants to marry her because, after all,
In Republic v. Obrecido, however, the
SUGGESTED ANSWER :
Supreme Court ruled that a Filipino

Yes, he can validly marry Elizabeth, spouse is given the capacity to remarry

applying the doctrine laid down by the even though the spouse who obtained

Supreme Court in Republic v. Obrecido the divorce was a Filipino at the time of

(427 SCRA 114 [2005]). Under the second the marriage, if the latter was already a

paragraph of Article 26 of the Family foreigner when the divorce was already

Code, for the Filipino spouse to have obtained abroad. According to the court,

capacity to remarry, the law expressly to rule otherwise will violate the equal

requires the spouse who obtained the protection clause of the Constitution.

divorce to be a foreigner at the time of

the marriage. Applying this requirement


Marriage; Divorce Decrees; Foreign
to the case of Harry it would seem that
Spouse Divorces Filipino Spouse (2012)
he is not given the capacity to remarry.

This is because Wilma was a Filipino at (b) Cipriano and Lady Miros married each

the time of her marriage to Harry. other. Lady Miros then left for the US and
there, she obtained American citizenship.

Never Let The Odds Keep You From Pursuing What You Know In Your Heart You Were Meant To Do.-
Leroy Satchel Paige
Page 24 of 180
Civil Law Q&As (2007-2013) hectorchristopher@yahoo.com dbaratbateladot@gmail.com

(A). Under Article 26 of the Family Code,


when a foreign spouse divorces his/her
Cipriano later learned all about this
Filipino spouse, the latter may re-marry by
including the fact that Lady Miros has
divorced him in America and that she had

remarried there. He then filed a petition for


authority to remarry, invoking Par. 2, Art.

26 of the Family Code. Is Cipriano


capacitated to re-marry by virtue of the

divorce decree obtained by his Filipino


spouse who was later naturalized as an

American citizen? Explain. (5%)

SUGGESTED ANSWER:

Yes, he is capacitated to remarry. While

the second paragraph of Art 26 of the

Family Code is applicable only to a

Filipino who married a foreigner at the

time of marriage, the Supreme Court

ruled in the case of Republic v. Orbecido,

G.R. No. 154380, 5 Oct, 2005, that the

said provision equally applies to a

Filipino who married another Filipino at

the time of the marriage, but who was

already a foreigner when the divorce was

obtained.

Marriage; Divorce Decrees; Foreign

Spouse Divorces Filipino Spouse (2010)

No.I. True or False.


abroad by the foreigner spouse is

proving only that the foreign spouse has sufficient to capacitate the Filipino

obtained a divorce against her or him spouse to remarry.

abroad. (1%)

SUGGESTED ANSWER :
Marriage; Legal Separation; Prescription
FALSE, In Garcia v. Recio , 366 SCRA
(2012)
437 (2001) , the SC held that for a

Filipino spouse to have capacity to No.IV.a) After they got married, Nikki

contract a subsequent marriage, it must discovered that Christian was having an

affair with another woman. But Nikki


also be proven that the foreign divorced
decided to give it a try and lived with him
obtained abroad by the foreigner spouse
for two (2) years. After two (2) years, Nikki
give such foreigner spouse capacity to
filed an action for legal separation on the
remarry.
ground of Christians sexual infidelity. Will

ALTERNATIVE ANSWER: the action prosper? Explain. (5%)

TRUE, Art 26 (2) (FC), clearly provides

that the decree of divorce obtained

Never Let The Odds Keep You From Pursuing What You Know In Your Heart You Were Meant To Do.-
Leroy Satchel Paige
Page 25 of 180
Civil Law Q&As (2007-2013) hectorchristopher@yahoo.com dbaratbateladot@gmail.com

FALSE. The five-year prescriptive period

SUGGESTED ANSWER: for filing legal separation runs from the

occurrence of sexual infidelity


Although the action for legal separation
committed in 2002 runs from 2002, for
has not yet prescribed, the prescriptive

period being 5 years, if Obecidos affair

with another woman was ended when

Nikki decided to live with him again,

Nikkis action will not prosper on

account of condonation. However, if

such affair is still continuing, Nikkis

action would prosper because the action

will surely be within five (5) years from

the commission of the latest act of

sexual infidelity. Every act of sexual

liaison is a ground for legal separation.

Marriage; Legal Separation; Prescription

(2007)

No.VII. Write "TRUE" if the statement is


true or "FALSE" if the statement is false. If

the statement is FALSE, state the reason.


(2% each).

(2). If a man commits several acts of sexual

infidelity, particularly in 2002, 2003, 2004,


2005, the prescriptive period to file for legal

separation runs from 2002.

SUGGESTED ANSWER:
drunkard, a womanizer, a gambler, and a
mama's boy- traits that she never knew or
the sexual infidelity committed in 2003,
saw when Neil was courting her. Although
the prescriptive period runs from 2003
summoned, Neil did not answer Maria's
and so on. The action for legal
petition and never appeared in court.
separation for the last act of sexual

infidelity in 2005 will prescribe in 2010. To support her petition, Maria presented
three witnesses- herself, Dr. Elsie Chan,

and Ambrosia. Dr. Chan testified on the


psychological report on Neil that she
Marriage; Psychological Incapacity
prepared. Since Neil never acknowledged
(2013)
n9r responded to her invitation for

No.I. You are a Family Court judge and interviews, her report is solely based on her

before you is a Petition for the Declaration interviews with Maria and the spouses'

of Nullity of Marriage (under Article 36 of minor children. Dr. Chan concluded that

the Family Code)filed by Maria against Neil. Neil is suffering from Narcissistic

Maria claims that Neil is psychologically Personality Disorder, an ailment that she

incapacitated to comply with the essential found to be already present since Neil's

obligations of marriage because Neil is a

Never Let The Odds Keep You From Pursuing What You Know In Your Heart You Were Meant To Do.-
Leroy Satchel Paige
Page 26 of 180
Civil Law Q&As (2007-2013) hectorchristopher@yahoo.com dbaratbateladot@gmail.com

serious enough to prevent Neil from

early adulthood and one that is grave and performing his essential marital

incurable. Maria testified on the specific obligations. Dr. Chans report contains

instances when she found Neil drunk, with

another woman, or squandering the


family's resources in a casino. Ambrosia,

the spouses' current household help,


corroborated Maria's testimony.

On the basis of the evidence presented, will


you grant the petition? (8%)

SUGGESTED ANSWER:

No. The petition should be denied.

The psychological incapacity under Art.

36 of the Family Code must be

characterized by (a) gravity, (b) juridical

antecedence, and (c) incurability. It is

not enough to prove that the parties

failed to meet their responsibilities and

duties as married persons; it is essential

that they must be shown to be incapable

of doing so, due to some physiological

(not physical) illness (Republic v. CA and

Molina, G.R. No. 108763, Feb 13, 1997).

In this case, the pieces of evidence

presented are not sufficient to conclude

that indeed Neil is suffering from

psychological incapacity [Narcissistic

Personality Disorder] existing already

before the marriage, incurable and


Yes. The petition should be granted.
mere conclusions. Being a drunkard, a

womanizer, a gambler and a mamas boy,


The personal medical or psychological
merely shows Neils failure to perform
examination of respondent is not a
his marital obligations. In a number of
requirement for declaration of
cases, the Supreme Court did not find
psychological incapacity. It is the
the existence of psychological incapacity
totality of the evidence presented which
in cases where the respondent showed
shall determine the existence of
habitual drunkenness (Republic v.
psychological incapacity (Marcos v.
Melgar, G.R. No. 139676, 2006), blatant
Marcos, G.R. No. 136490, Oct 19, 2000).
display of infidelity and irresponsibility
Dr. Chans report corroborated by Marias
(Dedel v. CA, 2004) or being hooked to
and Ambrosias testimonies, therefore,
gambling and drugs (Republic v. Tanyag-
sufficiently prove Neils psychological
San Jose, G.R. No. 168328, 2007).
incapacity to assume his marital

obligations.
ALTERNATIVE ANSWER:

Never Let The Odds Keep You From Pursuing What You Know In Your Heart You Were Meant To Do.-
Leroy Satchel Paige
Page 27 of 180
Civil Law Q&As (2007-2013) hectorchristopher@yahoo.com dbaratbateladot@gmail.com

cannot be annulled on the ground of

Marriage; Psychological Incapacity respondents psychological incapacity if

(2012)

No.II.b) The petitioner filed a petition for

declaration of nullity of marriage based


allegedly on the psychological incapacity of

the respondent, but the psychologist was


not able to personally examine the

respondent and the psychological report


was based only on the narration of

petitioner. Should the annulment be


granted? Explain. (5%)

SUGGESTED ANSWER:

The annulment cannot be guaranteed

solely on the basis of the psychological

report. For the report to prove the

psychological incapacity of the

respondent, it is required that the

psychologist should personally examine

the respondent and the psychological

report should be based on the

psychologists independent assessment

of the facts as to whether or not the

respondent is psychologically

incapacitated.

Since, the psychologist did not

personally examine the respondent, and

his report is based solely on the story of

the petitioner who has an interest in the

outcome of the petition, the marriage


When Faye was 25 years old, Brad discovered

her continued liason with Roderick and in one


the said report is the only evidence of
of their heated arguments, Faye shot Brad to
respondents psychological incapacity.
death. She lost no time in marrying her true

love Roderick, without a marriage license,

Marriage; Requisites (2008) claiming that they have been continuously

cohabiting for more than 5 years.


No. III. Roderick and Faye were high school

sweethearts. When Roderick was 18 and Faye, (A). Was the marriage of Roderick and Faye

16 years old, they started to live together as valid? (2%)

husband and wife without the benefit of


SUGGESTED ANSWER:
marriage. When Faye reached 18 years of age,

her parents forcibly took her back and


No. The marriage of Roderick and Faye is
arranged for her marriage to Brad. Although
not valid. Art. 4, FC provides that the
Faye lived with Brad after the marriage,
absence of any of the essential or formal
Roderick continued to regularly visit Faye
requisites renders the marriage void ab
while Brad was away at work. During their
initio. However, no license shall be
marriage, Faye gave birth to a baby girl, Laica.

Never Let The Odds Keep You From Pursuing What You Know In Your Heart You Were Meant To Do.-
Leroy Satchel Paige
Page 28 of 180
Civil Law Q&As (2007-2013) hectorchristopher@yahoo.com dbaratbateladot@gmail.com

a divorced German national born of a


German father and a Filipino mother
necessary for the marriage of a man and
residing in Stuttgart. To avoid being
a woman who have lived together as

husband and wife for at least 5 years and

without any legal impediment to marry

each other. In Republic v. Dayot, G.R.

No. 175581, 28 March 2008, reiterating

the doctrine in Nial v. Bayadog, G.R.

No. 133778, 14 March 2000, this five-

year period is characterized by

exclusivity and continuity. In the

present case, the marriage of Roderick

and Faye cannot be considered as a

marriage of exceptional character,

because there were 2 legal impediments

during their cohabitation: minority on

the part of Faye, during the first two

years of cohabitation; and, lack of legal

capacity, since Faye married Brad at the

age of 18. The absence of a marriage

license made the marriage of Faye and

Roderick void ab initio.

Marriage; Subsequent Marriage (2008)

No. I. Ana Rivera had a husband, a Filipino

citizen like her, who was among the


passengers on board a commercial jet plane

which crashed in the Atlantic Ocean ten


(10) years earlier and had never been heard

of ever since. Believing that her husband


had died, Ana married Adolf Cruz Staedtler,
subsistence of his previous marriage

reqiured to submit the required certificate provided that: (a) his prior spouse in the

of capacity to marry from the German first marriage had been absent for four

Embassy in Manila, Adolf stated in the consecutive years; (b) that the spouse

application for marriage license that he was present has a well-founded belief that
a Filipino citizen. With the marriage license the absent spouse was already dead, and
stating that Adolf was a Filipino, the couple
(C) present spouse instituted a summary
got married in a ceremony officiated by the
proceeding for the declaration of the
Parish Priest of Calamba, Laguna in a
presumptive death of absent spouse.
beach in Nasugbu, Batangas, as the local
Otherwise, the second marriage shall be
parish priest refused to solemnize
null and void. In the instant case, the
marriages except in his church. Is the
husband of Ana was among the
marriage valid? Explain fully. (5%)
passengers on board a commercial jet
SUGGESTED ANSWER: plane which crashed in the Atlantic

Ocean. The body of the deceased


No. The marriage is not valid. Art. 41 FC
husband was not recovered to confirm
allows the present spouse to contract a
his death. Thus, following Art. 41, Ana
subsequent marriage during the

Never Let The Odds Keep You From Pursuing What You Know In Your Heart You Were Meant To Do.-
Leroy Satchel Paige
Page 29 of 180
Civil Law Q&As (2007-2013) hectorchristopher@yahoo.com dbaratbateladot@gmail.com

Yes. Jon and Jane can marry each other;


should have first secured a judicial
Jon is an illegitimate child of Andrew
declaration of his presumptive death

before she married Adolf. The absence of

the said judicial declaration

incapacitated Ana from contracting her

second marriage, making it void ab

initio.

Marriage; Void Marriages; By Reason of

Public Policy (2008)

No.V. Despite several relationships with


different women, Andrew remained

unmarried. His first relationship with


Brenda produced a daughter, Amy, now 30

years old. His second, with Carla, produced


two sons: Jon and Ryan. His third, with

Donna, bore him no children although


Elena has a daughter Jane, from a previous

relationship. His last, with Fe, produced no


biological children but they informally

adopted without court proceedings, Sandy's


now 13 years old, whom they consider as

their own. Sandy was orphaned as a baby


and was entrusted to them by the midwife

who attended to Sandy's birth. All the


children, including Amy, now live with

andrew in his house.

(D). Can Jon and Jane legally marry? (1%)

SUGGESTED ANSWER:
20 years old. David had a son, Julian, with
his ex-girlfriend Sandra. Julian and Thelma
while Jane is a child of Elena from a
can get married.
previous relationship. Thus, their

marriage is not one of the prohibited SUGGESTED ANSWER:


marriages enumerated under Art. 38 of

the FC. TRUE. Julian and Thelma can get

married. Marriage between stepbrothers

and stepsisters are not among the

marriages prohibited under the Family


Marriage; Void Marriages; By Reason of
Code.
Public Policy (2007)
No. VII. Write "TRUE" if the statement is
true or "FALSE" if the statement is false. If
Marriage; Void Marriages; Property
the statement is FALSE, state the reason.
Relations (2009)
(2% each).

No. III. In December 2000, Michael and


(5). Amor gave birth to Thelma when she
Anna, after obtaining a valid marriage
was 15 years old. Thereafter, Amor met
David and they got married when she was

Never Let The Odds Keep You From Pursuing What You Know In Your Heart You Were Meant To Do.-
Leroy Satchel Paige
Page 30 of 180
Civil Law Q&As (2007-2013) hectorchristopher@yahoo.com dbaratbateladot@gmail.com

of the Family Code. This is on the

license, went to the Office of the Mayor of assumption that there was no

Urbano, Bulacan, to get married. The


Mayor was not there, but the Mayors

secretary asked Michael and Anna and their

witnesses to fill up and sign the required

marriage contract forms. The secretary then

told them to wait, and went out to look for the

Mayor who was attending a wedding in a

neighboring municipality.

When the secretary caught up with the

Mayor at the wedding reception, she showed


him the marriage contract forms and told

him that the couple and their witnesses


were waiting in his office. The Mayor

forthwith signed all the copies of the


marriage contract, gave them to the

secretary who returned to the Mayors


office. She then gave copies of the marriage

contract to the parties, and told Michael


and Anna that they were already married.

Thereafter, the couple lived together as


husband and wife, and had three sons.

(C). What property regime governs the

properties acquired by the couple? Explain.


(2%)

SUGGESTED ANSWER:

The marriage being void, the property

relationship that governed their union is

special co-ownership under Article 147


When the secretary caught up with the
impediment for them to validity marry
Mayor at the wedding reception, she showed
each other. him the marriage contract forms and told
him that the couple and their witnesses

were waiting in his office. The Mayor


Marriage; Void Marriages; Status of
forthwith signed all the copies of the
Children (2009)
marriage contract, gave them to the
secretary who returned to the Mayors
No. III. In December 2000, Michael and
office. She then gave copies of the marriage
Anna, after obtaining a valid marriage
contract to the parties, and told Michael
license, went to the Office of the Mayor of
and Anna that they were already married.
Urbano, Bulacan, to get married. The
Thereafter, the couple lived together as
Mayor was not there, but the Mayors
husband and wife, and had three sons.
secretary asked Michael and Anna and their

witnesses to fill up and sign the required


(A). Is the marriage of Michael and Anna
marriage contract forms. The secretary then
valid, voidable, or void? Explain your
told them to wait, and went out to look for the
answer. (3%)
Mayor who was attending a wedding in a

neighboring municipality.

Never Let The Odds Keep You From Pursuing What You Know In Your Heart You Were Meant To Do.-
Leroy Satchel Paige
Page 31 of 180
Civil Law Q&As (2007-2013) hectorchristopher@yahoo.com dbaratbateladot@gmail.com

representing that he was single when, in


fact, he was not; and exemplary, to teach a
SUGGESTED ANSWER :
lesson to like-minded Lotharios.

The marriage is void because the formal

requisite of marriage ceremony was

absent ( Art.3, F.C. 209, Family Code).

ALTERNATIVE ANSWER:

The marriage is void because an

essential requisite was absent: consent

of the parties freely given in the

presence of the solemnizing officer (Art

.2, FC).

(B). What is the status of the three children


of Michael and Anna? Explain your answer.
(2%)

SUGGESTED ANSWER:

The children are illegitimate, having

been born outside a valid marriage.

Parental Authority; Illegitimate Minor

Child (2009)

No.XIV. Rodolfo, married to Sharon, had an


illicit affair with his secretary, Nanette, a

19-year old girl, and begot a baby girl,


Rona. Nanette sued Rodolfo for damages:

actual, for hospital and other medical


expenses in delivering the child by

caesarean section; moral, claiming that


Rodolfo promised to marry her,
and even though he is giving support for

(C). When Rona reaches seven (7) years old, the child. To acquire custody over Rona,

she tells Rodolfo that she prefers to live Rodolfo should first deprive Nanette of

with him, because he is better off parental authority if there is ground

financially than Nanette. If Rodolfo files an under the law, and in a proper court
action for the custody of Rona, alleging that proceedings. In the same action, the
he is Ronas choice as custodial parent, will
court may award custody of Rona to
the court grant Rodolfos petition? Why or
Rodolfo if it is for her best interest.
why not? (2%)

SUGGESTED ANSWER:
Parental Authority; In Vitro Fertilization
No, because Rodolfo has no parental
(2010)
authority over Rona. He who has the

parental authority has the right to No.VI. Gigolo entered into an agreement
custody. Under the Family Code, the with Majorette for her to carry in her womb

mother alone has parental authority over his baby via in vitro fertilization. Gigolo

the illegitimate child. This is true even if undertook to underwrite Majorettes pre-

illegitimate father recognized the child natal expenses as well as those attendant

Never Let The Odds Keep You From Pursuing What You Know In Your Heart You Were Meant To Do.-
Leroy Satchel Paige
Page 32 of 180
Civil Law Q&As (2007-2013) hectorchristopher@yahoo.com dbaratbateladot@gmail.com

The relationship produced one offspring,


Venus. The couple acquired a residential lot
to her delivery. Gigolo would thereafter pay
Majorette P2 million and, in return, she
would give custody of the baby to him.

After Majorette gives birth and delivers the

baby to Gigolo following her receipt of P2


million, she engages your services as her

lawyer to regain custody of the baby.

(C) Who of the two can exercise parental


authority over the child? Explain. (2.5%)

SUGGESTED ANSWER:

Majorette, the mother, can exercise

parental authority. Since the child was

born out of wedlock, the child is

illegitimate and the mother has the

exclusive parental authority and custody

over the child.

ALTERNATIVE ANSWER:

Gigolo can exercise parental authority

over the child. Majorette has no blood

relation to the child. She is just a

carrier of the child.

Paternity & Filiation; Child Born Under a

Void Marriage (2010)

No.X. In 1997, B and G started living


together without the benefit of marriage.
legitimated by the subsequent marriage

in Paraaque. After four (4) years or in of her parents, such legitimation was

2001, G having completed her 4-year rendered ineffective when the said

college degree as a fulltime student, she marriage was later on declared null and
and B contracted marriage without a void due to absence of a marriage
license. license.

The marriage of B and G was, two years Under Article 178 of the Family Code,
later, declared null and void due to the legitimation shall take place by a
absence of a marriage license. subsequent valid marriage between

parents. The annulment of a voidable


(B). Is Venus legitimate, illegitimate, or
marriage shall not affect the
legitimated? Explain briefly. (3%)
legitimation. The inclusion of the
SUGGESTED ANSWER: underscored portion in the Article

necessarily implies that the Article's


Venus is illegitimate. She was conceived
application is limited to voidable
and born outside a valid marriage. Thus,
marriages. It follows that when the
she is considered illegitimate (Art 165,

Family Code). While Venus was

Never Let The Odds Keep You From Pursuing What You Know In Your Heart You Were Meant To Do.-
Leroy Satchel Paige
Page 33 of 180
Civil Law Q&As (2007-2013) hectorchristopher@yahoo.com dbaratbateladot@gmail.com

presumed as the child of B under Article

subsequent marriage is null or void, the 164 of the Family Code. In the same

legitimation must also be null and void.

In the present problem, the marriage

between B and G was not voidable but

void. Hence, Venus has remained an

illegitimate child.

Paternity & Filiation; Impugning

Legitimacy (2010)

No.IV. Spouses B and G begot two

offsprings. Albeit they had serious


personality differences, the spouses

continued to live under one roof. B begot a


son by another woman. G also begot a

daughter by another man.

(A). If G gives the surname of B to her


daughter by another man, what can B do to

protect their legitimate children's interests?


Explain. (5%)

SUGGESTED ANSWER:

B can impugn the status of G's daughter

by another man as his legitimate

daughter on the ground that for

biological reason he could not have been

the father of the child, a fact that may

be proven by the DNA test. Having been

born during the marriage between B and

G, G's daughter by another man is


action to impugn, B can pray for the

correction of the status of the said Paternity & Filiation; In Vitro

daughter in her record of birth. Fertilization; Surrogate Mothers

Remedy to Regain Custody (2010)


(B). If B acquiesces to the use of his
No.VI. Gigolo entered into an agreement
surname by Gs daughter by another man,
what is/are the consequence/s? Explain. with Majorette for her to carry in her womb

(5%) his baby via in vitro fertilization. Gigolo

undertook to underwrite Majorettes pre-


SUGGESTED ANSWER: natal expenses as well as those attendant

to her delivery. Gigolo would thereafter pay


If B acquiesces and does not file the
Majorette P2 million and, in return, she
action to impugn the legitimacy of the
would give custody of the baby to him.
child within the prescriptive period for

doing so in Article 170 of the Family After Majorette gives birth and delivers the

Code, G's daughter by another man shall baby to Gigolo following her receipt of P2

be conclusively presumed as the

legitimate daughter of B by G.

Never Let The Odds Keep You From Pursuing What You Know In Your Heart You Were Meant To Do.-
Leroy Satchel Paige
Page 34 of 180
Civil Law Q&As (2007-2013) hectorchristopher@yahoo.com dbaratbateladot@gmail.com

(B) Can Gigolo demand from Majorette the


return of the P2 million if he returns the
million, she engages your services as her
baby? Explain. (2.5%)
lawyer to regain custody of the baby.

SUGGESTED ANSWER:
(A) What legal action can you file on behalf
of Majorette? Explain. (2.5%)

SUGGESTED ANSWER:

As her lawyer, I can file a petition for

habeas corpus on behalf Majorette to

recover custody of her child. Since she is

the mother of the child that was born

out of wedlock, she has exclusive

parental authority and custody over the

child. Gigolo, therefore, has no right to

have custody of the child and his refusal

to give up custody will constitute illegal

detention for which habeas corpus is the

proper remedy.

ALTERNATIVE ANSWER:

The action to regain custody will not

prosper. In the first place Majorette

cannot regain custody of the baby. As

surrogate mother she merely carries the

child in her womb for its development.

The child is the child of the natural

parents- Gigolo and his partner. The

agreement between Gigolo and Majorette

is a valid agreement.
If Gigolo voluntarily recognized the child

No, he cannot. Both he and Majorette are as his illegitimate child in accordance

guilty of violating the provision of the with Article 175 in relation to Article

Anti-Child Abuse Law (RA7610) on child 172 of the Family Code, the child is

trafficking. Being in pari delicto, the entitled to support and inheritance from

partners shall be left where they are and Gigolo.

Gigolo cannot demand the return of what


ALTERNATIVE ANSWER:
he paid.
Yes, because Gigolo is the natural and
ALTERNATIVE ANSWER: biological parent of the baby.

Yes. The agreement between Gigolo and

Majorette is a valid agreement.


Paternity & Filiation; Legitimacy;
(D) Is the child entitled to support and Presumption (2008)
inheritance from Gigolo? Explain. (2.5%) No. III. Roderick and Faye were high school
sweethearts. When Roderick was 18 and
SUGGESTED ANSWER:
Faye, 16 years old, they started to live
together as husband and wife without the

Never Let The Odds Keep You From Pursuing What You Know In Your Heart You Were Meant To Do.-
Leroy Satchel Paige
Page 35 of 180
Civil Law Q&As (2007-2013) hectorchristopher@yahoo.com dbaratbateladot@gmail.com

No. Laica cannot bring an action to

benefit of marriage. When Faye reached 18 impugn her own status. In Liyao Jr. v.

years of age, her parents forcibly took her Tanhoti-Liyao, G.R. No. 138961, 07

back and arranged for her marriage to Brad.

Although Faye lived with Brad after the

marriage, Roderick continued to regularly

visit Faye while Brad was away at work.

During their marriage, Faye gave birth to a

baby girl, Laica. When Faye was 25 years old,

Brad discovered her continued liason with

Roderick and in one of their heated

arguments, Faye shot Brad to death. She lost

no time in marrying her true love Roderick,

without a marriage license, claiming that they

have been continuosly cohabiting for more

than 5 years.

(B). What is the filiation status of Laica?


(2%)

SUGGESTED ANSWER:

Laica is legitimate because children

conceived or born during the marriage of

the parents are presumed to be

legitimate (Art. 164, FC).

(C).Can Laica bring an action to impugn her

own status on the ground that based on


DNA results, Roderick is her biological

father? (2%)

SUGGESTED ANSWER:
No. Laica cannot be legitimated by the

March 2002, the Supreme Court ruled marriage of her biological parents

that impugning the legitimacy of the because only children conceived and

child is a strictly personal right of born outside of wedlock of parents who

husband, except: (a) when the husband at the time of the conception of the

died before the expiration of the period former were not disqualified by any

fixed for bringing the action; (b) if he impediment to marry each other may be

should die after the filing of the legitimated (Art. 177, FC).

complaint, without having desisted

therefrom, or (c) if the child was born

after the death of the husband. Laica's Paternity & Filiation; Legitimation of a

case does not fall under any of the Child from a Previous Valid Marriage

exceptions. (2008)

No. IV. Gianna was born to Andy and


(D). Can Laica be legitimated by the
Aimee, who at the time Gianna's birth were
marriage of her biological parents? (1%)
not married to each other. While Andy was

single at the time, Aimee was still in the


SUGGESTED ANSWER:

Never Let The Odds Keep You From Pursuing What You Know In Your Heart You Were Meant To Do.-
Leroy Satchel Paige
Page 36 of 180
Civil Law Q&As (2007-2013) hectorchristopher@yahoo.com dbaratbateladot@gmail.com

No. I. TRUE or FALSE. Answer TRUE if the


process of securing a judicial declaration of
statement is true, or FALSE if the
nullity on her marriage to her ex-husband.
Gianna's birth certificate, which was signed

by both Andy and Aimee, registered the


status of Gianna as "legitimate", her

surname carrying that of Andy's and that


her parents were married to each other.

(C). Assuming that Aimee is successful in

declaring her former marriage void, and


Andy and Aimee subsequently married each

other, would Gianna be legitimated? (1%)

SUGGESTED ANSWER:

Gianna cannot be legitimated by the

subsequent marriage of Andy and Aimee.

Art. 177 of the FC provides that "only

children conceived and born outside of

wedlock of parents who, at the time of

the conception of the former, were not

disqualified by any impediment to marry

each other may be legitimated." In the

present case, a legal impediment was

existing at the time of the conception of

Gianna. Her mother, Aimee, was still

alive in the process of securing judicial

declaration of nullity on her marriage to

her ex-husband.

Paternity & Filiation; Legitimation of a

Dead Child (2009)


statement is false. Explain your answer in
Paternity & Filiation; Support:
not more than two (2) sentences.
Ascendants & Descendants; Collateral

Blood Relatives (2008)


(E). A dead child can be legitimated. (1%)

No.V. Despite several relationships with


SUGGESTED ANSWER:
different women, Andrew remained

TRUE To be legitimated, the law does not unmarried. His first relationship with

require a child to be alive at the same Brenda produced a daughter, Amy, now 30

time of the marriage of his / her parents years old. His second, with Carla, produced

two sons: Jon and Ryan. His third, with


( Article 177, FC ). Furthermore, Art.
Donna, bore him no children although
181 of the Family Code which states
Elena has a daughter Jane, from a previous
that [Th]e llegitimation of children who
relationship. His last, with Fe, produced no
died before the celebration of marriage
biological children but they informally
will benefit their descendants, does not
adopted without court proceedings, Sandy's
preclude instances where such now 13 years old, whom they consider as
legitimation will benefit no one but the

child's ascendants ,or other relatives .

Never Let The Odds Keep You From Pursuing What You Know In Your Heart You Were Meant To Do.-
Leroy Satchel Paige
Page 37 of 180
Civil Law Q&As (2007-2013) hectorchristopher@yahoo.com dbaratbateladot@gmail.com

their own. Sandy was orphaned as a baby Paternity & Filiation; Use of Surname;

and was entrusted to them by the midwife Illegitimate Child (2009)


who attended to Sandy's birth. All the
No.XIV. Rodolfo, married to Sharon, had an
children, including Amy, now live with
illicit affair with his secretary, Nanette, a
andrew in his house.
19-year old girl, and begot a baby girl,
(B). In his old age, can Andrew be legally Rona. Nanette sued Rodolfo for damages:
entitled to claim support from Amy, Jon, actual, for hospital and other medical
Ryan, Vina, Wilma, and Sandy assuming expenses in delivering the child by
that all of them have the means to support caesarean section; moral, claiming that
him? (1%) Rodolfo promised to marry her,

representing that he was single when, in


SUGGESTED ANSWER: fact, he was not; and exemplary, to teach a

lesson to like-minded Lotharios.


Andrew, in his old age, cannot be legally

entitled to claim support because Art. (B). Suppose Rodolfo later on acknowledges
195, par 2 of the FC limits the giving of Rona and gives her regular support, can he

support to "legitimate ascendants and compel her to use his surname? Why or

descendants." why not? (2%)

(C). Can Amy, Jon, Ryan, Vina, Wilma, and SUGGESTED ANSWER:

Sandy legally claim support from each No. he has no right to compel Rona to

other? (2%) use his surname. The law does not give

him the right simply because he gave her


SUGGESTED ANSWER:
support (RA 9255).

Amy, Jon, Ryan, Vina, Wilma and Sandy


Under the Family Code, an illegitimate
cannot legally claim support from each
child was required to use only the
other because Art. 195, par 5 limits the
surname of the mother. Under RA 9255,
giving of support to "legitimate brothers
otherwise known as the Revilla law,
and sisters, whether full or half blood."
however, the illegitimate child is given

the option to use the surname of the

illegitimate father when the latter has


recognized the former in accordance illegitimate child, Rodolfo cannot

with law. Since the choice belongs to the

Never Let The Odds Keep You From Pursuing What You Know In Your Heart You Were Meant To Do.-
Leroy Satchel Paige
Page 38 of 180
Civil Law Q&As (2007-2013) hectorchristopher@yahoo.com dbaratbateladot@gmail.com

No, Alberto and Baldomero are not


compel Rona, if already of age, to use
entitled to share in Edilbertos estate.
the surname against her will. If Rona is

still a minor, to use the surname of

Rodolfo will require the consent of

Rona's mother who has sole parental

authority over her.

Paternity & Filiation; Who May Impugn

Legitimacy (2009)

No.V. Four children, namely: Alberto,

Baldomero, Caridad, and Dioscoro, were


born to the spouses Conrado and Clarita de

la Costa. The childrens birth certificates


were duly signed by Conrado, showing them

to be the couples legitimate children.

Later, one Edilberto de la Cruz executed a

notarial document acknowledging Alberto


and Baldomero as his illegitimate children

>with Clarita. Edilberto died leaving


substantial properties. In the settlement of

his estate, Alberto and Baldomero


intervened claiming shares as the

deceaseds illegitimate children. The


legitimate family of Edilberto opposed the

claim.

Are Alberto and Baldomero entitled to share


in the estate of Edilberto? Explain. (4%)

SUGGESTED ANSWER:
illegitimate children. Before they can be

They are not related at all to Edilberto. conferred the status of Edilbertos

They were born during the marriage of illegitimate children, Condrado must

Conrado and Clarita, hence, are first impugn their legitimacy. Since

considered legitimate children of the Condrado has not initiated any action to

said spouses. This status is conferred on impugn their legitimacy, they continue

them at birth by law. to be the legitimate of Condrado. They

cannot be the illegitimate children of

Under Philippine law, a person cannot Edilberto at the same time. Not being

have more than one natural filiation. the illegitimate children of Edilberto,

The legitimate filiation of a person can they have no right to inherit from him.

be changed only if the legitimate father

will successfully impugn such status.


Property Relations; Adulterous

Relationship (2009)
In the problem, therefore, the filiation of

Alberto and Baldomero as legitimate


No. XI. TRUE or FALSE. Answer TRUE if
children of Condrado cannot be changed
the statement is true, or FALSE if the
by their recognition by Edilberto as his

Never Let The Odds Keep You From Pursuing What You Know In Your Heart You Were Meant To Do.-
Leroy Satchel Paige
Page 39 of 180
Civil Law Q&As (2007-2013) hectorchristopher@yahoo.com dbaratbateladot@gmail.com

building conjugal or paraphernal? Reasons.


(5%)
statement is false. Explain your answer in
not more than two (2) sentences.

(B). If there is no marriage settlement, the


salary of a "spouse" in an adulterous

marriage belongs to the conjugal


partnership of gains. (1%)

SUGGESTED ANSWER:

False. In adulterous relationship, the

salary of a married partner belongs to

the absolute community, or conjugal

partnership, of such married partner

with his or her lawful spouse. Under

Articles 148 of the Family Code, the

property relations between married

partner and his/her paramour is

governed by ordinary co-ownership

where the partners become co-owners

only when they contributed to the

acquisition of the property. The

paramour is deemed to have not

contributed in the earning of the salary

of the married partner.

Property Relations; Accession (2012)

No.III.(a) Maria, wife of Pedro, withdrew P 5

Million from their conjugal funds. With this


money, she constructed a building on a lot

which she inherited from her father. Is the


Note: The rule on reverse accession is
SUGGESTED ANSWER: applicable only to the regime of conjugal

partnership of gains in both the Family


It depends. If the value of the building is
Code and the New Civil Code. The foregoing
more than the value of the land, the
answer assumes that CPG is the regime of
building is conjugal and the land
the property relations of the spouses.
becomes conjugal property under Art.

120 of the Family Code. This is a case of

reverse accession, where the building is


Property Relations; Ante-Nuptial Debt
considered as the principal and the land,
(2007)
the accessory. If, on the other hand, the

value of the land is more than the value No. VII. Write "TRUE" if the statement is
of the building, then the ordinary rule of true or "FALSE" if the statement is false. If

accession applies where the land is the the statement is FALSE, state the reason.

principal and the building, the (2% each).

accessory. In such case, the land


(3). An individual, While single, purchases a
remains paraphernal property and the
house and lot in 1990 and borrows money
building becomes paraphernal propery.

Never Let The Odds Keep You From Pursuing What You Know In Your Heart You Were Meant To Do.-
Leroy Satchel Paige
Page 40 of 180
Civil Law Q&As (2007-2013) hectorchristopher@yahoo.com dbaratbateladot@gmail.com

in 1992 to repair it. In 1995, such Property Relations; Unions Without


individual gets married while the debt is Marriage (2012)
still being paid. After the marriage, the debt
No.V. a) Spouses Primo and Monina Lim,
is still the responsibility of such individual.
childless, were entrusted with the custody

SUGGESTED ANSWER: of two (2) minor children, the parents of

whom were unknown. Eager of having


FALSE. The absolute Community of
children of their own, the spouses made it
property is liable for the ante-nuptial appear that they were the childrens
debts of either spouse in so far as the parents by naming them Michelle P. Lim

same redounded to the benefit of the and Michael Jude Lim. Subsequently,

family (Art. 94 par.7, FC). Monina married Angel Olario after Primos
death.
ALTERNATIVE ANSWER:
She decided to adopt the children by
FALSE. The debt is already the availing the amnesty given under R.A. 8552

responsibility of the community to those individuals who simulated the

property, because the property already birth of a child. She filed separate petitions

constitutes absolute community for the adoption of Michelle, then 25 years

old and Michael, 18. Both Michelle and


property under Art. 91 of FC which took
Michael gave consent to the adoption.
effect in 1988 while the house and lot

here involved was purchased in 1990.


The trial court dismissed the petition and
There is no indication that the spouse ruled that Monina should have filed the
who bought the property had legitimate petition jointly with her new husband.

descendants by a former marriage, which Monina, in a Motion for Reconsideration

would exclude the house and lot from argues that mere consent of her husband

the community property, Art. 92 par 3, would suffice and that joint adoption is not

needed, for the adoptees are already


FC). If the spouses established a conjugal
emancipated.
partnership, the property belongs to the

individual spouse if full ownership was


Is the trial court correct in dismissing the
vested before marriage (Art. 118, FC). petitions for adoption? Explain. (5%)
SUGGESTED ANSWER:

Never Let The Odds Keep You From Pursuing What You Know In Your Heart You Were Meant To Do.-
Leroy Satchel Paige
Page 41 of 180
Civil Law Q&As (2007-2013) hectorchristopher@yahoo.com dbaratbateladot@gmail.com

rights and interests in the Agro-Macro


properties to Borromeo.
Yes, the trial court was correct. At the

time the petitions for adoptions were

filed, petitioner had already remarried.

Under the law, husband and wife shall

adopt jointly, except in the cases

enumerated in the law. The adoption

cases of Michelle and James do not fall

in any of the exceptions provided in the

law where a spouse is permitted to adopt

alone. Hence, Monina should adopt

jointly with her husband Angel (Adoption

of Michelle P. Lim, G.R. Nos. 168992-93,

May 21, 2009).

Property Relations; Unions Without

Marriage (2012)

No.V. b) Jambrich, an Austrian, fell in-love


and lived together with Descallar and

bought their houses and lots at Agro-Macro


Subdivision. In the Contracts to Sell,

Jambrich and Descallar were referred to as


the buyers. When the Deed of Absolute Sale

was presented for registration before the


Register of Deeds, it was refused because

Jambrich was an alien and could not


acquire alienable lands of the public

domain. After Jambrich and Descallar


separated, Jambrich purchased an engine

and some accessories for his boat from


Borromeo. To pay for his debt, he sold his
apply to their property relations and the

Borromeo discovered that titles to the three properties in question are owned by

(3) lots have been transfereed in the name them in equal shares even though all the

of Descallar. Who is the rightful owner of funds used in acquiring the properties
the properties? Explain. (5%) came only from the salaries or wages, or

the income of Jambrich from his


SUGGESTED ANSWER:
business or profession. In such case,
It depends. On the assumption that the while Jambrich is disqualified to own
Family Code is the applicable law, the any part of the properties, his
ownership of the properties depends on subsequent transfer of all his interest
whether or not, Jambrich and Descallar therein to Borromeo, a Filipino, was
are capacitated to marry each other valid as it removed the disqualification.
during their cohabitation, and whether In such case, the properties are owned
or not both have contributed funds for by Borromeo and Descallar in equal
the acquisition of the properties. shares.

If both of them are capacitated to marry If, on the other hand, Jambrich and
each other, Art 147- co-ownership will Descallar were not capacitated to marry

Never Let The Odds Keep You From Pursuing What You Know In Your Heart You Were Meant To Do.-
Leroy Satchel Paige
Page 42 of 180
Civil Law Q&As (2007-2013) hectorchristopher@yahoo.com dbaratbateladot@gmail.com

which bore no offspring, was declared void

ab initio under Article 36 of the Family


each other Art. 148-co-ownership
Code. At the time of the dissolution of the
governs their property relations. Under

this regime, Jambrich and Descallar are

co-owners of the properties but only if

both of them contributed in their

acquisition. If all the funds used in

acquiring the properties in question

came from Jambrich, the entire property

is his even though he is disqualified from

owning it. His subsequent transfer to

Borromeo, however, is valid as it

removed the disqualification. In such

case, all the properties are owned by

Borromeo. If, on the other hand

Descallar contributed to their

acquisition, the properties are co-owned

by Descallar and Borromeo in proportion

to the respective contributions of the

Descallar and Jambrich.

Note: The facts of the problem are not

exactly the same as in the case of

Borromeo v. Descallar, G.R. NO. 159310,

Feb 24, 2009, hence, the difference in

the resulting answer.

Property Relations; Void Marriages

(2010)

No.VII. G and B were married on July 3,

1989. On March 4, 2001, the marriage,


SUGGESTED ANSWER:
marriage, the couple possessed the
following properties: Since the marriage was declared void ab

initio in 2001, no Absolute Community


a house and lot acquired by B on
or Conjugal Partnership was ever
August 3, 1988, one third (1/3) of
established between B and G. Their
the purchase price (representing
property relation is governed by a
downpayment) of which he paid; one
special co-ownership under Article 147
third (1/3) was paid by G on
February 14, 1990 out of a cash gift of the Family Code because they were

given to her by her parents on her capacitated to marry each other.

graduation on April 6, 1989; and


Under that Article 147, wages and
the balance was paid out of the
salaries of the former spouses earned
spouses joint income; and
during their cohabitation shall be owned
an apartment unit donated to B by
an uncle on June 19, 1987. by them in equal shares while properties

acquired thru their work for industry


(A) Who owns the foregoing properties?
shall be owned by them in proportion to
Explain. (5%)

Never Let The Odds Keep You From Pursuing What You Know In Your Heart You Were Meant To Do.-
Leroy Satchel Paige
Page 43 of 180
Civil Law Q&As (2007-2013) hectorchristopher@yahoo.com dbaratbateladot@gmail.com

during the cohabitation, hence, it is his

their respective contributions. Care and exclusive property.

maintenance of the family is recognized


(2) 1/3 of the house and lot is owned by
as a valuable contribution. In the

absence of proof as to the value of their

respective contributions, they shall

share equally.

If ownership of the house and lot was

acquired by B on August 3, 1988 at the

time he bought it on installment before

he got married, he shall remain owner of

the house and lot but he must reimburse

G for all the amounts she advanced to

pay the purchase price and for one-half

share in the last payment from their

joint income. In such case, the house

and lot were not acquired during their

cohabitation, hence, are not co-owned by

B and G.

But if the ownership of the house and lot

was acquired during the cohabitation,

the house and lot will be owned as

follows:

(1) 1/3 of the house and lot is owned by

B. He is an undivided co-owner to that

extent for his contributions in its

acquisition in the form of the down

payment he made before the celebration

of the marriage. The money he used to

pay the down payment was not earned


After summing up their prospective
G. She is an undivided co-owner to the
shares, B and G are undivided co-owners
extent for her contribution in its
of the house and lot in equal shares.
acquisition when she paid 1/3 of the

purchase price using the gift from her As to the apartment, it is owned
parents. Although the gift was acquired exclusive by B because he acquired it
by G during her cohabitation with B, it is before their cohabitation. Even if he
her exclusive property. It did not consist acquired it during their cohabitation, it
of wage or salary or fruit of her work or will still be his exclusive property
industry. because it did not come from his wage or

salary, or from his work or industry. It


(3) 1/3 of the house is co-owned by B
was acquired gratuitously from his uncle.
and G because the payment came from

their co-owned funds, i.e., their joint (B) If G and B had married on July 3, 1987

income during their cohabitation which and their marriage was dissolved in 2007,
who owns the properties? Explain. (5%)
is shared by them equally in the absence

of any proof to the contrary.


SUGGESTED ANSWER:

Never Let The Odds Keep You From Pursuing What You Know In Your Heart You Were Meant To Do.-
Leroy Satchel Paige
Page 44 of 180
Civil Law Q&As (2007-2013) hectorchristopher@yahoo.com dbaratbateladot@gmail.com

No.X. In 1997, B and G started living


The answer is the same as in letter A.
together without the benefit of marriage.
Since the parties to the marriage which
The relationship produced one offspring,
was later declared void ab initio were Venus. The couple acquired a residential lot
capacitated to marry each other, the

applicable law under the New Civil Code

was Article 144.This Article is

substantially the same as Article 147 of

the Family Code.

Hence, the determination of ownership

will remain the same as in question A.

And even assuming that the two

provisions are not the same, Article 147

of the Family Code is still the law that

will govern the property relations of B

and G because under Article 256, the

Family Code has retroactive effect

insofar as it does not prejudice or impair

vested or acquired rights under the new

Civil Code or other laws. Applying Article

147 retroactively to the case of G and B

will not impair any vested right. Until

the declaration of nullity of the marriage

under the Family Code, B and G have not

as yet acquired any vested right over the

properties acquired during their

cohabitation.

Property Relations; Void Marriages

(2010)
and G. Their properties are governed by

in Paraaque. After four (4) years or in the special co-ownership provision of

2001, G having completed her 4-year Article 147 of the Family Code because

college degree as a fulltime student, she both B and G were capacitated to marry
and B contracted marriage without a each other. The said Article provides
license. that when a man and a woman who are

capacitated to marry each other, live


The marriage of B and G was, two years
exclusively with each other as husband
later, declared null and void due to the
and wife without the benefit of marriage,
absence of a marriage license.
or under a void marriage: (1) their wages
(A). If you were the judge who declared the and salaries shall be owned by them in
nullity of the marriage, to whom would you
equal shares; and (2) property acquired
award the lot? Explain briefly. (3%)
by both of them through their work or

industry shall be governed by the rules


SUGGESTED ANSWER:
on co-ownership. In co-ownership, the
Since the marriage was null and void, no
parties are co-owners if they contributed
Absolute Community or Conjugal
something of value in the acquisition of
Partnership was established between B

Never Let The Odds Keep You From Pursuing What You Know In Your Heart You Were Meant To Do.-
Leroy Satchel Paige
Page 45 of 180
Civil Law Q&As (2007-2013) hectorchristopher@yahoo.com dbaratbateladot@gmail.com

the property. Their share is in Succession


proportion to their respective

contributions. In an ordinary co- Disposition; Mortis Causa vs. Intervivos;

ownership the care and maintenance of Corpse (2009)

the family is not recognized as a valuable


No. XI. TRUE or FALSE. Answer TRUE if
contribution for the acquisition of a
the statement is true, or FALSE if the
property. In the Article 147 special co-
statement is false. Explain your answer in
ownership however, care and
not more than two (2) sentences.
maintenance is recognized as a valuable

contribution which will entitle the (E). A person can dispose of his corpse

contributor to half of the property through an act intervivos. (1%)

acquired.
SUGGESTED ANSWER:

Having been acquired during their False. A persons cannot dispose of his

cohabitation, the residential lot is corpse through an act inter vivos, i.e.,

presumed acquired through their joint an act to take effect during his lifetime.

work and industr under Article 147, Before his death there is no corpse to

hence, B and G are co-owners of the said dispose. But he is allowed to do so

property in equal shares. through an act mortis causa, i.e., an act

to take effect upon his death.


Article 147 also provides that when a

party to the void marriage was in bad

faith, he forfeits his share in the co-


Heirs; Fideicommissary Substitution
ownership in favor of the common
(2008)
children or descendants, the default of

children or descendants, the forfeited No. XIII. Raymond, single, named his sister
share shall belong to the innocent party. Ruffa in his will as a devisee of a parcel of
In the foregoing problem, there is no land which he owned. The will imposed

showing that one party was in bad faith. upon Ruffa the obligation of preseving the

Hence, both shall be presumed in good land and transferring it, upon her death, to
her illegitimate daughter Scarlet who was
faith and no forfeiture shall take place.
then only one year old. Raymond later died,
leaving behind his widowed mother, Ruffa
and Scarlet.

Never Let The Odds Keep You From Pursuing What You Know In Your Heart You Were Meant To Do.-
Leroy Satchel Paige
Page 46 of 180
Civil Law Q&As (2007-2013) hectorchristopher@yahoo.com dbaratbateladot@gmail.com

illegitimate child, cannot inherit the

(A). Is the condition imposed upon Ruffa, to property by intestate succession from

preserve the property and to transmit it Raymond who is a legitimate relative of

upon her death to Scarlet, valid? (1%)

SUGGESTED ANSWER:

Yes, the condition imposed upon Ruffa

to preserve the property and to transmit

it upon her death to Scarlet is valid

because it is tantamount to

fideicommissary substitution under Art.

863 of the Civil Code.

(B). If Scarlet predeceases Ruffa, who


inherits the property? (2%)

SUGGESTED ANSWER:

Ruffa will inherit the property as

Scarlet's heir. Scarlet acquires a right to

the succession from the time of

Raymond's death, even though she

should predecease Ruffa (Art. 866, Civil

Code).

(C). If Ruffa predeceases Raymond, can


Scarlet inherit the property directly from
Raymond? (2%)

SUGGESTED ANSWER:

If Ruffa predeceases Raymond,

Raymond's widowed mother will be

entitled to the inheritance. Scarlet, an


each of their 3 children as their
presumptive legitimes.
Ruffa (Art. 992, Civil Code). Moreover,

Scarlet is not a compulsory heir of


Peter later re-married and had two (2)
Raymond, hence she can inherit only by children by his second wife Marie. Peter and
testamentary succession. Since Marie, having successfully engaged in

Raymond executed a will in the case at business, acquired real properties. Peter

bar, Scarlet may inherit from Raymond. later died intestate.

(A). Who are Peters legal heirs and how will


his estate be divided among them? (5%)
Heirs; Intestate Succession; Legitime;

Computation (2010) SUGGESTED ANSWER:

No.XI. The spouses Peter and Paula had The legal heirs of Peter are his children

three (3) children. Paula later obtained a by the first and second marriages and

judgment of nullity of marriage. Their his surviving second wife.

absolute community of property having


Their shares in the estate of Peter will
been dissolved, they delivered P1 million to
depend, however, on the cause of the

Never Let The Odds Keep You From Pursuing What You Know In Your Heart You Were Meant To Do.-
Leroy Satchel Paige
Page 47 of 180
Civil Law Q&As (2007-2013) hectorchristopher@yahoo.com dbaratbateladot@gmail.com

Surviving second 1/6 of the estate


nullity of the first marriage. If the spouse

nullity of the first marriage was

psychological incapacity of one or both

spouses, the three children of that void

marriage are legitimate and all of the

legal heirs shall share the estate of Peter

in equal shares. If the judgment of

nullity was for other causes, the three

children are illegitimate and the estate

shall be distributed such that an

illegitimate child of the first marriage

shall receive half of the share of a

legitimate child of the second marriage,

and the second wife will inherit a share

equal to that of a legitimate child. In no

case may the two legitimate children of

the second marriage receive a share less

than one-half of the estate which is their

legitime. When the estate is not

sufficient to pay all the legitimes of the

compulsory heirs, the legitime of the

spouse is preferred and the illegitimate

children suffer the reduction.

Computation:

(A) If the ground of nullity is

psychological incapacity:

3 children by first 1/6 of the estate


marriage for each

2 children by second 1/6 of the estate


marriage for each
the estate is left after paying the

(B) If the ground of nullity is not legitime of the surviving spouse which is

psychological capacity: preferred.

2 legitimate of the estate for


Hence, the remaining of the estate
children each of second
marriage shall be divided among the 3 illegitimate

children.
Surviving of the estate
second spouse
(B). What is the effect of the receipt by
3 illegitimate 1/12 of estate for
children each of first marriage Peters 3 children by his first marriage of

their presumptive legitimes on their right to


inherit following Peters death? (5%)

Note: The legitime of an illegitimate


SUGGESTED ANSWER:
child is supposed to be the legitime of

a legitimate child or 1/8 of the estate. In the distribution of Peters estate, of

But the estate will not be sufficient to the presumptive received by the 3

pay the said legitime of the 3 children of the first marriage shall be

illegitimate children, because only of collated to Peters estate and shall be

Never Let The Odds Keep You From Pursuing What You Know In Your Heart You Were Meant To Do.-
Leroy Satchel Paige
Page 48 of 180
Civil Law Q&As (2007-2013) hectorchristopher@yahoo.com dbaratbateladot@gmail.com

heir of the decedent he seeks to inherit

imputed as an advance of their from.

respective inheritance from Peter. Only


While Arnel is a legal heir of Franco, he
half of the presumptive legitime is
is not a legal heir of Ricky because under
collated to the estate of Peter because

the other half shall be collated to the

estate of his first wife.

Heirs; Representation; Iron-Curtain Rule

(2012)

No.VIII.a) Ricky and Arlene are married.

They begot Franco during their marriage.


Franco had an illicit relationship with

Audrey and out of which, they begot Arnel.


Frnaco predeceased Ricky, Arlene and

Arnel. Before Ricky died, he executed a will


which when submitted to probate was

opposed by Arnel on the ground that he


should be given the share of his father,

Franco. Is the opposition of Arnel correct?


Why? (5%)

SUGGESTED ANSWER:

No, his opposition is not correct. Arnel

cannot inherit from Ricky in the

representation of his father Franco. In

representation, the representative must

not only be a legal heir of the person he

is representing, he must also be a legal


(B).In reservatroncal, all reservatarios (reser

vees) inherit as a class and in equal shares


Art 992 of the NCC, an illegitimate child
regardless of their proximity in degree to
has no right to inherit ab intestato from
the prepositus. (1%)
the legitimate children and relatives of

his father or mother. Arnel is SUGGESTED ANSWER:


disqualified to inherit from Ricky FALSE. Not all the relatives within the

because Arnel is an illegitimate child of third degree will inherit as reservatario ,

Franco and Ricky is a legitimate relative and not all those who are entitled to

of Franco. inherit will inherit in the equal shares .

The applicable laws of intestate

succession will determine who among


Heirs; Reserva Troncal (2009) the relatives will inherit as reservatarios

and what shares they will tak, i.e., the


No. I. TRUE or FALSE. Answer TRUE if the
direct line excludes the collateral, the
statement is true, or FALSE if the
descending direct line excludes the
statement is false. Explain your answer in

not more than two (2) sentences. ascending ,the nearer excludes the more

remote, the nephews and nieces exclude

Never Let The Odds Keep You From Pursuing What You Know In Your Heart You Were Meant To Do.-
Leroy Satchel Paige
Page 49 of 180
Civil Law Q&As (2007-2013) hectorchristopher@yahoo.com dbaratbateladot@gmail.com

blood, a half blood sibling will receive

the uncles and the aunts, and half blood half the share of a full-blood sibling.

relatives inherit half the share of full-

blooded relatives.

Intestate Succession (2008)

No. VII. Ramon Mayaman died intestate,

leaving a net estate of P10,000,000.00.


Determine how much each heir will receive

from the estate:

(A). If Ramon is survived by his wife, three


full-blood brothers, two half-brothers, and

one nephew (the son of a deceased full-


blood brother)? Explain. (3%)

SUGGESTED ANSWER:

Having died intestate, the estate of

Ramon shall be inherited by his wife and

his full and half blood siblings or their

respective representatives. In intestacy,

if the wife concurs with no one but the

siblings of the husband, all of them are

the intestate heirs of the deceased

husband. The wife will receive half of the

intestate estate, while the siblings or

their respective representatives, will

inherit the other half to be divided

among them equally. If some siblings are

of the full-blood and the other of the half


The wife will receive one half (1/2) of the

(1). The wife of Ramon will, therefore, estate or P5,000,000.00. The other half

receive one half () of the estate or the shall be inherited by (1) the full-blood

amount of P5,000,000.00. brother, represented by his three

(2). The three (3) full-blood brothers, will, children, and (2) the half-sister. They will

therefore, receive P1,000,000.00 each. divide the other half between them such

(3). The nephew will receive that the share of the half-sister is just

P1,000,000.00 by right of half the share of the full-blood brother.

representation. The share of the full-blood brother shall

(4). The two (2) half-brothers will receive in turn be inherited by the three

P500,000.00 each. nephews in equal shares by right of

presentation.
(B). If Ramon is survived by his wife, a half-
sister, and three nephews (sons of a Therefore, the three (3) nephews will
deceased full-blood brother)? Explain. (3%)
receive P1,111,111.10 each the half-

sister will receive the sum of


SUGGESTED ANSWER:
P1,666,666.60.

Never Let The Odds Keep You From Pursuing What You Know In Your Heart You Were Meant To Do.-
Leroy Satchel Paige
Page 50 of 180
Civil Law Q&As (2007-2013) hectorchristopher@yahoo.com dbaratbateladot@gmail.com

Intestate Succession (2008) (D). How should the house and lot, and the
cash be distributed? (1%)
No.X. Arthur executed a will which contained

only: (i) a provision disinheriting his daughter SUGGESTED ANSWER:


Bernica for running off with a married man,

and (ii) a provision disposing of his share in Since the probate of the will cannot be

the family house and lot in favor of his other allowed, the rules on intestate

children Connie and Dora. He did not make succession apply. Under Art. 996 of the

any provisions in favor of his wife Erica, Civil Code, if a widow or widower and
because as the will stated, she would anyway legitimate children or descendants are
get of the house and lot as her conjugal
left, the surviving spouse has the same
share. The will was very brief and
share as of the children. Thus, ownership
straightforward and both the above provisions
over the house and lot will be created
were contained in page 1, which Arthur and
among wife Erica and her children
his instrumental witness, signed at the
Bernice, Connie and Dora. Similarly, the
bottom. Page 2 contained the attestation
amount of P 1 million will be equally
clause and the signatures, at the bottom

thereof, of the 3 instrumental witnesses divided among them.

which included Lambert, the driver of Arthur;

Yoly, the family cook, and Attorney Zorba, the

lawyer who prepared the will. There was a 3rd Intestate Succession; Rights of
page, but this only contained the notarial Representation: Illegitimate, Adopted
acknowledgement. The attestation clause
Child; Iron Curtain Rule (2007)
stated the will was signed on the same

occasion by Arthur and his instrumental No. X. For purpose of this question, assume

witnesses who all signed in the presence of all formalities and procedural requirements

each other, and the notary public who have been complied with.

notarized the will. There are no marginal


In 1970, Ramon and Dessa got married.
signatures or pagination appearing on any of
Prior to their marriage, Ramon had a child,
the 3 pages. Upon his death, it was discovered
Anna. In 1971 and 1972, Ramon and Dessa
that apart from the house and lot, he had a P
legally adopted Cherry and Michelle
1 million account deposited with ABC bank.
respectively. In 1973, Dessa died while

giving birth to Larry Anna had a child, Lia.


Anna never married. Cherry, on the other
hand, legally adopted Shelly. Larry had

Never Let The Odds Keep You From Pursuing What You Know In Your Heart You Were Meant To Do.-
Leroy Satchel Paige
Page 51 of 180
Civil Law Q&As (2007-2013) hectorchristopher@yahoo.com dbaratbateladot@gmail.com

adopted. The law on representation

twins, Hans and Gretel, with his girlfriend, requires the representative to be a legal

Fiona. In 2005, Anna, Larry and Cherry heir of the person he is representing and

died in a car accident. In 2007, Ramon

died. Who may inherit from Ramon and


who may not? Give your reason briefly.

(10%)

SUGGESTED ANSWER:

The following may inherit from Ramon:

(1). Michelle, as an adopted child of

Ramon, will inherit as a legitimate child

of Ramon. As an adopted child, Michelle

has all the rights of a legitimate child

(Sec 18, Domestic Adoption Law).

(2). Lia will inherit in representation of

Anna. Although Lia is an illegitimate

child, she is not barred by Articles 992,

because her mother Anna is an

illegitimate herself. She will represent

Anna as regards Anna's legitime under

Art. 902, NCC and as regards Anna's

intestate share under Art. 990, NCC.

The following may not inherit from

Ramon:

(1). Shelly, being an adopted child, she

cannot represent Cherry. This is because

adoption creates a personal legal relation

only between the adopter and the


time, adoption was governed by the New

also of the person from whom the person Civil Code. Under the New Civil Code,

being represented was supposed to husband and wife were allowed to adopt

inherit. While Shelly is a legal heir of separately or not jointly with the other

Cherry, Shelly is not a legal heir of spouse. And since the problem does not

Ramon. Adoption created a purely specifically and categorically state, it is

personal legal relation only between possible to construe the use of the word

Cherry and Shelly. "respectively" in the problem as

indicative of the situation that Cherry


(2). Hans and Gretel are barred from was adopted by Ramon alone and
inheriting from Ramon under Art. 992, Michelle was adopted by Dessa alone. In
NCC. Being illegitimate children, they such case of separate adoption the
cannot inherit ab intestao from Ramon. alternative answer to the problem will be

as follows: Only Lia will inherit from


ALTERNATIVE ANSWER:
Ramon in representation of Ramon's

The problem expressly mentioned the illegitimate daughter Anna. Although Lia

dates of the adoption of Cherry and is an illegitimate child, she is not barred

Michelle as 1971 and 1972. During that from inheriting from Ramon because her

Never Let The Odds Keep You From Pursuing What You Know In Your Heart You Were Meant To Do.-
Leroy Satchel Paige
Page 52 of 180
Civil Law Q&As (2007-2013) hectorchristopher@yahoo.com dbaratbateladot@gmail.com

No.VIII.b) How can RJP distribute his estate


mother is herself illegitimate. Shelly by will, if his heirs are JCP, his wife; HBR
cannot inherit in representation of and RVC, his parents; and an illegitimate

Cherry because Shelly is just an adopted child, SGO?

child of Cherry. In representation, the

representative must not only be a legal

heir of the person he is representing but

also of the decedent from whom the

represented person is supposed to

inherit. In the case of Shelly, while she

is a legal heir of Cherry by virtue of

adoption, she is not a legal heir of

Ramon. Adoption creates a personal

legal relation only between the adopting

parent and the adopted child (Teotico v.

Del Val, 13 SCRA 406, 1965. Michelle

cannot inherit from Ramon, because she

was adopted not by Ramon but by Dessa.

In the eyes of the law, she is not related

to Ramon at all. Hence, she is not a legal

heir of Ramon. Hans and Gretel are not

entitled to inherit from Ramon, because

they are barred by Art. 992 NCC. Being

illegitimate children of Larry, they

cannot inherit from the legitimate

relatives of their father Larry. Ramon is

a legitimate relative of Larry who is the

legitimate father.

Legitimes; Compulsory Heirs (2012)


Middle East. He had saved P100.000 in his
saving account in Manila which intended to
SUGGESTED ANSWER:
use to start a business in his home
A testator may dispose of by will the free country. On his flight home, Ernesto had a
portion of his estate. Since the legitime fatal heart attack. He left behind his

of JCP is 1/8 of the estate, SGO is of widowed mother, his common-law wife and

the estate and that of HBR and RVC is their twins sons. He left no will, no debts,

of the hereditary estate under Art 889 of no other relatives and no other properties

except the money in his saving account.


the NCC, the remaining 1/8 of the estate
Who are the heirs entitled to inherint from
is the free portion which the testator
him and how much should each receive?
may dispose of by will.
(3%)

SUGGESTED ANSWER:

Legitime; Compulsory Heirs (2008)


The mother and twin sons are entitled to

No. XII. Ernesto, an overseas Filipino inherit from Ernesto. Art. 991 of the
worker, was coming home to the Philippines Civil Code, provides that if legitimate
after working for so many years in the

Never Let The Odds Keep You From Pursuing What You Know In Your Heart You Were Meant To Do.-
Leroy Satchel Paige
Page 53 of 180
Civil Law Q&As (2007-2013) hectorchristopher@yahoo.com dbaratbateladot@gmail.com

lawyer who prepared the will. There was a 3rd

page, but this only contained the notarial


ascendants are left, the twin sons shall
acknowledgement.
divide the inheritance with them taking

one-half of the estate. Thus, the widowed

mother gets P50,000.00 while the twin

sons shall receive P25,000.00 each. The

common-law wife cannot inherit from

him because when the law speaks "widow

or widower" as a compulsory heir, the

law refers to a legitimate spouse (Art.

887, par 3, Civil Code).

Preterition; Disinheritance (2008)

No.X. Arthur executed a will which contained

only: (i) a provision disinheriting his daughter

Bernica for running off with a married man,

and (ii) a provision disposing of his share in

the family house and lot in favor of his other

children Connie and Dora. He did not make

any provisions in favor of his wife Erica,

because as the will stated, she would anyway

get of the house and lot as her conjugal

share. The will was very brief and

straightforward and both the above provisions

were contained in page 1, which Arthur and

his instrumental witness, signed at the

bottom. Page 2 contained the attestation

clause and the signatures, at the bottom

thereof, of the 3 instrumental witnesses

which included Lambert, the driver of Arthur;

Yoly, the family cook, and Attorney Zorba, the


(B). What other defects of the will, if any,
The attestation clause stated the will was
can cause denial of probate? (2%)
signed on the same occasion by Arthur and
his instrumental witnesses who all signed SUGGESTED ANSWER:
in the presence of each other, and the
notary public who notarized the will. There The other defects of the will that can

are no marginal signatures or pagination cause its denial are as follows: (a) Atty.

appearing on any of the 3 pages. Upon his Zorba, the one who prepared the will was
death, it was discovered that apart from the one of the three witnesses, violating the
house and lot, he had a P 1 million account three-witnesses rule; (b) no marginal
deposited with ABC bank.
signature at the last page; (c ) the

attestation did not state the number of


(A). Was Erica preterited? (1%)
pages upon which the will is written;
SUGGESTED ANSWER: and, (d) no pagination appearing

correlatively in letters on the upper part


Erica cannot be preterited. Art. 854 of
of the three pages (Azuela v. C.A., G.R.
the Civil Code provides that only

compulsory heirs in the direct line can

be preterited.

Never Let The Odds Keep You From Pursuing What You Know In Your Heart You Were Meant To Do.-
Leroy Satchel Paige
Page 54 of 180
Civil Law Q&As (2007-2013) hectorchristopher@yahoo.com dbaratbateladot@gmail.com

No. 122880, 12 Apr 2006 and cited cases (B). Between Marian and the baby, who is

therein, Art 805 and 806, Civil Code). presumed to have died ahead? (1%)

(C). Was the disinheritance valid? (1%) SUGGESTED ANSWER:

SUGGESTED ANSWER: Marian is presumed to have died ahead

of the baby. Art. 43 applies to persons


Yes, the disinheritance was valid. Art.
who are called to succeed each other.
919, par 7, Civil Code provides that
The proof of death must be established
"when a child or descendant leads a
by positive or circumstantial evidence
dishonorable or disgraceful life, like
derived from facts. It can never be
running off with a married man, there is
established from mere inference. In the
sufficient cause for disinheritance."
present case, it is very clear that only

Marian and Pietro were hacked with

bolos. There was no showing that the


Succession; Proof of Death between baby was also hacked to death. The
persons called to succeed each other baby's death could have been due to lack
(2008) of nutrition.

No. II. At age 18, Marian found out that she


ALTERNATIVE ANSWER:
was pregnant. She insured her own life and
named her unborn child as her sole The baby is presumed to have died ahead

beneficiary. When she was already due to of Marian. Under Par. 5, rule 131, Sec. 5
give birth, she and her boyfriend Pietro, the (KK) of the Rules of Court, if one is
father of her unboarn child, were
under 15 or above 60 and the age of the
kidnapped in a resort in Bataan where they
other is in between 15 and 60, the latter
were vacationing. The military gave chase
is presumed to have survived. In the
and after one week, they were found in an
instant case, Marian was already 18
abandoned hut in Cavite. Marian and Pietro
when she found out that she was
were hacked with bolos. Marian and the
pregnant. She could be of the same age
baby delivered were both found dead, with

the baby's umbilical cord already cut. Pietro or maybe 19 years of age when she gave

survived. birth.
(C). Will Pietro, as surviving biological
father of the baby, be entitled to claim the

Never Let The Odds Keep You From Pursuing What You Know In Your Heart You Were Meant To Do.-
Leroy Satchel Paige
Page 55 of 180
Civil Law Q&As (2007-2013) hectorchristopher@yahoo.com dbaratbateladot@gmail.com

As to the Estate of Dr. Lopez:


proceeds of the life insurance on the life of
Marian? (2%)

SUGGESTED ANSWER:

Pietro, as the biological father of the

baby, shall be entitled to claim the

proceeds of life insurance of the Marian

because he is a compulsory heir of his

child.

Succession; Rule on Survivorship (2009)

No. II. Dr. Lopez, a 70-year old widower, and

his son Roberto both died in a fire that

gutted their home while they were sleeping


in their air-conditioned rooms. Robertos

wife, Marilyn, and their two children were


spared because they were in the province at

the time. Dr. Lopez left an estate worth


P20M and a life insurance policy in the

amount of P1M with his three children ---

one of whom is Roberto --- as beneficiaries.

Marilyn is now claiming for herself and her

children her husbands share in the estate


left by Dr. Lopez, and her husbands share

in the proceeds of Dr. Lopezs life insurance


policy. Rule on the validity of Marilyns

claims with reasons. (4%)

SUGGESTED ANSWER :
three children of Dr. Lopez . Marilyn

cannot represent her husband Roberto

Marilyn is not entitled to a share in the because the right is not given by the law

estate of Dr. Lopez. For purpose of to a surviving spouse.

succession, Dr. Lopez and his son

Roberto are presumed to have died at As to the proceeds of the insurance on

the same time, there being no evidence the life of Dr. Lopez:

to prove otherwise, and there shall be no


Since succession is not involved as
transmission of rights from one to the
regards the insurance contract, the
other (Article 43, NCC). Hence, Roberto,
provisions of the Rules of Court (Rule
inherited nothing from his father that
131, Sec. 3 , [jj] [5] ) on survivorship
Marilyn would in turn inherit from
shall apply. Under the Rules, Dr. Lopez,
Roberto .The children of Roberto,
who was 70 years old, is presumed to
however, will succeed their grandfather,
have died ahead of Roberto who is
Dr. Lopez ,in representation of their
presumably between the ages 15 and 60.
father Roberto and together Roberto will
Having survived the insured, Roberto's
receive 1/3 of the estate of Dr. Lopez
right as a beneficiary became vested
since their father Roberto was one of the

Never Let The Odds Keep You From Pursuing What You Know In Your Heart You Were Meant To Do.-
Leroy Satchel Paige
Page 56 of 180
Civil Law Q&As (2007-2013) hectorchristopher@yahoo.com dbaratbateladot@gmail.com

testator himself, Rosas claimed that the

upon the death of Dr. Lopez. When holographic will in its original tenor

Roberto died after Dr. Lopez, his right to

receive the insurance became part of his

hereditary estate, which in turn was

inherited in equal shares by his legal

heirs, namely, his spouse and children.

Therefore, Roberto's children and his

spouse are entitled to Roberto's one-

third share in the insurance proceeds.

Wills; Holographic Wills; Insertions &

Cancellations (2012)

No.VII.a) Natividads holographic will, which

had only one (1) substantial provision, as


first written, named Rosa as her sole heir.

However, when Gregorio presented it for


probate, it already contained an alteration,

naming Gregorio, instead of Rosa, as sole


heir, but without authentication by

Natividads signature. Rosa opposes the

probate alleging such lack of proper


authentication. She claims that the

unaltered form of the will should be given


effect. Whose claim should be granted?

Explain. (5%)

SUGGESTED ANSWER:

It depends. If the cancellation of Rosas

name in the will was done by the


The writing of Gregorios name as sole

should be given effect must be denied. heir was ineffective, even though written

The said cancellation has revoked the by the testator himself, because such is

entire will as nothing remains of the will an alteration that requires

after the name of Rosa was cancelled. authentication by the full signature of

Such cancellation is valid revocation of the testator to be valid and effective. Not

the will and does not require having an authenticated, the designation

authentication by the full signature of of Gregorio as an heir was ineffective,

the testator to be effective. (Kalaw v. Relova, G.R. No. L-40207, Sept

28, 1984).
However, if the cancellation of Rosas

name was not done by the testator

himself, such cancellation shall not be


Wills; Holographic Wills; Probate (2009)
effective and the will in its original tenor

shall remain valid. The effectively of the No.VI. On December 1, 2000, Dr. Juanito

holographic will cannot be left to the Fuentes executed a holographic will,

wherein he gave nothing to his recognized


mercy of unscrupulous third parties.
illegitimate son, Jay. Dr. Fuentes left for the

Never Let The Odds Keep You From Pursuing What You Know In Your Heart You Were Meant To Do.-
Leroy Satchel Paige
Page 57 of 180
Civil Law Q&As (2007-2013) hectorchristopher@yahoo.com dbaratbateladot@gmail.com

Civil Code (Art. 816, NCC). Since Dr.

United States, passed the New York medical Fuentes executed his will in accordance

licensure examinations, resided therein, with the Philippine law, the Philippine

and became a naturalized American citizen.

He died in New York in 2007. The laws of


New York do not recognize holographic wills

or compulsory heirs.

(A). Can the holographic will of Dr. Fuentes


be admitted to probate in the Philippines?
Why or why not? (3%)

SUGGESTED ANSWER:

Yes, the holographic will of Dr. Fuentes

may be admitted to probate in the

Philippines because there is no public

policy violated by such probate. The only

issue at probate is the due execution of

the will which includes the formal

validity of the will. As regards formal

validity, the only issue the court will

resolve at probate is whether or not the

will was executed in accordance with the

form prescribed by the law observed by

the testator in the execution of his will.

For purposes of probate in the

Philippines, an alien testator may

observe the law of the place where the

will was executed (Art 17, NCC), or the

formalities of the law of the place where

he resides, or according to the

formalities of the law of his own country,

or in accordance with the Philippine


No, Jay cannot insist because under New

court shall apply the New Civil Code in York law he is not a compulsory heir

determining the formal validity of the entitled to a legitime.

holographic will. The subsequent change


The national law of the testator
in the citizenship of Dr. Fuentes did not
determines who his heirs are, the order
affect the law governing the validity of
that they succeed, how much their
his will. Under the new Civil Code, which
successional rights are, and whether or
was the law used by Dr. Fuentes, the law
not a testamentary disposition in his will
enforced at the time of execution of the
is valid (Art 16, NCC). Since, Dr. Fuentes
will shall govern the formal validity of
was a US citizen, the laws of the New
the will (Art. 795, NCC).
York determines who his heirs are. And

(B). Assuming that the will is probated in since the New York law does not
the Philippines, can Jay validly insist that recognize the concept of compulsory
he be given his legitime? Why or why not?
heirs, Jay is not a compulsory heir of Dr.
(3%)
Fuentes entitled to a legitime.

SUGGESTED ANSWER:

Never Let The Odds Keep You From Pursuing What You Know In Your Heart You Were Meant To Do.-
Leroy Satchel Paige
Page 58 of 180
Civil Law Q&As (2007-2013) hectorchristopher@yahoo.com dbaratbateladot@gmail.com

(B). Are the testamentary dispositions


valid? (2%)
Wills; Joint Wills (2008)

No. XI. John and Paula, British citizens at

birth, acquired Philippine citizenship by


naturalization after their marriage. During

their marriage the couple acquired


substanial landholdings in London and in

Makati. Paula bore John three children,


Peter, Paul and Mary. In one of their trips to

London, the couple executed a joint will


appointing each other as their heirs and

providing that upon the death of the


survivor between them the entire estate

would go to Peter and Paul only but the two


could not dispose of nor divide the London

estate as long as they live. John and Paul


died tragically in the London Subway

terrorist attack in 2005. Peter and Paul filed


a petition for probate of their parent's will

before a Makati Regional Trial Court.

(A). Should the will be admitted to probate?


(2%)

SUGGESTED ANSWER:

No. The will cannot be admitted to

probate because a joint will is expressly

prohibited under Art. 818 of the Civil

Code. This provision applies John and

Paula became Filipino citizens after

their marriage.
No.VII.b) John Sagun and Maria Carla
Camua, British citizens at birth, acquired
SUGGESTED ANSWER:
Philippine citizenship by naturalization

No. The testamentary dispositions are after their marriage. During their marriage,

not valid because (a) omission of Mary, a the couple acquired substantial
landholdings in London and in Makati.
legitimate child, is tantamount to
Maria begot three (3) children, Jorge,
preterition which shall annul the
Luisito, and Joshur. In one of their trips to
institution of Peter and Paul as heirs
London, the couple executed a joint will
(Art. 854, Civil Code); and, (b) the
appointing each other as their heirs and
disposition that Peter and Paul could not
providing that upon the death of the
dispose of nor divide the London estate
survivor between them, the entire estate
for more than 20 years is void (Art. 870, would go to Jorge and Luisito only but the
Civil Code). two (2) could not dispose of nor divide the

London estate as long as they live. John


and Maria died tragically in the London

subway terrorist attack in 2005. Jorge and


Wills; Joint Wills; Probate (2012)
Luisito filed a petition for probate of their

parents will before a Makati Regional Trial

Never Let The Odds Keep You From Pursuing What You Know In Your Heart You Were Meant To Do.-
Leroy Satchel Paige
Page 59 of 180
Civil Law Q&As (2007-2013) hectorchristopher@yahoo.com dbaratbateladot@gmail.com

(3) Is the testamentary prohibition against


Court. Joshur vehemently objected because
the division of the London estate valid?
he was preterited.
Explain. (1%)

(1) Should the will be admitted to probate?


Explain. (2%)

SUGGESTED ANSWER:

No, the will should not be admitted to

probate. Since the couples are both

Filipino citizens, Art 818 and 819 of the

NCC shall apply. Said articles prohibits

the execution of joint wills and make

them void, even though authorized of

the country where they were executed.

(2) Are the testamentary dispositions valid?


Explain. (2%)

SUGGESTED ANSWER:

Since the joint will is void, all the

testamentary disposition written therein

are also void. However, if the will is

valid, the institutions of the heirs shall

be annulled because Joshur was

preterited. He was preterited because he

will receive nothing from the will, will

receive nothing in testacy, and the facts

do not show that he received anything as

an advance on his inheritance. He was

totally excluded from the inheritance of

his parents.
cannot be partitioned for as long as the
youngest of his four children desires to stay
SUGGESTED ANSWER:
there. As coheirs and co-owners, the other
Assuming the will of John and Maria was three may demand partition anytime. (1%)
valid, the testamentary prohibition on the

division of the London estate shall be valid SUGGESTED ANSWER:

but only for 20 years. Under Arts 1083 and


FALSE, The other three co heirs may
494 of the NCC, a testamentary disposition
not anytime demand the partition of the
of the testator cannot forbid the partition of
house and lot since it was expressly
all or part of the estate for a period longer
provided by the decedent in his will that
than twenty (20) years.
the same cannot be partitioned while his

youngest child desires to stay there.

Article 1083 of the New Civil Code allows


Wills; Prohibition to Partition of a Co-
a decedent to prohibit, by will, the
Owned Property (2010)
partition of a property and his estate for

No.I. True or False. a period not longer than 20 years no

(B) X, a widower, died leaving a will stating


that the house and lot where he lived

Never Let The Odds Keep You From Pursuing What You Know In Your Heart You Were Meant To Do.-
Leroy Satchel Paige
Page 60 of 180
Civil Law Q&As (2007-2013) hectorchristopher@yahoo.com dbaratbateladot@gmail.com

of Arts. 804, 805 and 806 of the Civil

matter what his reason maybe. Hence, Code.

the three co-heir cannot demand its

partition at anytime but only after 20

years from the death of their father.

Even if the deceased parent did not leave

a will, if the house and lot constituted

their family home, Article 159 of the

Family Code prohibits its partition for a

period of ten (10) years, or for as long as

there is a minor beneficiary living in the

family home.

Wills; Notarial Wills; Blind Testator;

Requisites (2008)

No. XIV. Stevie was born blind. He went to


school for the blind, and learned to read in

Baille Language. He Speaks English


fluently. Can he:

(A). Make a will? (1%)

SUGGESTED ANSWER:

Assuming that he is of legal age (Art.

797, Civil Code) and of sound mind at

the time of execution of the will (Art.

798, Civil Code), Stevie, a blind person,

can make a notarial will, subject to

compliance with the "two-reading rule"

(Art. 808, Civil Code) and the provisions


subscribing witnesses, and again, by the

(B). Act as a witness to a will? (1%) notary public before whom the will is

acknowledged (Art. 808, Civil Code).


SUGGESTED ANSWER:

Stevie cannot be a witness to a will. Art.

820 of the Civil Code provides that "any Wills; Testamentary Disposition; Period

person of sound mind and of the age of to Prohibit Partition (2008)

eighteen years or more, and not blind,


No. XI. John and Paula, British citizens at
deaf or dumb, and able to read and write,
birth, acquired Philippine citizenship by
may be a witness to the execution of a
naturalization after their marriage. During
will. their marriage the couple acquired

substanial landholdings in London and in


(C). In either of the above instances, must
Makati. Paula bore John three children,
the will be read to him? (1%)
Peter, Paul and Mary. In one of their trips to
London, the couple executed a joint will
SUGGESTED ANSWER:
appointing each other as their heirs and

If Stevie makes a will, the will must be

read to him twice, once by one of the

Never Let The Odds Keep You From Pursuing What You Know In Your Heart You Were Meant To Do.-
Leroy Satchel Paige
Page 61 of 180
Civil Law Q&As (2007-2013) hectorchristopher@yahoo.com dbaratbateladot@gmail.com

Coming from the hospital, Clara insisted on


signing her will by thumb mark and said
providing that upon the death of the

survivor between them the entire estate


would go to Peter and Paul only but the two

could not dispose of nor divide the London


estate as long as they live. John and Paul

died tragically in the London Subway


terrorist attack in 2005. Peter and Paul filed

a petition for probate of their parent's will


before a Makati Regional Trial Court.

(C). Is the testamentary prohibition against


the division of the London estate valid? (2%)

SUGGESTED ANSWER:

No. the testamentary prohibition against

the division of the London estate is void

(Art. 870, Civil Code). A testator,

however, may prohibit partition for a

period which shall not exceed twenty

(20) years (Art. 870 in relation to Art.

494, par 3, Civil Code).

Wills; Witnesses to a Will, Presence

required; Thumbmark as Signature

(2007)

No.VI. Clara, thinking of her mortality,

drafted a will and asked Roberta, Hannah,


Luisa and Benjamin to be witnesses.

During the day of signing of her will, Clara


fell down the stairs and broke her arms.
Probate should be denied. The

that she can sign her full name later. While requirement that the testator and at

the will was being signed, Roberta least three (3) witnesses must sign all in

experienced a stomach ache and kept going the "presence" of one another was not
to the restroom for long periods of time. complied with. Benjamin who notarized
Hannah, while waiting for her turn to sign the will is disqualified as a witness,
th
the will, was reading the 7 Harry Potter hence he cannot be counted as one of
book on the couch, beside the table on the three witnesses (Cruz v. Villasor, 54
which everyone was signing. Benjamin,
SCRA 31, 1973). The testatrix and the
aside from witnessing the will, also offered
other witnesses signed the will not in
to notarize it. A week after, Clara was run
the presence of Roberta because she was
over by a drunk driver while crossing the
in the restroom for extended periods of
street in Greenbelt.
time. Inside the restroom, Roberta could
May the will of Clara be admitted to not have possibly seen the testatrix and
probate? Give your reasons briefly. (10%) the other witnesses sign the will by

merely casting her eyes in the proper


SUGGESTED ANSWER:
direction (Jaboneta v. Gustilo, 5 Phil

541, 1906; Nera v. Rimando, 18 Phil

Never Let The Odds Keep You From Pursuing What You Know In Your Heart You Were Meant To Do.-
Leroy Satchel Paige
Page 62 of 180
Civil Law Q&As (2007-2013) hectorchristopher@yahoo.com dbaratbateladot@gmail.com

Jennifer acquired it and placed it in his


bedroom. In 1990, Brad and Jennifer broke
451, 1914). Therefore, the testatrix
up. While Brad was mending his broken
signed the will in the presence of only
heart, he met Angie and fell in love.
two witnesses, and only two witnesses

signed the will in the presence of the

testatrix and of one another.

It is to be noted, however, that the

thumb mark intended by the testator to

be his signature in executing his last will

and testament is valid (Payad v.

Tolentino, 62 Phil 848, 1936; Matias v.

Salud, L-104 Phil 1046, 23 June, 1958).

The problem, however, states that Clara

"said that she can sign her full name

later;" Hence, she did not consider her

thumb mark as her "complete" signature,

and intended further action on her part.

The testatrix and the other witness

signed the will in the presence of

Hannah, because she was aware of her

function and role as witness and was in a

position to see the testatrix and the

other witnesses sign by merely casting

her eyes in the proper direction.

Donation

Donations; Formalities; In Writing (2007)

No. VIII. In 1986, Jennifer and Brad were


madly in love. In 1989, because a certain

Picasso painting reminded Brad of her,


cannot validly bequeath the same to

Because the Picasso painting reminded Angie (Art. 930, NCC). Even assuming

Angie of him, Brad in his will bequeathed that the painting was impliedly given or

the painting to Angie. Brad died in 1995. donated by Jennifer to Brad, the

Saddened by Brad's death, Jennifer asked donation is nevertheless void for not
for the Picasso painting as a remembrance being in writing. The Picasso painting
of him. Angie refused and claimed that
must be worth more than 5,000 pesos.
Brad, in his will, bequeathed the painting
Under Art. 748, NCC, the donation and
to her. Is Angie correct? Why or why not?
acceptance of a movable worth more
(10%)
than 5,000 pesos must be in writing,

SUGGESTED ANSWER: otherwise the donation is void. The

donation being void, Jennifer remained


NO. Angie is not correct. The Picasso
the owner of the Picasso painting and
painting is not given or donated by
Brad could not have validly disposed of
Jennifer to Brad. She merely "placed it
said painting in favor of Angie in his will.
in his bedroom." Hence, she is still the

owner of the painting. Not being the ALTERNATIVE ANSWER:

owner of the Picasso painting, Brad

Never Let The Odds Keep You From Pursuing What You Know In Your Heart You Were Meant To Do.-
Leroy Satchel Paige
Page 63 of 180
Civil Law Q&As (2007-2013) hectorchristopher@yahoo.com dbaratbateladot@gmail.com

written. Such conditions, shall therefore,

YES. Angie is correct. Even assuming be disregarded but the donation remains

that there was void donation because the valid (Art. 727, NCC). On the other hand,

same was not in writing, Brad was in

uninterrupted possession of the Picasso

painting from 1989 to 1995, lasting for

six (6) years prior to his death. Brad has

already acquired ownership of the

painting through acquisitive

prescription. Under Art. 1132, NCC,

ownership of movables prescribes

through continuous possession for four

(4) years in good faith and for eight (8)

years without need of other conditions.

A void donation may be the basis of

possession in the concept of owner and

of just title for purposes of acquisitive

prescription.

Donations; Illegal & Impossible

Conditions (2007)

No.I. Distinguish the following concepts:

(B). Illegal and impossible conditions in a


simple donation v. illegal and impossible
conditions in an onerous donation. (5%)

SUGGESTED ANSWER:

Illegal and impossible conditions in a

simple donation are considered as not


given to me, I hereby freely,

voluntarily and irrevocably donate to


illegal and impossible donations imposed
her my one-hectare rice land covered
in an onerous donation shall annul the
by TCT No. 11550, located in San
donation (Art. 1183, NCC). This is so,
Fernando, Pampanga. This donation
because onerous donations are governed
shall take effect upon my death."
by the law on contracts (Art. 733, NCC).

The deed also contained Jennifer's signed


acceptance, and an attached notarized

declaration by Josefa and Jennifer that the


Donation; Inter Vivos (2013)
land will remain in Josefa's possession and

No.V. Josefa executed a deed of donation cannot be alienated, encumbered, sold or

covering a one-hectare rice land in favor of disposed of while Josefa is still alive.

her daughter, Jennifer. The deed specifically


Advise Jennifer on whether the deed is a
provides that:
donation inter vivos or mortis causa and

"For and in consideration of her love explain the reasons supporting your advice.

and service Jennifer has shown and (8%)

Never Let The Odds Keep You From Pursuing What You Know In Your Heart You Were Meant To Do.-
Leroy Satchel Paige
Page 64 of 180
Civil Law Q&As (2007-2013) hectorchristopher@yahoo.com dbaratbateladot@gmail.com

donation which state that the same will

SUGGESTED ANSWER: only take effect upon the death of the

donor and that there is a prohibition to


The donation is a donation inter vivos. alienate, encumber, dispose, or sell the

When the donor intends that the

donation shall take effect during the

lifetime of the donor, though the

property shall not be delivered till after

the donors death, this shall be a

donation inter vivos (Art. 729, Civil

Code).

The Civil Code prefers inter vivos

transmissions. Moreover, mortis causa

donations should follow the formalities of a

will (Art. 728, Civil Code). Here there is no

showing that such formalities were

followed. Thus, it is favorable to Jennifer

that the deed is a donation inter vivos.

Furthermore, what is most significant in

determining the type of donation is the

absence of stipulation that the donor

could revoke the donation; on the

contrary, the deeds expressly declare

them to be irrevocable, a quality

absolutely incompatible with the idea of

conveyances mortis causa where

revocability is the essence of the act, to

the extent that a testator cannot

lawfully waive or restrict his right of

revocation. The provisions of the deed of


As the donation is in the nature of a

same should be harmonized with its mortis causa disposition, the formalities

express irrevocability (Austria-Magat v. of a will should have been complied with

CA, G.R. No. 106755, Feb 1, 2002). under Art. 728 of the Civil Code,

otherwise, the donation is void and


ALTERNATIVE ANSWER: would produce no effect (The National

Treasure of the Philippines v. Vda. de


The donation is donation mortis causa.
Meimban, G.R. No. L-61023, Aug 22,

The deed clearly states that the donation 1984).

shall take effect upon the death of the

donor, Josefa. The donor, moreover, Property


retained ownership of the subject
Accretion; Alluvium (2008)
property as it was declared that the

property cannot be alienated, No. IX. The properties of Jessica and Jenny,
encumbered, sold or disposed of while who are neighbors, lie along the banks of

the donor is still alive. the Marikina River. At certain times of the
year, the river would swell and as the water

recedes, soil, rocks and other materials are

Never Let The Odds Keep You From Pursuing What You Know In Your Heart You Were Meant To Do.-
Leroy Satchel Paige
Page 65 of 180
Civil Law Q&As (2007-2013) hectorchristopher@yahoo.com dbaratbateladot@gmail.com

current of the water." Where the land is

deposited on Jessica's and Jenny's not formed solely by the natural effect of

properties. This pattern of the river the water current of the river bordering

swelling, receding and depositing soil and

other materials being deposited on the


neighbors' properties have gone on for

many years. Knowing his pattern, Jessica


constructed a concrete barrier about 2

meters from her property line and


extending towards the river, so that when

the water recedes, soil and other materials


are trapped within this barrier. After several

years, the area between Jessica's property


line to the concrete barrier was completely

filled with soil, effectively increasing


Jessica's property by 2 meters. Jenny's

property, where no barrier was constructed,


also increased by one meter along the side

of the river.

(A). Can Jessica and Jenny legally claim


ownership over the additional 2 meters and

one meter, respectively, of land deposited


along their properties?(2%)

SUGGESTED ANSWER:

Only Jenny can claim ownership over

the additional one meter of land

deposited along her property. Art. 457 of

the Civil Code provides that "to the

owners of lands adjoining the banks of

river belong the accretion which they

gradually receive from the effects of the


(B). If Jessica's and Jenny's properties are
registered, will the benefit of such
land but is also the consequences of the
registration extend to the increased area of
direct and deliberate intervention of
their properties? (2%)
man, it is man-made accretion and a

part of the public domain (Tiongco v. SUGGESTED ANSWER:


Director of Lands, 16 C.A. Rep 211, cited
If the properties of Jessica and Jenny
in Nazareno v. C.A., G.R. No. 98045, 26
are registered, the benefit of such
June 1996). Thus, Jessica cannot legally
registration does not extend to the
claim ownership of the additional 2
increased area of their properties.
meters of land along her property
Accretion does not automatically
because she constructed a concrete
become registered land because there is
barrier about 2 meters from her property
a specific technical description of the lot
causing deposits of soil and other
in its Torrens title. There must be a
materials when the water recedes. In
separate application for registration of
other words, the increase in her property
the alluvial deposits under the Torrens
was not caused by nature but was man-
System (Grande v. CA, G.R. No. L-17652,
made.
30 June, 1962).

Never Let The Odds Keep You From Pursuing What You Know In Your Heart You Were Meant To Do.-
Leroy Satchel Paige
Page 66 of 180
Civil Law Q&As (2007-2013) hectorchristopher@yahoo.com dbaratbateladot@gmail.com

up in the mountains and forests so that


gradually the land owned by Marciano
(C). Assume the two properties are on a cliff
increased in area by three hectares.
adjoining the shore of Laguna Lake. Jessica
and Jenny had a hotel built on the

properties. They had the erath and rocks


excavated from the properties dumped on

the adjoining shore, giving rise to a new


patch of dry land. Can they validly lay claim

to the patch of land? (2%)

SUGGESTED ANSWER:

No. Jessica and Jenny cannot validly lay

claim to the patch of land because in

order to acquire land by accretion, there

should be a natural and actual

continuity of the accretion to the land of

the riparian owner caused by natural ebb

and flow of the current of the river

(Delgado v. Samonte, CA-G.R. No. 34979-

R, 10 Aug 1966).

Accretion; Rights of the Riparian Owner

(2009)

No.XVI. Marciano is the owner of a parcel of


land through which a river runs out into

the sea. The land had been brought under


the Torrens System, and is cultivated by

Ulpiano and his family as farmworkers


therein. Over the years, the river has

brought silt and sediment from its sources


SUGGESTED ANSWER:
Ulpiano built three huts on this additional
Marcianos contention is correct. Since
area, where he and his two married
children live. On this same area, Ulpiano that accretion was deposited on his land

and his family planted peanuts, monggo by the action of the waters of the river

beans and vegetables. Ulpiano also and he did not construct any structure
regularly paid taxes on the land, as shown to increase the deposition of soil and
by tax declarations, for over thirty years. silt, Marciano automatically owns the

accretion. His real right of ownership is


When Marciano learned of the increase in
enforceable against the whole world
the size of the land, he ordered Ulpiano to
demolish the huts, and demanded that he including Ulpiano and his two married

be paid his share in the proceeds of the children. Although Marcianos land is

harvest. Marciano claims that under the registered, the three (3) hectares land

Civil Code, the alluvium belongs to him as a deposited through accretion was not
registered riparian owner to whose land the automatically registered. As an
accretion attaches, and that his right is unregistered land, it is subject to
enforceable against the whole world.
acquisitive prescription by third persons.

(A). Is Marciano correct? Explain. (3%)

Never Let The Odds Keep You From Pursuing What You Know In Your Heart You Were Meant To Do.-
Leroy Satchel Paige
Page 67 of 180
Civil Law Q&As (2007-2013) hectorchristopher@yahoo.com dbaratbateladot@gmail.com

accounting of the fruits he has gathered,

he has the right to deduct from the

Although Ulpiano and his children live in value of the fruits the expenses for

the three (3) hectare unregistered land

owned by Marciano, they are farm

workers; therefore, they are possessors

not in the concept of owners but in the

concept of mere holders. Even if they

possess the land for more than 30 years,

they cannot become the owners thereof

through extraordinary acquisitive

prescription, because the law requires

possession in the concept of the owner.

Payment of taxes and tax declaration are

not enough to make their possession one

in the concept of owner. They must

repudiate the possession in the concept

of holder by executing unequivocal acts

of repudiation amounting to ouster of

Marciano, known to Marciano and must

be proven by clear and convincing

evidence. Only then would his

possession become adverse.

(B). What rights, if any, does Ulpiano have


against Marciano? Explain. (3%)

SUGGESTED ANSWER:

Although Ulpiano is a possessor in bad

faith, because he knew he does not own

the land, he will lose the three huts he

built in bad faith and make an


CRC likewise sold to the spouses Rodriguez,
a 700-square meter land (Lot B) which is
production, gathering and preservation
adjacent to Lot A. Lot B has a present fair
of the fruits (Art 443, NCC).
market value of P1,500,000.

He may also ask for reimbursement of


The spouses Dela Cruz constructed a house
the taxes he has paid, as these are on Lot B, relying on their presentation of
charges on the land owned by Marciano. the CRC sales agent that it is the property
This obligation is based on a quasi- they purchased. Only upon the completion

contract (Art 2175, NCC). of their house did the spouses Dela Cruz

discovered that they had built on Lot B


owned by the spouses Rodriguez, not on Lot

A that they purchased. They spent P 1


Builder; Good Faith; Requisites (2013)
000,000 for the house.

No.VIII. Ciriaco Realty Corporation (CRC)


As their lawyer, advise the spouses Dela
sold to the spouses Del a Cruz a500-square
Cruz on their rights and obligations under
meter land (Lot A) in Paranaque. The land
the given circumstances, and the recourses
now has a fair market value of Pl,200,000.

Never Let The Odds Keep You From Pursuing What You Know In Your Heart You Were Meant To Do.-
Leroy Satchel Paige
Page 68 of 180
Civil Law Q&As (2007-2013) hectorchristopher@yahoo.com dbaratbateladot@gmail.com

(1) to appropriate as his own the works

and options open to them to protect their after payment of the indemnity provided

interests. (8%) for in Art 546 and 548, or

SUGGESTED ANSWER: (2) to oblige the one who built to pay the

price of the land.


Based on the fact as stated, the spouses

Dela Cruz as builders and the spouses

Rodriguez as land owners, are both in

good faith. The spouses Dela Cruz are

builder in good faith because before

constructing the house they exercised

due diligence by asking the Agent of CRC

the location of the lot A, and they relied

on the information given by the agent

who is presumed to know the identity of

the lot purchased by the Dela Cruz

spouses (Pleasantville v. CA, 253 SCRA

10, 1996). On the other hand, there is no

showing that the land owners, spouse

Rodriguez acted in bad faith. The facts

do not show that the building was done

with their knowledge and without

opposition on their part (Art 453, Civil

Code). The good faith is always presumed

(Art. 527, Civil Code).

The owner of the land on which anything

has been built, sown, or planted in good

faith shall have the right:


entitled to the right of retention pending

However, the builder cannot be obliged reimbursement of the expenses they

to buy the land if its value is incurred or the increase in value which

considerable more than that of the the thing may have acquired by reason of

building.. In such case, he shall pay the improvement (Art 546, Civil Code).

reasonable rent of the owner of the land Thus, the spouses Dela Cruz may

does not choose to appropriate the demand P1,000,000.00 as payment of

building or trees after proper indemnity the expenses in building the house or

(Art 448, Civil Code). increase in value of the land because of

the house as a useful improvement, as


The house constructed by the spouses may be determined by the court form
Dela Cruz is considered as a useful the evidence presented during the trial
expense, since it increased the value of (Depra v. Dumlao, 136 SCRA 475, 1985;
the lot. As such, should the spouses Technogas Phils v. CA, 268 SCRA 5,
Rodriguez decides to appropriate the 1997).
house, the spouses Dela Cruz are

Never Let The Odds Keep You From Pursuing What You Know In Your Heart You Were Meant To Do.-
Leroy Satchel Paige
Page 69 of 180
Civil Law Q&As (2007-2013) hectorchristopher@yahoo.com dbaratbateladot@gmail.com

been using this pathway (pathway A) since


1980.
Easement; Prescription; Acquisitive

Prescription (2009)

No. XI. TRUE or FALSE. Answer TRUE if

the statement is true, or FALSE if the


statement is false. Explain your answer in

not more than two (2) sentences.

(C). Acquisitive prescription of a negative

easement runs from the time the owner of


the dominant estate forbids, in a notarized

document, the owner of the servient estate


from executing an act which would be

lawful without the easement. (1%)

SUGGESTED ANSWER:

True. In negative easements, acquisitive

prescription runs from the moment the

owner of the dominant estate forbade, by

an instrument acknowledged before

notary public, the owner of the servient

estate from executing an act which

would be lawful without the easement

(Art. 621, NCC).

Easement; Right of Way (2013)

No.VII.In 2005, Andres built a residential


house on a lot whose only access to the

national highway was a pathway crossing


Brando's property. Andres and others have
determine the shortest and the least
prejudicial way through the servient
In 2006, Brand0 fenced off his property,
estates. After the survey, the engineer
thereby blocking Andres' access to the
concluded that pathway B is the longer
national highway. Andres demanded that
route and will need improvements and
part of the fence be removed to maintain
repairs, but will not significantly affect the
his old access route to the highway
use of Brando's property. On the other
(pathway A), but Brando refused, claiming
hand, pathway A that had long been in
that there was another available pathway
place, is the shorter route but would
(pathway B) for ingress and egress to the
significantly affect the use of Brando's
highway. Andres countered that pathway B
property.
has defects, is circuitous, and is extremely
inconvenient to use.
In light of the engineer's findings and the

circumstances of the case, resolve the


To settle their dispute, Andres and Brando
parties' right of way dispute. (6%)
hired Damian, a geodetic and civil engineer,

to survey and examine the two pathways


SUGGESTED ANSWER:
and the surrounding areas, and to

Never Let The Odds Keep You From Pursuing What You Know In Your Heart You Were Meant To Do.-
Leroy Satchel Paige
Page 70 of 180
Civil Law Q&As (2007-2013) hectorchristopher@yahoo.com dbaratbateladot@gmail.com

The first and fourth requisites are not


Andres is not entitled to the easement of
complied with. First, there is another
right of way for Pathway A. Pathway B
available outlet to the national highway
must be used.

The owner of a dominant estate may

validly obtain a compulsory right of way

only after he has established the

existence of four requisites, to wit:

(1) The (dominant) estate is surrounded

by other immovables and is without

adequate outlet to a public highway;

(2) After payment of the proper

indemnity;

(3) The isolation was not due to the

proprietors own acts; and

(4) The right of way claimed is at a point

least prejudicial to the servient estate,

and insofar as consistent with this rule,

where the distance from the dominant

estate to the public highway maybe the

shortest (Art 650, civil Code).

However, the Supreme Court has

consistently ruled that in case both

criteria cannot be complied with, the

right of way shall be established at the

point least prejudicial to the servient

estate.
to the servient estate is controlling

(Pathway B). Second, the right of way (Quimen v. Quimen and CA, G.R. No.

obtained (Pathway A) is not the least 112331, May 29, 1996).

prejudicial to Brandos property, as


(Note: It is not clear from the problem if there
evidence by the reports of the geodetic
exists an easement in favor of the lot
and civil engineer.
belonging to Andres and if Brandos lot is

burdened as a servient estate by a right of


When there is already an existing
way as a servient estate. If there is such an
adequate outlet from the dominant
easement burdening Brandos lot, was it
estate to the public highway, even if the
created as legal easement or as a voluntary
said outlet, for one reason or another, be
easement. If the used pathway was only a
inconvenient, the need to open up tolerance, then Brando may close it. Andres
another servitude is entirely unjustified must ask for the constitution of a legal
(Costabella Corporation v. CA, G.R. No. easement through Brandos lot by proving

80511, Jan 25, 1991). The rule that the the four requisites required by Art 649 and

easement of right of way shall be 65, Civil Code).

established at the point least prejudicial

Never Let The Odds Keep You From Pursuing What You Know In Your Heart You Were Meant To Do.-
Leroy Satchel Paige
Page 71 of 180
Civil Law Q&As (2007-2013) hectorchristopher@yahoo.com dbaratbateladot@gmail.com

(2) The sale of the property includes the


Easement; Right of Way (2010)

No.XIII. Franz was the owner of Lot E which

was surrounded by four (4) lots one of


which Lot C he also owned. He promised

Ava that if she bought Lot E, he would give


her a right of way in Lot C.

Convinced, Ava bought Lot E and, as


promised, Franz gave her a right of way in
Lot C.

Ava cultivated Lot E and used the right of


way granted by Franz.

Ava later found gainful employment abroad.


On her return after more than 10 years, the

right of way was no longer available to her


because Franz had in the meantime sold

Lot C to Julia who had it fenced.

(A). Does Ava have a right to demand from


Julia the activation of her right of way?
Explain. (2.5%)

SUGGESTED ANSWER:

Yes. Ava has the right to demand from

Julia the activation of the right of way,

for the following reasons:

(1) The easement of the right of way is a

real right which attaches to, and is

inseperable from, the estate to which it

belongs.
ten years of non-user, shall be computed

easement or servitude, even if the deed from the day it ceased to be used under

of sale is silent on the matter. Act 6341 (2) CC.

(5) Renunciation or waiver of an


(3) The vendee of the property in which a
easement must be specific, clear, express
servitude or easement exists cannot
and made in a public instrument in
close or put obstructions thereon to
accordance of Art 1358 of the New Civil
prevent the dominant estate from using
Code.
it.
ALTERNATIVE ANSWER:

(4) Avas working abroad for more than Yes. Ava has the right to demand from

ten (10) years should not be construed as Julia the activation of the right of way. A

non-user, because it cannot be implied voluntary easement of right of way, like

from the fact that she or those she left any other contract, could be

behind to cultivate the lot no longer use extinguished only by mutual agreement

the right of way. or by renunciation of the owner of the

dominant estate. Also, like any other


Note: Since a right of way is a

discontinuous easement, the period of

Never Let The Odds Keep You From Pursuing What You Know In Your Heart You Were Meant To Do.-
Leroy Satchel Paige
Page 72 of 180
Civil Law Q&As (2007-2013) hectorchristopher@yahoo.com dbaratbateladot@gmail.com

ALTERNATIVE ANSWER:

contract, an easement is generally


No. There was merely a promise to Ava
effective between parties, their heirs and
that a right of way shall be granted to
assignees, except in case where the

rights and obligations arising from the

contract are not transmissible by their

nature, or by stipulations or by provision

of law (Unisource Commercial v. Chung,

593 SCRA 530 [2009]).

(B). Assuming Ava opts to demand a right of


way from any of the owners of Lots A, B,
and D, can she do that? Explain. (2.5%)

SUGGESTED ANSWER:

Yes. Ava has the option to demand a

right of way on any of the remaining lots

of Franz more so after Franz sold lot C

to Julia. The essential elements of a

legal right of way under Art 649 and 650

of the New Civil Code are complied with.

ALTERNATIVE ANSWER:

Yes. Ava has the option to demand a

right of way from the other lots. The law

provides that whenever a piece of land

acquired by sale, exchange or partition is

surrounded by other estates of the

vendor, exchanger, or co-owner, he shall

be obliged to grant a right of way

without indemnity (Art 652, NCC).


lot which he (Blas) owns. While digging on
the lot in order to lay down the foudation of
her in lot C if Ava purchase lot E. The
the house, Adam hit a very hard object. It
promise was not reduced to writing (Obra
turned out to be the vault of the old Banco
v. Baldria, 529 SCRA 621 [2007]). Hence,
de las Islas Filipinas. Using a detonation
it was not or could not have been
device, Adam was able to open the vault
registered as to warn buyers of lot C
containing old notes and coins which were
about the existence of the easement on in circulation during the Spanish era. While
the property. Not having been annotated the notes and coins are no longer legal
on the TCT to lot C, the buyer acquired tender, they were valued at P100 million

lot C free from such right of way granted because of their historical value and the

to Ava. coins silver nickel content. The following

filed legal claims over the notes and coins:

(i). Adam, as finder;


Hidden Treasure (2008)
(ii). Blas, as owner of the property where
No. VIII. Adam, a building contractor, was they were found;
engaged by Blas to construct a house on a

Never Let The Odds Keep You From Pursuing What You Know In Your Heart You Were Meant To Do.-
Leroy Satchel Paige
Page 73 of 180
Civil Law Q&As (2007-2013) hectorchristopher@yahoo.com dbaratbateladot@gmail.com

made on the property of another, or of

(iii). Bank of the Philippine Islands, as the State and by chance, one-half of it

successor-in-interest of the owner of the shall belong to the finder who is not a

vault; and trespasser (Art. 438, Civil Code). In the

(iv). The Philippine Government because of


their historical value.

(A). Who owns the notes and coins? (4%)

SUGGESTED ANSWER:

The notes and coins are no longer owned

by the Bank of the Philippine Islands,

which has either lost or abandoned the

vault and its contents, and it has not

taken any effort to search, locate or

recover the vault. In any case, since the

vault is in actual possession of Adam,

BPI may attempt, in a judicial action to

recover, to rebut the presumption of

ownership in favor of Adam and Blas

(Art. 433, Civil Code). Hidden treasure is

any hidden and unknown deposit of

money, jewelry, or other precious

objects, the lawful ownership of which

does not appear. Given the age and

importance of the items found, it would

be safe to consider the vault, notes and

coins abandoned by BPI and its

predecessor (Art. 439, Civil Code). It

belongs to the owner of the land on

which it is found. When the discovery is


If either or both Adam and Blas are
present case, Adam, as finder, and Blas,
adjudged as owners, the notes and coins
as owner of the land, are entitled to
shall be deemed part of their absolute
share 50-50 in the treasure. The
community or conjugal partnership of
government can only claim if it can
gains with their respective spouses (Art.
establish that the notes and coins are of
117, par 4, FC).
interest to science or the arts, then it

must pay just price of the things found,

to be divided equally between Adam and

Blas (Art. 438, Civil Code). Mortgage; Public or Private Instrument

(2013)
(B). Assuming that either or both Adam and

Blas are adjudged as owners, will the notes No.VI. Lito obtained a loan of P1,000,000

and coins be deemed part of their absolute from Ferdie, payable within one year. To

community or conjugal partnership of gains secure payment, Lito executed a chattel

with their respective spouses? (2%) mortgage on a Toyota Avanza and a real
estate mortgage on a 200-square meter
SUGGESTED ANSWER: piece of property.

Never Let The Odds Keep You From Pursuing What You Know In Your Heart You Were Meant To Do.-
Leroy Satchel Paige
Page 74 of 180
Civil Law Q&As (2007-2013) hectorchristopher@yahoo.com dbaratbateladot@gmail.com

It is important to note that under Art.


(A) Would it be legally significant - from the
1358 of the Civil Code, all the other
point of view of validity and enforceability -
contracts where the amount involved
if the loan and the mortgages were in public

or private instruments? (6%)

SUGGESTED ANSWER:

From the point of view of validity and

enforceability, there would be legal

significance if the mortgage was in a

public or private instrument. As for the

loan, there is no legal significance

except of interest were charged on the

loan, in which case, the charging of

interest must be in writing.

A contract of loan is a real contract and

is perfected upon delivery of the object

of the obligation (Art 1934, Civil Code).

Thus, a contract of loan is valid and

enforceable even if it is neither in a

private nor in a public document.

As a rule, contracts shall be obligatory in

whatever form they may have been

entered into provided all the essential

requisites for their validity are present.

With regards to its enforceability, a

contact of loan is not among those

enumerated under Art. 1403 (2) of the

Civil Code, which are covered by the

Statute of Frauds.
order that a mortgage may be validly

exceeds Five Hundred pesos (P500.00) constituted the document in which it

must appear in writing, even in private appears be recorded. If the instrument is

one. However, the requirement is not for not recorded, the mortgage is

validity of the contract, but only for its nevertheless valid and binding between

greater efficacy. the parties. Hence, for validity of both

chattel and real estate mortgages, they


With regard to the chattel mortgage, Art. must appear in a public instrument. But
1508, the Chattel Mortgage Law, requires the purpose of enforceability, it is
an affidavit of good faith stating that the submitted that the form of the contract,
chattel mortgage is supposed to stand as whether in a public or private document,
security of the loan; thus, for the would be immaterial (Mobil Oil v.
validity of the chattel mortgage, it must Diocaresa, 29 SCRA 656, 1969).
be in a public document and recorded in

the Chattel Mortgage Register in the Also, under Art 1358, acts and contracts

Register of Deeds. A real estate which have for their object the creation

mortgage, under the provisions of Art. or transmission of real rights over

2125 of the Civil Code, requires that in immovable property must be in a public

Never Let The Odds Keep You From Pursuing What You Know In Your Heart You Were Meant To Do.-
Leroy Satchel Paige
Page 75 of 180
Civil Law Q&As (2007-2013) hectorchristopher@yahoo.com dbaratbateladot@gmail.com

intention to become the owner thereof.

document for greater efficacy and a real Possession, on the other hand, is the

estate mortgage is a real right over holding of the thing or an enjoyment of a

immovable property.

Occupation vs. Possession (2007)

No.I. Distinguish the following concepts:

(A). Occupation v. possession. (5%)

SUGGESTED ANSWER:

Occupation is an original mode of

acquiring ownership (Art. 712, NCC).

Things appropriable by nature which are

without an owner, such as animals that

are the object of hunting and fishing,

hidden treasure and abandoned

movables, are acquired by occupation

(Art. 713, NCC). However, ownership of a

piece of land cannot be acquired by

occupation (Art. 714, NCC).

ALTERNATIVE ANSWER:

Occupation is a mode of acquiring

dominion by the seizure of corporeal

things which have no owner, with the

intention of acquiring the ownership

thereof. It is an original mode of

acquiring ownership upon seizure of a

res nullius by the occupant who has the


No. XI. TRUE or FALSE. Answer TRUE if
right. Possession may be the real right of
the statement is true, or FALSE if the
possession or jus possessiones or it can
statement is false. Explain your answer in
be merely the right to possess or jus
not more than two (2) sentences.
possedendi, which are among the basic

rights of ownership. If the real right of (D). The renunciation by a co-owner of his

possession is possession in the concept undivided share in the co-owned property

of owner, but subject to certain in lieu of the performance of his obligation

limitations, it may ripen into full to contribute to taxes and expenses for the
preservation of the property constitutes
ownership of the thing or property right
dacion en pago. (1%)
through acquisitive prescription

depending on whether it is a case of


SUGGESTED ANSWER:
ordinary or extraordinary prescription True, Under the Civil Code, a co-owner
and whether the property is movable or may renounce his share in the co-owned
immovable. property in lieu of paying for his share in

the taxes and expenses for the

preservation of the co-owned property.


Ownership; Co-Ownership (2009)

Never Let The Odds Keep You From Pursuing What You Know In Your Heart You Were Meant To Do.-
Leroy Satchel Paige
Page 76 of 180
Civil Law Q&As (2007-2013) hectorchristopher@yahoo.com dbaratbateladot@gmail.com

(A). Can Cathy lawfully ask for demolition of


In effect, there is dacion en pago
Bobby's house? (3%)
because the co-owner is discharging his

monetary obligation by paying it with his

non-monetary interest in the co-owned

property. The fact that he is giving up

his entire interest simply means that he

is accepting the value of his interest as

equivalent to his share in the taxes and

expenses of preservation.

Ownership; Co-Ownership (2008)

No. VI. Alex died without a will, leaving only

an undeveloped and untitled lot in Tagiug


City. He is survived by his wife and 4

children. His wife told the children that she


is waiving her share in the property, and

allowed Bobby, the eldest son who was


about to get married, to construct his

house on of the lot, without however


obtaining the consent of his siblings. After

settlement of Alex's estate and partition


among the heirs, it was discovered that

Bobby's house was constructed on the


portion allocated to his sister, Cathy asked

Bobby to demolish his house and vacate the


portion alloted to her. In leiu of demolition,

Bobby offered to purchase from Cathy the


lot portion on which his house was

constructed. At that time, the house


constructed was valued at P350.000.
dominion or ownership such as

SUGGESTED ANSWER: construction of a house." In the present

case, of Alex is the real owner of the


Yes, Cathy can lawfully ask for the undeveloped and untitled lot in Taguig,
demolition of Bobby's house. Where co-ownership is created among his wife
there are two or more heirs, the whole and four children over said property
estate of the decedent, is, before upon his death. Since the construction
partition, owned in common by such of the house by Bobby was done without
heirs, subject to the payment of debts of obtaining the consent of his siblings, the
the deceased (Art. 1078, Civil Code), alteration effected is illegal. Bobby is
Under the rules on co-ownership, "none considered to be in bad faith and as a
of the co-owners shall, without the sanction for his conduct, he can be
consent of the others make alterations compelled by Cathy to demolish or
in the thing owned in common, even remove the structure at his own
though benefits for all would results expense.
therefrom." In Cruz v. Catapang, G.R. No.

164110, 12 Feb., 2008, the Court held (B). Can Bobby legally insist on purchasing

that "alterations include any act of strict the land? (2%)

Never Let The Odds Keep You From Pursuing What You Know In Your Heart You Were Meant To Do.-
Leroy Satchel Paige
Page 77 of 180
Civil Law Q&As (2007-2013) hectorchristopher@yahoo.com dbaratbateladot@gmail.com

that "docks and structures which,

SUGGESTED ANSWER: though floating, are intended by their

No. Bobby cannot legally insist on

purchasing the land. Being in bad faith,

he has no option to pay for the price of

the lot (Art. 450, Civil Code).

Property; Movable or Immovable (2007)

No.II. Manila Petroleum Co. owned and

operated a petroleum operation facility off


the coast of Manila. The facility was located

on a floating platform made of wood and


metal, upon which was permanently

attached the heavy equipment for the


petroleum operations and living quarters of

the crew. The floating platform likewise


contained a garden area, where trees,

plants and flowers were planted. The


platform was tethered to a ship, the MV

101, which was anchored to the seabed.

Please briefly give the reason for your


answers. (10%)

(A).Is the platform movable or immovable


property?

SUGGESTED ANSWER:

The platform is an immovable property

under Art. 415 (9) NCC, which provides


hence, it remains a movable property. If

nature and object to remain at a fixed the intention was to make the platform

place on a river, lake or coast." Since the stay permanent where it was moored, it

floating platform is a petroleum would not have been simply tethered to a

operation facility, it is intended to vessel but itself anchored to the seabed.

remain permanently where it is situated,


(B). Are the equipment and living quarters
even if it is tethered to a ship which is
movable or immovable property?
anchored to the seabed.

SUGGESTED ANSWER:
ALTERNATIVE ANSWER:
The thing and living quarters of the crew
The platform is a movable property
are immovable property under Art. 415
because it is attached to a movable
(3) NCC, classifies as an immovable
property, i.e. the vessel which was
"everything attached to an immovable in
merely anchored to the seabed. The fact
a fixed manner, in such a way that it
that the vessel is merely anchored to the
cannot be separated therefrom without
sea bed only shows that it is not
breaking the material or deterioration of
intended to remain at a fixed place;

Never Let The Odds Keep You From Pursuing What You Know In Your Heart You Were Meant To Do.-
Leroy Satchel Paige
Page 78 of 180
Civil Law Q&As (2007-2013) hectorchristopher@yahoo.com dbaratbateladot@gmail.com

(C). Are the trees, plants and flowers


immovable or movable property?
the object." Both the equipment and the

living quarters are permanently attached


SUGGESTED ANSWER:
to the platform which is also an

immovable. The equipment can also be

classified as an immovable property

under Art. 415 (5) NCC because such

equipment are "machinery, receptacles,

instruments or implements intended by

the owner of the tenement for an

industry or works which may be carried

on in a building or on a piece of land and

which tend directly to meet the needs of

the industry or works." It is logically

assumed that the petroleum industry

may be carried on in a building or on a

piece of land and the platform is

analogous to a building.

ALTERNATIVE ANSWER:

The equipment and living quarters of the

crew are movable properties since they

are attached to a platform which is also

a movable property, because it is simply

attached to a vessel is likewise a

movable property since it was merely

anchored on the seabed only shows that

it is not intended to remain at a fixed

place; hence, it remains a movable

property.
immovable. The platform is not an

The trees, plants and flowers planted in immovable property for the same reason

the garden area of the platform are already given in the Alternative Answer

immovable property under Art. 415 (2) to Item (a) above.

NCC which classifies as an immovable


Land Titles and Deeds
property "trees, plants and growing

fruits, while they are attached to the


Acquisition of Lands; Sale of Real
land or form an integral part of an
Property to an Alien (2009)
immovable, the petroleum operation

facility. No.XIX. In 1972, Luciano de la Cruz sold to

Chua Chung Chun, a Chinese citizen, a


ALTERNATIVE ANSWER: parcel of land in Binondo. Chua died in

1990, leaving behind his wife and three


The trees, plants and flowers planted in
children, one of whom, Julian, is a
the garden area of the platform are
naturalized Filipino citizen. Six years after
movable property because they are not
Chuas death, the heirs executed an
permanently attached t the land and do extrajudicial settlement of estate, and the
not form an integral part of an parcel of land was allocated to Julian. In

Never Let The Odds Keep You From Pursuing What You Know In Your Heart You Were Meant To Do.-
Leroy Satchel Paige
Page 79 of 180
Civil Law Q&As (2007-2013) hectorchristopher@yahoo.com dbaratbateladot@gmail.com

2007, Luciano filed suit to recover the land Non-Registrable Properties (2007)
he sold to Chua, alleging that the sale was
No.IV. (B). What properties are not
void because it contravened the
registrable? (5%)
Constitution which prohibits the sale of
private lands to aliens. Julian moved to
Supply this information.
dismiss the suit on grounds of pari delicto,

laches and acquisitive prescription. Decide SUGGESTED ANSWER:


the case with reasons. (4%)
The following properties are not
SUGGESTED ANSWER: registrable:

The case must be dismissed. Julian, who (1.) Properties of the Public dominion;
is a naturialized Filipino citizen and to

whom the property was allocated in a n (2.) Properties for public use or public

extra-judicial partition of the estate, is service;

now the owner of the property. The


(3.) Inalienable lands of the public
defect in ownership of the property of
domain;
Julians alien father has already been

cured by its transfer to Julian. It has (4.) Military installations, civil and quasi-

been validated by the transfer of the public lands; and

property to a Filipino citizen. Hence,


(5.) All lands not classified as alienable
there is no more violation of the
and disposable.
Constitution because the subject real

property is now owned by a Filipino


ALTERNATIVE ANSWER:
citizen (Halili v. CA, 287 SCRA 465,

[1998]). Further, after the lapse of 35 (1). Properties of public dominium

year, laches has set in and the motion to intended for public use, like roads,

dismiss may be granted, for the failure of canals, rivers, torrents, ports and

Luciano to question the ownership of bridges constructed by the State, banks,

Chua before its transfer of ownership to shores, roadsteads, and the like, are

Julian. incapable of private appropriation, much

less registration (Art. 420 NCC). This


includes public markets, public plazas,

municipal streets and public buildings

Never Let The Odds Keep You From Pursuing What You Know In Your Heart You Were Meant To Do.-
Leroy Satchel Paige
Page 80 of 180
Civil Law Q&As (2007-2013) hectorchristopher@yahoo.com dbaratbateladot@gmail.com

to the public domain, and can only be

(Municipality of Antipolo v. Zapanta, 133 acquired by lease if not needed by the

SCRA 820, 1986; Martinez v. CA, 56 government for public or quasi-public

SCRA 647, 1974; Navera v. Quicho, 5 purposes.

SCRA 454, 1962).

(2.) Lands proclaimed or classified as

forest, timberlands, mineral lands and

national parks. Under Sec 2, Art XII,

Constitution of the Philippines, these

lands are inalienable.

(3.) Lands that are reserved by law or

Presidential proclamation for military,

civic or quasi-public purpose, Under Sec

88, Chapter XII of the Public Land Act,

such lands shall be inalienable and shall

not be subject to occupation, entry, sale,

lease or other disposition.

(4.) In general, all lands of the public

domain that has not been classified as

alienable and disposable under the Public

Land Act.

(5.) Lands that form part of the seabed,

riverbed or lakebed. These lands are not

susceptible to private appropriation.

(6.) Foreshore lands is that strip of land

that lies between the high and low water

marks and alternately wet and dry

according to the flow of the tide belong


on the property. Anthony is not aware of the
defect in Bert's title, but has been in actual
(7.) Lands reclaimed by the government
physical possession of the property from
from the sea, lakes, or other bodies of
the time he bought it from Bert, who had
water are disposed or acquisible only by
never been in possession of the property for
lease and not otherwise, under the
one year.
Public Land Act.

(A). Can Anthony acquire ownership of the

property by acquisitive prescription? How


many more years does he have possess it to
Prescription; Acquisitive Prescription
acquire ownership? (2%)
(2008)

SUGGESTED ANSWER:
No. VII. Anthony bought a piece of untitled
agricultural land from Bert. Bert, in turn,
Yes, Anthony can acquire ownership of
acquired the property by forging carlo's
the property through acquisitive
signature in a deed of sale over the
prescription. In the present case,
property. Carlo had been in possession of
Anthony is a buyer/possessor in good
the property for 8 years, declared it for tax

purposes, and religiously paid all taxes due

Never Let The Odds Keep You From Pursuing What You Know In Your Heart You Were Meant To Do.-
Leroy Satchel Paige
Page 81 of 180
Civil Law Q&As (2007-2013) hectorchristopher@yahoo.com dbaratbateladot@gmail.com

Yes, Carlos can appropriate only a

faith because he was not aware of the portion of the standing crops on the

defect in Bert's title (Art. 526, Civil property once he recovers possession.

Code). As such, Anthony can acquire Anthony being a possessor in good faith,

ownership and other real rights over

immovable property through open,

continuous possession of 10 years (Art.

1134, Civil Code). Anthony needs nine

(9) more years of possession, in addition

to his one (1) year of possession in good

faith.

(B).If Carlo is able to legally recover his

property, can he require Anthony to


account for all the fruits he has harvested

from the property while in possession? (2%)

SUGGESTED ANSWER:

If Carlo is able to legally recover his

property, he cannot require Anthony to

account for all the fruits harvested from

the property. Anthony is entitled to the

fruits harvested in good faith before his

possession was legally interrupted (Art.

544, Civil Code).

(C).If there are standing crops on the


property when Carlo recovers possession,
can Carlo appropriate them? (2%)

SUGGESTED ANSWER:
SUGGESTED ANSWER:

shall have a right to a part of the Yes, it prescribes in five (5) years. If the
expenses of cultivation, and to a part of real property mortgaged is judicially
the net harvest of the standing crops, foreclosed, the action for judicial
both in proportion to the time of the foreclosure should be filed within a
possession (Art 545, Civil Code). period of ten (10) years. The request for

issuance of a writ of possession should

be filed upon motion of the winning


Prescription; Judicially Foreclosed Real bidder within five (5) years after the
Property Mortgage (2012) judgment of foreclosure. The writ of

possession is an order commanding the


No.IX.a) Does the right to request for the
sheriff to place a person named therein
issuance of a writ of possession over a
foreclosed real property prescribe in five (5) in possession of real property (BPI v.

years? (5%) Icot. G.R. No. 168081, Oct 12, 2009).

Never Let The Odds Keep You From Pursuing What You Know In Your Heart You Were Meant To Do.-
Leroy Satchel Paige
Page 82 of 180
Civil Law Q&As (2007-2013) hectorchristopher@yahoo.com dbaratbateladot@gmail.com

Bank registered the Sheriff's Deed of


Conveyance in the day book of the Register
Purchaser in Good Faith; Mortgaged

Property (2008)

No. XIX. Juliet offered to sell her house and

lot, together with all the furniture and


appliances therein to Dehlma. Before

agreeing to purchase the property, Dehlma


went to the Register of Deeds to verify

Juliet's title. She discovered that while the


property was registered in Juliet's name
under the Land Registration Act, as
amended by the Property Registration

Decree, it property, Dehlma told Juliet to


redeem the property from Elaine, and gave

her an advance payment to be used for


purposes of realesing the mortgage on the

property. When the mortgage was released,


Juliet executed a Deed of Absolute Sale over

the property which was duly registered with


the Registry of Deeds, and a new TCT was

issued in Dehlma's name. Dehlma


immediately took possession over the house

and lot and the movables therein.


Thereafter, Dehlma went to theAssessor's

Office to get a new tax declaration under


her name. She was surprised to find out

that the property was already declared for


tax purposes in the name of XYZ Bank

which had foreclosed the mortgage on the


property before it was sold to her. XYZ Bank

was also the purchaser in the foreclosure


sale of the property. At that time, the

property was still unregistered but XYZ


bought the property. Thus, Dehlma is a

of Deeds under Act. 3344 and obtained a purchaser in good faith (Mathay v. CA,

tax declaration in its name. G.R. No. 115788, 17 Sept, 1998).

(A). Was Dehlma a purchaser in good faith? (B). Who as between Dehlma and XYZ Bank

(2%) has a better right to the house and lot? (2%)

SUGGESTED ANSWER: SUGGESTED ANSWER:

Yes, Dehlma is a purchaser in good faith. Between Dehlma and XYZ Bank, Dehlma

In the present case, before Dehlma has a better right to the house and lot.

bought the property, she went to the After the release of the mortgage, the

Register of Deeds to verify Juliet's title. Deed of Absolute Sale was registered and

When she discovered that the property a new title was issued in Dehlma's name.

was mortgaged to Elaine, she gave an Act 3344 is applicable exclusively to

advance payment so that Juliet could instruments resulting from agreement of

release the mortgage. It was only after parties thereto and does not apply to

the mortgage was released and free from deeds of a sheriff conveying to a

the claims of other persons that Dehlma

Never Let The Odds Keep You From Pursuing What You Know In Your Heart You Were Meant To Do.-
Leroy Satchel Paige
Page 83 of 180
Civil Law Q&As (2007-2013) hectorchristopher@yahoo.com dbaratbateladot@gmail.com

manual containing a summary of the


pertinent laws and regulations relating to
purchaser unregistered lands sold to him

under execution (Williams v. Suer, 49

Phil. ,534).

(C). Who owns the movables inside the


house? (2%)

SUGGESTED ANSWER:

Dehlma owns the movables because

when she acquired the house and lot

from Juliet, all the furniture and

appliances therein were included in the

sale. As owner of the real property,

Dehlma also owns the movables found

therein (Art. 542, Civil Code).

Registration; Governing Law (2007)

No.IV. Bedrock Land & Property

Development Corp. is a development


company engaged in developing and selling

subdivisions, condominium units and


industrial estates. In order to replenish its

inventories, it embarked on an aggressive


land banking program. It employed "scouts"

who roam all over the Philippines to look for


and conduct investigations on prospective

sites for acquisition and development,


whether developed, semi-developed or raw

land. The management of Bedrock asks you


as the company counsel to prepare a
ALTERNATIVE ANSWER:
land registration and acquisition of title to
land. The manual should include the In general, the governing law relating to
following items:
registration and acquisition of title to

land is Act 496 of 1902 as amended by


(A). What is the governing law? (5%)
PD 1529, otherwise known as Property
SUGGESTED ANSWER: Registration Decree of June 11, 1978.

The governing law is the Land (1.) Chapter III-I governs original
Registration Act as amended by Property registration of land title under the
Registration Decree (Act 496 as amended Torrens System by voluntary ordinary
by PD 1529). judicial proceedings.

[Note: It is respectfully recommended (2.) Chapter II-II governs compulsory


that full credit be given to examinees registration of lands through cadastral
who did not give the exact title or proceedings.
number of the law but merely stated a

description of the law.]

Never Let The Odds Keep You From Pursuing What You Know In Your Heart You Were Meant To Do.-
Leroy Satchel Paige
Page 84 of 180
Civil Law Q&As (2007-2013) hectorchristopher@yahoo.com dbaratbateladot@gmail.com

(b) Amendment and alteration of


(3.) Section 103 governs registration of
certificate of title;
homestead, sales, free patent under CA

No. 141, as amended, otherwise known

as the Public Land Act.

(4.) Section 104 governs registration of

certificates of land transfers,

emancipation patents and Certificates of

Land Ownership Award (CLOA) under

Comprehensive Land Reform Law.

(5.) Chapter V governs the registration of

land dealings on registered land like

conveyances, transfers, mortgages,

leases, powers of attorney, trusts and

similar contracts inter vivos.

(6.) Chapter V-II governs the registration

of involuntary dealings on registered

land like attachments, adverse claims,

enforcement of liens on registered land,

notices of lis pendens. (7.) Chapter VI

governs the registration of judgments,

orders and partitions, condemnation in

eminent domain proceedings, judicial

and extra-judicial settlement of estates.

(8.) Sections 107, 108 and 109 govern

petitions and actions after original

registration like: (a).Compulsory

surrender of withheld owner's duplicate

certificate of title;
register a deed or when he is in doubt as

(c) Replacement of lost or destroyed to what action to take on an instrument

owner's duplicate certificate of title. presented for registration.

(9.) R.A. No. 26 governs judicial

reconstitution of lost or destroyed


Registration; Party Who First took
originals of the certificate of title.
Possession (2013)

(10.) R.A. No. 6732 governs No.IX.Rica petitioned for the annulment of
administrative reconstitution of lost or her ten-year old marriage to Richard.
destroyed original certificates of title. Richard hired Atty. Cruz to represent him in
the proceedings. In payment for Atty. Cruz's
(11.) Section 113 governs the acceptance and legal fees, Richard conveyed
registration of instruments affecting to Atty. Cruz a parcel of land in Taguig that
unregistered private lands. he recently purchased with his lotto
winnings. The transfer documents were
(12.) Section 117 governs "consultas,"
duly signed and Atty. Cruz
where the Register of Deeds refuses to

Never Let The Odds Keep You From Pursuing What You Know In Your Heart You Were Meant To Do.-
Leroy Satchel Paige
Page 85 of 180
Civil Law Q&As (2007-2013) hectorchristopher@yahoo.com dbaratbateladot@gmail.com

urged that the land itself is not the

immediately took possession by fencing off object of the litigation, the annulment of

the property's entire perimeter. marriage, if granted, will carry with it

Desperately needing money to pay for his


mounting legal fees and his other needs

and despite the transfer to Atty. Cruz,


Richard offered the same parcel of land for

sale to the spouses Garcia. After inspection


of the land, the spouses considered it a

good investment and purchased it from


Richard. Immediately after the sale, the

spouses Garcia commenced the


construction of a three-story building over

the land, but they were prevented from


doing this by Atty. Cruz who claimed he has

a better right in light of the prior


conveyance in his favor.

Is Atty. Cruz's claim correct? (8%)

SUGGESTED ANSWER:

No. Atty. Cruz is not correct. At first

glance, it may appear that Atty. Cruz is

the one who has the better right because

he first took possession of the property.

However, a lawyer is prohibited under

Art 1491 of the Civil Code from

acquiring the property and rights which

may be the object of any litigation in

which they may take part by virtue of

their profession. While the suit is for

annulment of marriage and it may be


property of the community or the

the liquidation of the absolute conjugal property without the consent of

community or conjugal partnership of the other spouse is void (Art 96 and Art

the spouses as the case may be (Art. 50 124, Family Code).

in relation to Art 43 of the Family Code).

Richard purchased the land with his

lotto winnings during the pendency of Registration; Requisites; Proof (2013)

the suit for annulment and on the


No.X. Manuel was born on 12 March 1940
assumption that the parties are
in a 1 000-square meter property where he
governed by the regime of absolute
grew up helping his father, Michael,
community or conjugal partnership,
cultivate the land. Michael has lived on the
winnings from gambling or betting will property since the land was opened for
form part thereof. Also, since the land is settlement at about the time of the

part of the absolute community or Commonwealth government in 1935, but

conjugal partnership of Richard and for some reason never secured any title to

Rica, it may not be sold or alienated the property other than a tax declaration in

his name. He has held the property through


without the consent of the latter and

any disposition or encumbrance of the

Never Let The Odds Keep You From Pursuing What You Know In Your Heart You Were Meant To Do.-
Leroy Satchel Paige
Page 86 of 180
Civil Law Q&As (2007-2013) hectorchristopher@yahoo.com dbaratbateladot@gmail.com

has been in open, continuous and

the years in the concept of an owner and notorious possession and occupation

his stay was uncontested by others. He has thereof under bona fide claim of

also conscientiously and continuously paid

the realty taxes on the land.

Michael died in 2000 and Manuel - as


Michaels only son and heir -now wants to

secure and register title to the land in his


own name. He consults you for legal advice

as he wants to perfect his title to the land


and secure its registration in his name.

(A) What are the laws that you need to


consider in advising Manuel on how he can

perfect his title and register the land in his


name? Explain the relevance of these laws

to your projected course of action. (4%)

SUGGESTED ANSWER:

(Note: With all due respect, it is

recommended that the examiner accept and

give full credit to any of the answers given in

each of the following paragraphs.)

I would advice Manuel to file an

application for registration under Sec 14

of Pres. Decree No. 1529, or the Property

Registration Decree (PRD), specifically

Sec14 (1) which requires (a) that the land

applied for forms part of the alienable

and disposable (A & D) portion of the

public domain, and (b) that the applicant


Manuel could also file an application for
ownership since June 12, 1945, or
confirmation of imperfect or incomplete
earlier. However, it is only necessary
title through judicial legalization
that the land is already declared A & D
under Sec. 48 (b) of CA no. 141, or the
land at the time for application for
Public Land Act (PLA). But, as held in
registration is filed (Malabanan v.
Malabanan, there is no substantial
Republic, G.R. No. 180067, June 30,
difference between this provision and
2009).
Sec 14 (1) of the PRD. Both refer to

agricultural lands already classified as


Manuel could also invoke Sec 14 (2) of
alienable and disposable at the time the
the same Decree, which allows
application is filed, and require
registration through ordinary acquisitive
possession and occupation since June
prescription for thirty years, provided,
12, 1945. The only difference is that
however, that the land is patrimonial
under the PRD, there already exists a
in character, i.e. already declared by the
title which is to be confirmed, whereas
government (a) as A & D, and (b) no
under the PLA, the presumption is that
longer needed for public use or public
land is still public land (Republic v.
service (Malabanan, supra).

Never Let The Odds Keep You From Pursuing What You Know In Your Heart You Were Meant To Do.-
Leroy Satchel Paige
Page 87 of 180
Civil Law Q&As (2007-2013) hectorchristopher@yahoo.com dbaratbateladot@gmail.com

(B) What do you have to prove to secure


Manuel's objectives and what
Aquino, G.R. No. L-33983, January 27,
documentation are necessary? (4%)
1983).

SUGGESTED ANSWER:
Manuel may also invoke vested rights

acquired under Rep. Act. No. 1942, dated

June 2, 1957, which amended Sec. 48 (b)

of the PLA by providing for a prescriptive

period of thirty years for judicial

confirmation of imperfect title. It must

only be demonstrated that possession

and occupation commenced on January

24, 1947 and the 30-year period was

completed prior to the effectivity of PD

No. 1073 on January 25, 1977. PD No.

1073 now requires possession and

occupation since June 12, 1945

(Republic v. Espinosa, G.R. No. 171514,

July 18, 2012).

Another alternative is for Manuel to

secure title through administrative

proceedings under the homestead or free

patent provisions of the PLA. The title

issued has the same efficacy and validity

as a title issued through judicial

proceedings, but with the limitations

that the land cannot be sold or disposed

of within five years from the issuance of

patent (Sec. 118, CA No. 141, as

amended).
valuable improvements like fencing the

Manuel has a the burden to overcome land, constructing a residential house

the presumption of State ownership by thereon, cultivating the land and

well-nigh incontrovertible evidence planting fruit bearing trees, declaring

(Ong v. Republic, G.R. No. 175746, the land for taxation purposes and

March 12, 2008). Accordingly, he must paying realty taxes, all of which are

show that ht eland is already classified corroborative proof of possession.

as A & D at the time the application for


To identify the land, he must submit the
registration is filed and that he has
tracing cloth plan or a duly-certified
been in possession and occupation
blueprint or whiteprint copy thereof
thereof in the manner required by law
(Director of Lands v. Reyes, G.R. No. L-
since June 12, 1945, or earlier.
27594, November 28, 1975; Director of

Manuel may tack his possession to that Lands v. CA and Iglesia ni Cristo, G.R.

of his predecessor-in-interest (Michael) No. L-56613, March 14, 1988).

by the testimony of disinterested and


To show the classification of the land as
knowledgeable eyewitnesses. Overt acts
A & D, the application must be
of possession may consist in introducing
accompanied by (1) a CENRO or PENRO

Never Let The Odds Keep You From Pursuing What You Know In Your Heart You Were Meant To Do.-
Leroy Satchel Paige
Page 88 of 180
Civil Law Q&As (2007-2013) hectorchristopher@yahoo.com dbaratbateladot@gmail.com

SUGGESTED ANSWER:
certification; and (2) a certified true

copy of the original classification

approved by the DENR Secretary

(Republic v. Bantigue, G.R.No. 162322,

March 14, 2012). A presidential or

legislative act may also be considered.

Remedies; Fraud; Rights of Innocent

Purchaser (2009)

No.IX. Before migrating to Canada in 1992,


the spouses Teodoro and Anita entrusted

all their legal papers and documents to


their nephew, Atty. Tan. Taking advantage

of the situation, Atty. Tan forged a deed of


sale, making it appear that he had bought

the couples property in Quezon City. In

2000, he succeeded in obtaining a TCT over


the property in his name. Subsequently,

Atty. Tan sold the same property to Luis,


who built an auto repair shop on the

property. In 2004, Luis registered the deed


of conveyance, and title over the property

was transferred in his name.

In 2006, the spouses Teodoro and Anita


came to the Philippines for a visit and

discovered what had happened to their


property. They immediately hire you as

lawyer. What action or actions will you


institute in order to vindicate their rights?

Explain fully. (4%)


(c). A criminal action for forgery or

falsification of public document;

I will institute the following actions

against Atty. Tan: (d). A complaint with the Supreme

Court/Integrated Bar of the Philippines

(a). A civil action for damage for the to disbar or suspend him or other

fraudulent transfer of the title in his disciplinary action for violation or the

name and to recover the value of the Code of Professional Ethics.

property;
Any action against Luis will not prosper

(b). An action against the National because he is an innocent purchaser for

Treasurer for compensation from the value. The Title to the land he bought

State Assurance Fund which is set aside was already in the name of the person

by law to pay those who lose their land who sold the property to him, and there

suffer damages as a consequence of the is nothing on the title which will make

operation of the Torrens system; him suspect about the fraud committed

by Atty. Tan.

Never Let The Odds Keep You From Pursuing What You Know In Your Heart You Were Meant To Do.-
Leroy Satchel Paige
Page 89 of 180
Civil Law Q&As (2007-2013) hectorchristopher@yahoo.com dbaratbateladot@gmail.com

Contracts Rescission of Contract; Fortuitous Event

(2008)

Contract to Sell vs. Conditional Contract


No.XVIII. AB Corp. entered into a contract
of Sale (2012)
with XY Corp. whereby the former agreed to

No.X.a) A contract to sell is the same as a construct the research and laboratory

conditional contract of sale. Do you agree? facilities of the latter. Under the terms of

Explain your answer. (5%) the contract, AB Corp. agreed to complete

the facility in 18 months, at the total


SUGGESTED ANSWER: contract price of P10 million. XY Corp. paid

50% of the total contract price, the balance


No. A contract to sell is a species of
to be paid upon completion of the work. The
conditional sale. The contract to sell
work stated immediately, but AB Corp. later
does not sell a thing or property; it sells
experienced work slippage because of labor
the right to buy property. A conditional
unrest in his company. AB Corp.'s
sale is a sale subject to the happening or employees claimed that they are not being
performance of a condition, such as paid on time; hence, the work slowdown. As
payment of the full purchase price, or of the 17th month, work was only 45%

the performance of other prestation to completed. AB Corp. asked for extension of

give, to do or not to do. Compliance with time, claiming that its labor problems is a

case of fortuitous event, but this was


the condition automatically gives the
denied by XY Corp. When it became certain
right to the vendee to demand the
that the contruction could not be finished
delivery of the object of the sale. In a
on time, XY Corp. sent written notice
contract to sell, however, the compliance
cancelling the contract, and requiring AB
with the condition does not
Corp. to immediately vacate the premises.
automatically sell the property to the

vendee. It merely gives the vendee the (A). Can the labor unrest be considered a

right to compel the vendor to execute fortuitous event? (1%)

the deed of absolute sale.


SUGGESTED ANSWER:

No. The labor unrest cannot be

considered a fortuitous event under Art.


1174 of the Civil Code. A fortuitous

event should occur independent of the

Never Let The Odds Keep You From Pursuing What You Know In Your Heart You Were Meant To Do.-
Leroy Satchel Paige
Page 90 of 180
Civil Law Q&As (2007-2013) hectorchristopher@yahoo.com dbaratbateladot@gmail.com

No. XI. TRUE or FALSE. Answer TRUE if

the statement is true, or FALSE if the


will of the debtor or without his

participation or aggravation (Paras, Civil

Code Annotated, vol. IV, 2000 ed., p

159). As mentioned in the facts, labor

unrest of the employees was caused by

AB Corp.'s failure to pay its employees

on time.

(B). Can XY Corp. unilaterrally and


immediately cancel the contract? (2%)

SUGGESTED ANSWER:

No, XY Corp. cannot unilaterally and

immediately cancel the contract. In the

absence of any stipulation for automatic

rescission, rescission must be judicial

(Art. 1191, Civil Code).

(C). Must AB Corp. return the 50%


downpayment? (2%)

SUGGESTED ANSWER:

AB Corp. need not return the 50% down

payment because 45% of the work was

already completed, otherwise, XY Corp.

would be unjustly enriching itself at the

expense of AB Corp.

Stipulation; Arbitration Clause (2009)


No.XV. Sarah had a deposit in a savings
account with Filipino Universal Bank in the
statement is false. Explain your answer in
amount of five million pesos
not more than two (2) sentences.
(P5,000,000.00). To buy a new car, she

(A). A clause in an arbitration contract obtained a loan from the same bank in the

granting one of the parties the power to amount of P1,200,000.00, payable in twelve

choose more arbitrators than the other monthly installments. Sarah issued in favor

renders the arbitration contract void. (1%) of the bank post-dated checks, each in the

amount of P100,000.00, to cover the twelve


SUGGESTED ANSWER: monthly installment payments. On the
True. The Civil Code provides that Any third, fourth and fifth months, the
clause giving one of the parties power to corresponding checks bounced.
choose more arbitrators than the other
The bank then declared the whole
is void and of no effect (Art 2045, NCC).
obligation due, and proceeded to deduct the
Obligations amount of one million pesos

(P1,000,000.00) from Sarahs deposit after


Extinguishment; Compensation (2009)
notice to her that this is a form of

Never Let The Odds Keep You From Pursuing What You Know In Your Heart You Were Meant To Do.-
Leroy Satchel Paige
Page 91 of 180
Civil Law Q&As (2007-2013) hectorchristopher@yahoo.com dbaratbateladot@gmail.com

compensation allowed by law. Is the bank Extinguishment; Compensation (2008)


correct? Explain. (4%)
No. XV. Eduardo was granted a loan by XYZ

SUGGESTED ANSWER: Bank for the purpose of improving a

No, the bank is not correct. While the building which XYZ leased from him.

Bank is correct about the applicability of Eduardo, executed the promissory note

compensation, it was not correct as to ("PN") in favor of the bank, with his friend

Recardo as co-signatory. In the PN, they


the amount compensated.
both acknowledged that they are

A bank deposit is a contract of loan, "individually and collectively" liable and

where the depositor is the creditor and waived the need for prior demand. To

secure the PN, Recardo executed a real


the bank the debtor. Since Sarah is also
estate mortgage on his own property. When
the debtor of the bank with respect to
Eduardo defaulted on the PN, XYZ stopped
the loan, both are mutually principal
payment of rentals on the building on the
debtors and creditors of each other. Both
ground that legal compensation had set in.
obligation are due, demandable and
Since there was still a balance due on the
liquidated but only up to the extent of PN after applying the rentals, XYZ
P300,000.00 (covering the unpaid third, foreclosed the real estate mortgage over
fourth and fifth monthly installments). Recardo's property. Recardo opposed the

The entire one million was not yet due foreclosure on the ground that he is only a

because the loan has no acceleration co-signatory; that no demand was made

clause in case of default. And since there upon him for payment, and assuming he is

liable, his liability should not go beyond


is no retention or controversy
half the balance of the loan. Further,
commenced by third person and
Recardo said that when the bank invoked
communicated in due time to the debtor,
compensation between the reantals and the
then all the requisites of legal
amount of the loan, it amounted to a new
compensation are present but only up to
contract or novation, and had the effect of
the amount of P300,000.00. The bank, extinguishing the security since he did not
therefore, may deduct P300,000.00 from give his consent (as owner of the property
Sarahs bank deposit by way of under the real estate mortgage) thereto.

compensation.
(A). Can XYZ Bank validly assert legal
compensation? (2%)

Never Let The Odds Keep You From Pursuing What You Know In Your Heart You Were Meant To Do.-
Leroy Satchel Paige
Page 92 of 180
Civil Law Q&As (2007-2013) hectorchristopher@yahoo.com dbaratbateladot@gmail.com

estate mortgage on his own property. When


Eduardo defaulted on the PN, XYZ stopped
SUGGESTED ANSWER:
payment of rentals on the building on the

Yes, XYZ Bank can validly assert legal

compensation. In the present case, all of

the elements of legal compensation are

present: (1) XYZ Bank is the creditor of

Eduardo while Eduardo is the lessor of

XYZ Bank; (2) both debts consist in a

sum of money, or if the things due are

consumable, they be of the same kind,

and also of the same quality if the latter

has been stated; (3) the two debts be

due; (4) they be liquidated and

demandable, and (5) over neither of them

there be any retention or controversy,

commenced by third persons and

communicated in due time to the debtor

(Art. 1279, Civil Code).

Extinguishment; Novation (2008)

No. XV. Eduardo was granted a loan by XYZ


Bank for the purpose of improving a

building which XYZ leased from him.


Eduardo, executed the promissory note

("PN") in favor of the bank, with his friend


Recardo as co-signatory. In the PN, they

both acknowledged that they are


"individually and collectively" liable and

waived the need for prior demand. To


secure the PN, Recardo executed a real
(C). Does Recardo have basis under the
ground that legal compensation had set in.
Civil Code for claiming that the original
Since there was still a balance due on the
contract was novated? (2%)
PN after applying the rentals, XYZ

foreclosed the real estate mortgage over SUGGESTED ANSWER:


Recardo's property. Recardo opposed the

foreclosure on the ground that he is only a No. Recardo has no basis for claiming

co-signatory; that no demand was made novation of the original contract when

upon him for payment, and assuming he is the bank invoked compensation because
liable, his liability should not go beyond there was simply partial compensation
half the balance of the loan. Further,
(Art. 1290, Civil Code) and this would
Recardo said that when the bank invoked
not bar the bank from recovering the
compensation between the reantals and the
remaining balance of the obligation.
amount of the loan, it amounted to a new

contract or novation, and had the effect of ALTERNATIVE ANSWER:


extinguishing the security since he did not

give his consent (as owner of the property No. In order that an obligation may be

under the real estate mortgage) thereto. extinguished by another, it is imperative

Never Let The Odds Keep You From Pursuing What You Know In Your Heart You Were Meant To Do.-
Leroy Satchel Paige
Page 93 of 180
Civil Law Q&As (2007-2013) hectorchristopher@yahoo.com dbaratbateladot@gmail.com

debt is not a valid tender of payment and

that it be so declared in unequivocal may be refused receipt by the oblige

terms, or that the old and new

obligations be on every point compatible

with each other. Novation is never

presumed (Art. 1292, Civil Code).

Extinguishment; Payment of Check

(2013)

No.VI. Lito obtained a loan of P1,000,000


from Ferdie, payable within one year. To

secure payment, Lito executed a chattel


mortgage on a Toyota Avanza and a real

estate mortgage on a 200-square meter


piece of property.

(B) Lito's failure to pay led to the extra-


judicial foreclosure of the mortgaged real

property. Within a year from foreclosure,


Lito tendered a manager's check to Ferdie to

redeem the property. Ferdie refused to


accept payment on the ground that he

wanted payment in cash: the check does


not qualify as legal tender and does not

include the interest payment. Is Ferdie's


refusal justified? (4%)

SUGGESTED ANSWER:

A check, whether a managers check or

an ordinary check is not legal tender,

and an offer of a check in payment of a


is sufficient to compel redemption but is

or creditors (Philippine Airlines v. CA not in itself a payment that relieves the

and Amelia Tan, G.R. No. L-49188, redemptioner from his liability to pay

1990). Mere delivery of checks does not the redemption price (Biana v. Gimenez,

discharge the obligation under a G.R. No. 132768, Sept 9, 2005, citing

judgment. A check shall produce the Fortunado v. CA).

effect of payment only when they have


Redemption within the period allowed by
been cashed or where through the fault
law is not a matter of intent but a
of the creditor they have been impaired
question of payment or valid tender of
(Art 1249, Civil Code).
full redemption prices within the said

However, it is not necessary that the period. Whether redemption is being

right of redemption be exercised by made under Art. 3135 or under the

delivery of legal tender. A check may be General Banking Law, the mortgagor or

used for the exercise of right of his assignee is required to tender

redemption, the same being a right and payment to make said redemption valid

not an obligation. The tender of a check (Heirs of Quisumbing v. PNB and SLDC,

G.R. No. 178242, Jan 20, 2009).

Never Let The Odds Keep You From Pursuing What You Know In Your Heart You Were Meant To Do.-
Leroy Satchel Paige
Page 94 of 180
Civil Law Q&As (2007-2013) hectorchristopher@yahoo.com dbaratbateladot@gmail.com

had become stale. Gustavo now wants


Felipe to pay him in cash the amount of
Moreover, Ferdies refusal was justified
P5,600. Claiming that the previous
on the ground that the amount tendered
payment was not in legal tender, and that
does not include interest. In order to

effect the redemption of the foreclosed

property, the payment to the purchaser

must include the following sums: (a) the

bid price; (b) the interest on the bid

price, computed at one per centum (1%)

per month; and (c) the assessments and

taxes, if any, paid by the purchaser with

the same rate of interest (Sec 28, 1997

Rules of Civil Procedure). Unless there is

an express stipulation to that effect, the

creditor cannot be compelled to receive

partial payment of the prestation (Art.

1248, Civil Code).

Extinguishment; Payment of Check;

Legal Tender (2008)

No. XVII. Felipe borrowed $100 from


Gustavo in 1998, when the Phil P - US$

exchange rate was P56 - US$1. On March


1, 2008, Felipe tendered to Gustavo a

cashier's check in the amount of P4,135 in


payment of his US$ 100 debt, based on the

Phil P - US$ exchange rat at that time.


Gustavo accepted the check, but forgot to

deposit it until Sept. 12, 2008. His bank


refused to accepted the check because it
check became stale. He is now estopped

there has been extraordinary deflation from raising the issue that a cashier's

since 1998, and therefore, Felipe should check is not legal tender.

pay him the value of the debt at the time it


(B). Can Felipe validly refuse to pay Gustavo
was incurred. Felipe refused to pay him
again? (2%)
again, claiming that Gustavo is estopped

from raising the issue of legal tender,


SUGGESTED ANSWER:
having accepted the check in March, and

that it was Gustavo's negligence in not Yes, Felipe can refuse to pay Gustavo,
depositing the check immediately that who allowed the check to become stale.
caused the check to become stale.
Although a check is not legal tender

(Belisario v. Natividad. 60 Phil 156),


(A). Can Gustavo now raised the issue that
the cashier's check is not legal tender? (2%) there are instances when a check

produces the effects of payment, for


SUGGESTED ANSWER: example: (a) when the creditor is in

estoppel or he had previously promised


No. Gustavo previously accepted a check
he would accept a check (Paras, Civil
as payment. It was his fault why the

Never Let The Odds Keep You From Pursuing What You Know In Your Heart You Were Meant To Do.-
Leroy Satchel Paige
Page 95 of 180
Civil Law Q&As (2007-2013) hectorchristopher@yahoo.com dbaratbateladot@gmail.com

both acknowledged that they are


"individually and collectively" liable and
Code Annotated, Vol IV, 2000 ed., p.
waived the need for prior demand. To
394); (b) when the check has lost its

value because of the fault of the creditor

(Art. 1249, 2nd par.),as when he was

unreasonably delayed in presenting the

check for payment (PNB v. Seeto, G.R.

No, L-4388, 13 August 1952).

(C). Can Felipe compel Gustavo to receive


US$100 instead? (1%)

SUGGESTED ANSWER:

Felipe cannot compel Gustavo to receive

US$100 because under RA 529, payment

of loans should be at Philippine currency

at the rate of exchange prevailing at the

time of the stipulated date of payment.

Felipe could only compel Gustavo to

receive US$ 100 if they stipulated that

obligation be paid in foreign currency

(R.A. 4100).

Liability; Solidary Liability (2008)

No. XV. Eduardo was granted a loan by XYZ


Bank for the purpose of improving a

building which XYZ leased from him.


Eduardo, executed the promissory note

("PN") in favor of the bank, with his friend


Recardo as co-signatory. In the PN, they
contract or novation, and had the effect of
extinguishing the security since he did not
secure the PN, Recardo executed a real
give his consent (as owner of the property
estate mortgage on his own property. When
under the real estate mortgage) thereto.
Eduardo defaulted on the PN, XYZ stopped

payment of rentals on the building on the


(B). Can Recardo's property be foreclosed to
ground that legal compensation had set in.
pay the full balance of the loan? (2%)
Since there was still a balance due on the
PN after applying the rentals, XYZ SUGGESTED ANSWER:
foreclosed the real estate mortgage over
Recardo's property. Recardo opposed the Yes, Recardo's property can be foreclosed

foreclosure on the ground that he is only a to pay the full balance of the loan

co-signatory; that no demand was made because when he signed as co-signatory


upon him for payment, and assuming he is in the promissory note, he acknowledged
liable, his liability should not go beyond he is solidarily liable with Eduardo. In
half the balance of the loan. Further,
solidary obligations, a creditor has the
Recardo said that when the bank invoked
right to demand full payment of the
compensation between the reantals and the
obligation from any of
amount of the loan, it amounted to a new

Never Let The Odds Keep You From Pursuing What You Know In Your Heart You Were Meant To Do.-
Leroy Satchel Paige
Page 96 of 180
Civil Law Q&As (2007-2013) hectorchristopher@yahoo.com dbaratbateladot@gmail.com

takes charge of the agency or

the solidary debtors (Art. 1207, Civil management of the business or property

Code). of another without any power from the

Obligations; Without Agreement (2007)

No.V. What are obligations without an

agreement"? Give five examples of


situations giving rise to this type of

obligations? (10%)

SUGGESTED ANSWER:

"Obligations without an agreement" are

obligations that do not arise from

contract such as those arising from: 1.

delicts; 2. quasi-delicts; 3. solutio

indebiti; 4. negotiorum gestio; and 5. all

other obligations arising from law.

ALTERNATIVE ANSWER:

"Obligations without an agreement" refer

to the juridical relation of quasi-contract

which arise from certain lawful,

voluntary and unilateral acts to the end

that no one shall be unjustly enriched or

benefited at the expense of another. (Art.

2142, NCC)

First Example of an obligation without

an agreement is a case of negotiorum

gestio, whereby one who voluntarily


support, it is given by a stranger, the

latter, is obliged to continue the same latter shall have a right to claim the

until the termination of the affair and its same from the former, unless it appears

incidents, or to require the person that he gave it out of piety and without

concerned to substitute him, if the intention of being repaid (Art. 2164,

owner is in a position to do so (Art. NCC).

2144, NCC).
Fourth example, is when through

Second example, a case of solutio accident or other causes a person is

indebiti may also give rise to an injured or becomes seriously ill, and he

obligation without an agreement. This is treated or helped while he is not in a

refers to the obligation to return which condition to give consent to a contract,

arises when something is received when he shall be liable to pay for the services

there is no right to demand it, and it was of the physician or other person aiding

unduly delivered through mistake (Art. him, unless the service has been

2154, NCC). rendered out of pure generosity (Art.

2167, NCC).
Third example, is when without the

knowledge of the person obliged to give

Never Let The Odds Keep You From Pursuing What You Know In Your Heart You Were Meant To Do.-
Leroy Satchel Paige
Page 97 of 180
Civil Law Q&As (2007-2013) hectorchristopher@yahoo.com dbaratbateladot@gmail.com

a trust against one who, by fraud, duress

Fifth instance of an obligation without or abuse of confidence, obtains or holds

an agreement is when the person obliged the legal right to property which he

to support an orphan or an insane or

other indigent person unjustly refuses to

give support to the latter, any third

person may furnish support to the needy

individual, with right of reimbursement

from the person obliged to give support.

The provisions of this article apply when

the father or mother of a child under

eighteen years of age unjustly refuses to

support him (Art. 2166, NCC).

Trust

Trust De Son Tort (2007)

No.III. Explain the following concepts and


doctrines and give an example of each:

(A). concept of trust de son


tort (constructive trust) (5%)

SUGGESTED ANSWER:

A constructive trust is a trust NOT

created by any word or phrase, either

expressly or impliedly, evincing a direct

intention to create a trust, but is one

that arises in order to satisfy the

demands of justice. It does not come

about by agreement or intention but

mainly operation of law and construed as


provides: "If an absolute conveyance of

ought not, in equity and good property is made in order to secure the

conscience, to hold (Heirs of Lorenzo Yap performance of an obligation of the

v. CA, 371 Phil 523, 1991). The following grantor toward the grantee, a trust by

are examples of constructive trust: 1. virtue of law is established. If the

Art. 1456 NCC which provides: "If fulfillment of the obligation is offered by

property is acquired through mistake or the grantor when it becomes due, he

fraud, the person obtaining it is, by force may demand the reconveyance of the

of law considered a trustee of an implied property to him." 4. Art 1455 NCC which

trust for the benefit of the person for provides: "When any trustee, guardian or

whom the property comes." 2. Art 1451 any person holding a fiduciary

NCC which provides: "When land passes relationship uses trust funds for the

by succession through any person and he purchase of property and causes

causes the legal title to be put in the conveyance to be made to him or to

name of another, a trust is established third person, a trust is established by

by implication of law for the benefit of operation of law in favor of the person to

the true owner." 3. Art 1454 NCC which whom the funds belong."

Never Let The Odds Keep You From Pursuing What You Know In Your Heart You Were Meant To Do.-
Leroy Satchel Paige
Page 98 of 180
Civil Law Q&As (2007-2013) hectorchristopher@yahoo.com dbaratbateladot@gmail.com
No.VI. (b) Eulalia was engaged in the
business of buying and selling large cattle.
Sales
In order to secure the financial capital, she

Condominium Act; Partition of a

Condominium (2009)

No.XVIII. The Ifugao Arms is a


condominium project in Baguio City. A

strong earthquake occurred which left huge


cracks in the outer walls of the building. As

a result, a number of condominium units


were rendered unfit for use. May Edwin,

owner of one of the condominium units


affected, legally sue for partition by sale of

the whole project? Explain. (4%)

SUGGESTED ANSWER:

Yes, Edwin may legally sue for partition

by sale of the whole condominium

project under the following conditions:

(a) the damage or destruction caused by

the earthquake has rendered one-half

(1/2) or more of the units therein

untenantable, and (b) that the

condominium owners holding an

aggregate of more than thirty percent

(30%) interests of the common areas are

opposed to the restoration of the

condominium project (Sec 8 [b], Republic

Act No. 472 Condominium Act).

Mortgage; Equitable Mortgage (2012)


mortgage for the purpose of securing the
shortage incurred by Domeng in the
advanced for her employees (biyaheros). She
amount of P 70, 000.00 while employed as
required them to surrender TCT of their
"biyahero" by Eulalia. Was the Deed of Sale
properties and to execute the corresponding
between Domeng and Eulalia a contract of
Deeds of Sale in her favor. Domeng
sale or an equitable mortgage? Explain.
Bandong was not required to post any
(5%)
security but when Eulalia discovered that
he incurred shortage in cattle procurement
SUGGESTED ANSWER:
operation, he was required to execute a
Deed of Sale over a parcel of land in favor of The contract between Domeng Bandong

Eulalia. She sold the property to her and Eulalia was an equitable mortgage
grandneice Jocelyn who thereafter rather than a contract of sale. The
instituted an action for ejectment against purported deed of sale was actually
the Spouses Bandong.
intended to merely secure the payment

of the shortage incurred by Domeng in


To assert their right, Spouses Bandong filed

an action for annulment of sale against the conduct of the cattle-buying

Eulalia and Jocelyn alleging that there was operations. Under Art 1602, Civil Code,

no sale intended but only equitable

Never Let The Odds Keep You From Pursuing What You Know In Your Heart You Were Meant To Do.-
Leroy Satchel Paige
Page 99 of 180
Civil Law Q&As (2007-2013) hectorchristopher@yahoo.com dbaratbateladot@gmail.com

On June 15, 2013, Sergio learned of


the contract shall be presumed to be an
another buyer, Roberto, who was
equitable mortgage when it may be fairly

inferred that the real intention of the

parties is simply to secure the payment

of a debt or the performance of any

other obligation. The present transaction

was clearly intended to just secure the

shortage incurred by Eulalia because

Bandung remained in possession of the

property inspite of the execution of the

sale.

Option Contract; Liquor & Pulutan as

Consideration (2013)

No.III.Sergio is the registered owner of a


500-square meter land. His friend, Marcelo,

who has long been interested in the


property, succeeded in persuading Sergio to

sell it to him. On June 2, 2012, they agreed


on the purchase price of P600,000 and that

Sergio would give Marcelo up to June30,


2012 within which to raise the amount.

Marcelo, in a light tone usual between


them, said that they should seal their

agreement through a case of Jack Daniels


Black and P5,000 "pulutan" money which

he immediately handed to Sergio and which


the latter accepted. The friends then sat

down and drank the first bottle from the


case of bourbon.
Yes. Marcelo has a cause of action
offering P800,000 in ready cash for the
against Sergio.
land. When Roberto confirmed that he
could pay in cash as soon as Sergio could Under Art. 1324, when the offerer has
get the documentation ready, Sergio allowed the offeree a certain period to
decided to withdraw his offer to Marcelo,
accept, the offer may be withdrawn at
hoping to just explain matters to his friend.
any time before acceptance by
Marcelo, however, objected when the
communicating such withdrawal, except
withdrawal was communicated to him,
when the option is founded upon
taking the position that they have a firm
consideration, as something paid or
and binding agreement that Sergio cannot
promised.
simply walk away from because he has an

option to buy that is duly supported by a


An accepted unilateral promise to buy or
duly accepted valuable consideration.
sell a determinate thing for a price

(A) Does Marcelo have a cause of action certain is binding upon him if the
against Sergio? (5%) promise is supported by a consideration

distinct from the price (Art. 1479).


SUGGESTED ANSWER:

Never Let The Odds Keep You From Pursuing What You Know In Your Heart You Were Meant To Do.-
Leroy Satchel Paige
Page 100 of 180
Civil Law Q&As (2007-2013) hectorchristopher@yahoo.com dbaratbateladot@gmail.com

of sale for the real property. Therefore,

Consideration in an option contract may Art. 1403 does not apply.

be anything of value,, unlike in sale

where it must be the price certain in

money or its equivalent (San Miguel

Properties Inc. v. Spouses Huang, G.R.

No. 137290, July 31, 2000).

Here, the case of Jack Daniels Black and

the P5,000.00 pulutan money was a

consideration to seal their agreement,

an agreement that Marcelo is given until

June 30, 2012 to buy the parcel of land.

There is also no showing that such

consideration will be considered part of

the purchase price. Thus, Sergios

unilateral withdrawal of the offer

violated the Option Contract between

him and Marcelo.

(B) Can Sergio claim that whatever they

might have agreed upon cannot be enforced


because any agreement relating to the sale

of real property must be supported by


evidence in writing and they never reduced

their agreement to writing? (3%)

SUGGESTED ANSWER:

No. Sergios claim has no legal basis.

The contract at issue in the present case

is the option contract, not the contract


ALTERNATIVE ANSWER:

The Statute of Frauds covers an


No. Sergios claim has no legal basis.
agreement for the sale of real property

or of an interest therein. Such The contract of sale has already been

agreement is unenforceable by action, partially executed which takes it outside

unless the same, or some note or the ambit of the Statute of Frauds is

memorandum, thereof, be in writing, applicable only to executory contracts,

(Art. 1403 (e), Civil Code). Here, Marcelo not to contracts that are totally or

and Sergio merely entered into an partially performed (Carbonnel v. Poncio,

Option Contract, which refers to a G.R. No. L-11231, May 12, 1958).

unilateral promise to buy or sell, which

need not be in writing to be enforceable

(Sanchez v. Rigos, G.R. No. L-25494, Right of First Refusal; Lessee; Effect
June 14, 1972, citing Atkins, Kroll and (2008)

Co. Inc. v. Cua Hian Tek and


No.XVI. Dux leased his house to Iris for a
Southwestern Sugar & Molasses Co. v.
period of 2 years, at the rate of P25,000.00
Atlantic Gulf & Pacific Co.).

Never Let The Odds Keep You From Pursuing What You Know In Your Heart You Were Meant To Do.-
Leroy Satchel Paige
Page 101 of 180
Civil Law Q&As (2007-2013) hectorchristopher@yahoo.com dbaratbateladot@gmail.com

that the mother is not a third party, this

monthly, payable annually in advance. The would make her privy to the agreement

contract stipulated that it may be renewed of Dux and Iris, aware of the right of first

for another 2-year period upon mutual

agreement of the parties. The contract also


granted Iris the right of first refusal to

purchase the property at any time during


the lease, if Dux decides to sell the property

at the same price that the property is


offered for sale to a third party. Twenty-

three months after execution of the lease


contract, Dux sold breach of her right of

first refusal. Dux said there was no breach


because the property was sold to his

mother who is not a third party. Iris filed an


action to rescind the sale and to compel

Dux to sell the property to her at the same


price. Alternatively, she asked the court to

extend the lease for another 2 years on the


same terms.

(A). Can Iris seek rescission of the sale of


the property to Dux's mother? (3%)

SUGGESTED ANSWER:

Yes, because the right of first refusal is

included in the contract signed by the

parties. Only if the lessee failed to

exercise the right of first refusal could

the lessor lawfully sell the subject

property to others, under no less than

the same terms and conditions

previously offered to the lessee. Granting


No. The contract stipulated that it may
refusal. This makes the mother a buyer
be renewed for another 2-year period
in bad faith, hence giving more ground
upon mutual agreement of the parties.
for rescission of the sale to her
Contracts are binding between the
(Equatorial Realty, et al. v. Mayfair
parties; validity or compliance cannot be
Theater, G.R. No. 106063, 21 Nov. 1996).
left to the will of one of the parties (Art.

ALTERNATIVE ANSWER: 1308, Civil Code).

No, Iris cannot seek rescission of the ALTERNATIVE ANSWER:


sale of the property to Duxs mother
It depends. The alternative prayer for
because the sale is not one of those
the extension of the lease may prosper if
rescissible contracts under Art. 1381 of
(a) there is a stipulation in the contract
the Civil Code.
of sale; (b) Dux's mother is aware of the
(B). Will the alternative prayer for extension existing contract of lease; or (c) the lease
of the lease prosper? (2%) is recorded in the Registry of Property

(Art. 1676, Civil Code).


SUGGESTED ANSWER:

Never Let The Odds Keep You From Pursuing What You Know In Your Heart You Were Meant To Do.-
Leroy Satchel Paige
Page 102 of 180
Civil Law Q&As (2007-2013) hectorchristopher@yahoo.com dbaratbateladot@gmail.com

(IVa) Resolve Boboy's claim that as a builder


in good faith, he should be
Lease

Builder; Good Faith; Useful

Improvements (2013)

No.IV.Anselmo is the registered owner of a


land and a house that his friend Boboy

occupied for a nominal rental and on the


condition that Boboy would vacate the

property on demand. With Anselmo's


knowledge, Boboy introduced renovations

consisting of an additional bedroom, a


covered veranda, and a concrete block

fence, at his own expense.

Subsequently, Anselmo needed the property


as his residence and thus asked Boboy to

vacate and turn it over to him. Boboy,


despite an extension, failed to vacate the

property, forcing Anselmo to send him a


written demand to vacate.

In his own written reply, Boboy signified


that he was ready to leave but Anselmo

must first reimburse him the value of the


improvements he introduced on the

property as he is a builder in good faith.


Anselmo refused, insisting that Boboy

cannot ask for reimbursement as he is a


mere lessee. Boboy responded by removing

the improvements and leaving the building


in its original state.
As Boboy is a lessee of the property,

reimbursed the value of the improvements even if he was paying nominal rental,

he introduced. (4%) Art. 1678, Civil Code, is applicable.

Under this provision, if the lessee makes,


SUGGESTED ANSWER:
in good faith, useful improvements

which are suitable to the use for which


Boboys claim that he is a builder in
the lease is intended, without altering
good faith has no basis. A builder in good
the form or substance of the property
faith is someone who occupies the
leased, the lessor upon the termination
property in concept of an owner. The
of the lease, shall pay the lessee one-half
provisions on builder-planter-sower
of the value of improvements at that
under the Civil Code cover cases in
time. Should the lessor refuse to
which the builder, planter and sower
reimburse said amount, the lessee may
believe themselves to be owners of the
remove the improvements, even though
land, or at least, to have a claim of title
the principal thing may suffer damage
thereto.
thereby.

Never Let The Odds Keep You From Pursuing What You Know In Your Heart You Were Meant To Do.-
Leroy Satchel Paige
Page 103 of 180
Civil Law Q&As (2007-2013) hectorchristopher@yahoo.com dbaratbateladot@gmail.com

when the term of their lease shall have


expired. Is Ildefonso bound to respect the
(IVb) Can Boboy be held liable for damages
for removing the improvements over
Anselmo's objection? (4%)

SUGGESTED ANSWER:

No. Boboy cannot be held liable for

damages.

The lessor, Anselmo, refused to

reimburse one-half of the value of the

improvements, so the lessee, Boboy, may

remove the same, even though the

principal thing may suffer damage

thereby. If in removing the useful

improvements Boboy caused more

impairment in the property leased than

is necessary he will be liable for damages

(Art. 1678, Civil Code).

Lease; Caveat Emptor (2009)

No.VIII. Jude owned a building which he


had leased to several tenants. Without

informing his tenants, Jude sold the


building to Ildefonso. Thereafter, the latter

notified all the tenants that he is the new


owner of the building. Ildefonso ordered the

tenants to vacate the premises within thirty


(30) days from notice because he had other

plans for the building. The tenants refused


to vacate, insisting that they will only do so
status of the occupants of their right to

lease contracts between Jude and his occupy the building before buying it.

tenants? Explain your answer. (3%)

SUGGESTED ANSWER:
Agency
Yes, Ildefonso must respect the lease
Agency; Sale of a Real Property through
contracts between Jude and his tenants.
an Agent (2010)
While it is true that the said lease

contracts were not registered and No.XVI. X was the owner of an unregistered
annotated on the title to the property, parcel of land in Cabanatuan City. As she

Ildefonso is still not an innocent was abroad, she advised her sister Y via

purchaser for value. He ought to know overseas call to sell the land and sign a

the existence of the lease because the contract of sale on her behalf.

building was already occupied by the


Y thus sold the land to B1 on March 31,
tenants at the time he bought it.
2001 and executed a deed of absolute sale
Applying the principle of caveat emptor, on behalf of X. B1 fully paid the purchase
he should have checked and known the price.

Never Let The Odds Keep You From Pursuing What You Know In Your Heart You Were Meant To Do.-
Leroy Satchel Paige
Page 104 of 180
Civil Law Q&As (2007-2013) hectorchristopher@yahoo.com dbaratbateladot@gmail.com

inscribed in the Registry of Deeds, the

B2, unaware of the sale of the land to B1, case is governed by Art 1544 of the New

signified to Y his interest to buy it but Civil Code which provides that in case of

asked Y for her authority from X. Without

informing X that she had sold the land to


B1, Y sought X for a written authority to

sell.

X e-mailed Y an authority to sell the land. Y

thereafter sold the land on May 1, 2001 to


B2 on monthly installment basis for two
years, the first installment to be paid at the
end of May 2001.

Who between B1 and B2 has a better right


over the land? Explain. (5%)

SUGGESTED ANSWER:

B-2 has a better title. This is not a case

of double sale. Since the first sale was

void. The law provides that when a sale

of a piece of land or any interest therein

is through an agent, the authority of the

latter shall be in writing; otherwise, the

sale shall be void (Art 1874, NCC). The

property was sold by Y to B1 wihtout any

written authority from the owner X.

Hence, the sale to B1 was void.

ALTERNATIVE ANSWER:

Under the facts, B-1 has a better right to

the land. Given the fact that the Deed of

Sale in favor of B-1 and B-2 are not


extinguishing right to real property not

double sales of an immovable property, registered under Act 496 shall be valid

the ownership shall pertain to the except as between the parties. Thus, the

person who is in good faith was first in Deed of Sale of B-2 has no binding effect

possession and in the absence thereof to on B-1.

the person who presents the oldest title,


Partnership
provide there is good faith.

Liability; Liability of a Partner (2010)


In a case, the Supreme Court has held

that in a sale of real estate the execution


No.XV. A, B, and C entered into a
of a notarial document of sale is partnership to operate a restaurant
tantamount to delivery of the possession business. When the restaurant had gone
of the property sold. The ownership of past break-even stage and started to garner

the land therefore pertains to the first considerable profits, C died. A and B

buyer. It may also be mentioned that continued the business without dissolving

under Art 3344 no instruments or deed the partnership. They in fact opened a
branch of the restaurant, incurring
establishing, transmitting,
obligations in the process. Creditors started
acknowledging, modifying, or

Never Let The Odds Keep You From Pursuing What You Know In Your Heart You Were Meant To Do.-
Leroy Satchel Paige
Page 105 of 180
Civil Law Q&As (2007-2013) hectorchristopher@yahoo.com dbaratbateladot@gmail.com

1829, 1835, par 2, NCC; Testate Estate

demanding for the payment of their of Mota v. Serra, 47 Phil

obligations. 464 [1925]). However, the liability of Cs

(A). Who are liable for the settlement of the


partnerships obligations? Explain? (3%)

SUGGESTED ANSWER:

The two remaining partners, A and B, are

liable. When any partner dies and the

business is continued without any

settlement of accounts as between him

or his estate, the surviving partners are

held liable for continuing the business

despite the death of C (Art 1841, 1785,

par 2, and Art 1833 of NCC).

(B).What are the creditors recourse/s?


Explain. (3%)

SUGGESTED ANSWER:

Creditors can file the appropriate

actions, for instance, an action for

collection of sum of money against the

partnership at will and if there are no

sufficient funds, the creditors may go

after the private properties of A and B

(Art 816, NCC). Creditors may also sue

the estate of C. The estate is not excused

from the liabilities of the partnership

even if C is dead already but only up to

the time that he remained a partner (Art


ALTERNATIVE ANSWER:
individual properties shall be subject
TRUE. An oral is a consensual of the
first to the payment of his separate
partnership is valid even though not in
debts (Art 1835. NCC).
writing. However, If it involves

contribution of an immovable property

or a real right, an oral contract of


Oral Partnership (2009)
partnership is void. In such a case, the

No.I. TRUE or FALSE. Answer TRUE if the contract of partnership to be valid, must

statement is true, or FALSE if the be in a public instrument ( Art. 1771

statement is false. Explain your answer in ,NCC ), and the inventory of said
not more than two (2) sentences. property signed by the parties must be

attached to said public instrument (Art.


(C). An oral partnership is valid. (1%)
1773, NCC).

SUGGESTED ANSWER:

TRUE. Partnership is a consensual ALTERNATIVE ANSWER:

contract, hence, it is valid even though

not in writing.

Never Let The Odds Keep You From Pursuing What You Know In Your Heart You Were Meant To Do.-
Leroy Satchel Paige
Page 106 of 180
Civil Law Q&As (2007-2013) hectorchristopher@yahoo.com dbaratbateladot@gmail.com

the existence of a partnership. Do you


agree? Explain your answer. (5%)
TRUE. Partnership is a consensual

contract, hence, it is valid even though


SUGGESTED ANSWER:
not in writing. The oral contract of

partnership is also valid even if an

immovable property or real right is

contributed thereto. While the law, in

such a case, requires the partnership to

be in a public document, the law does

not expressly declare the contract void if

not executed in the required form

(Article 1409 (7 ,NCC ). And there being

nothing in the law from which it can be

inferred that the said requirement is

prohibitory or mandatory (Article 5,

NCC), the said oral contract of

partnership must also be valid. The

interested party may simply require the

contract to be made into a public

document in order to comply with the

required form (Article 1357, NCC). The

purpose of the law in requiring a public

document is simply to notify the public

about the contribution.

Share; Demand during the Existence of

Partnership (2012)

No.X.b) A partner cannot demand the


return of his share (contribution) during
Such agreement was not reduced to writing.
Siga-an demanded interest which was paid
Yes, he is not entitled to the return of
by Villanueva in cash and checks. The total
his contribution to the capital of the
amount Villanueva paid accumulated to P
partnership, but only to the net profits
1, 200, 000.00. Upon advice of her lawyer,
from the partnership business during the
Villanueva demanded for the return of the
life of the partnership period. If he is a
excess amount of P 660, 000.00 which was
limited partner, however, he may ask for ignored by Siga-an.
the return of his contributions as
(1) Is the payment of interest valid? Explain.
provided in Art 1856 and 1857, Civil
(3%)
Code.

SUGGESTED ANSWER:
Commodatum & Mutuum

Mutuum; Interest; Solutio Indebiti No, Art. 1956, Civil Code, provides that

no interest shall be due unless it has


(2012)
been expressly stipulated in writing.
No.VI.a) Siga-an granted a loan to
Villanueva in the amount of P 540, 000.00. (2) Is solution indebiti applicable? Explain.
(2%)

Never Let The Odds Keep You From Pursuing What You Know In Your Heart You Were Meant To Do.-
Leroy Satchel Paige
Page 107 of 180
Civil Law Q&As (2007-2013) hectorchristopher@yahoo.com dbaratbateladot@gmail.com

promise answer for the debt, or

SUGGESTED ANSWER: miscarriage of another, the Statute of

Frauds requires it to be in writing to be


Yes, Solutio Indebiti is applicable

because Villanueva Overpaid by

P600,000.00 representing interest

payment which is not due. He can,

therefore, demand its return.

Guaranty

Guaranty (2009)

No.I. TRUE or FALSE. Answer TRUE if the

statement is true, or FALSE if the

statement is false. Explain your answer in


not more than two (2) sentences.

(D). An oral promise of guaranty is valid


and binding. (1%)

SUGGESTED ANSWER :

FALSE. An oral contract of guaranty,

being a special promise to answer for the

debt of

another, is unenforceable unless in

writing (Article 1403 [2] b, NCC ).

ALTERNATIVE ANSWER:

TRUE. An oral promise of guaranty is

valid and binding. While the contract is

valid, however ,it is unenforceable

because it is not writing . Being a special


(1)The obligation in guaranty is

enforceable ( Article 1403 [2] b, NCC).The secondary; whereas, in suretyship, it is

validity of the contract should be primary.

distinguished from its enforceability .


(2) In guranty, the undertaking is to pay

Surety if the principal debtor cannot pay;

whereas, in suretyship, the undertaking


Surety (2010)
is to pay if the principal debtor does not

pay .
No.III. Define, Enumerate or Explain. (2%

each) (3) In guranty, the guarantor is entitled

to the benefit of excussion; whereas, in


(A). What is the difference between
suretyship the surety is not entitled.
"guaranty" and "suretyship"?

(4) Liability in guaranty depends upon


SUGGESTED ANSWER:
an independent agreement to pay the

Guaranty and Suretyship distinguished obligations of the principal if he fails to

do so; whereas, in suretyship, the surety

assumes liability as a regular party.

Never Let The Odds Keep You From Pursuing What You Know In Your Heart You Were Meant To Do.-
Leroy Satchel Paige
Page 108 of 180
Civil Law Q&As (2007-2013) hectorchristopher@yahoo.com dbaratbateladot@gmail.com

by the pledgee of the thing pledged in

(5)The Guarantor insures the solvency case of default by the pledgor.

of the principal debtor; whereas, the

surety insures the debt.

(6)In a guaranty, the guarantor is

subsidiarlty liable; whereas, in a

suretyship, the surety binds himself

solidarity with the principal debtor (Art

2047, Civil Code).

Pledge

Pledge; Pactum Commissorium (2009)

No.XVII. Rosario obtained a loan of

P100,000.00 from Jennifer, and pledged her


diamond ring. The contract signed by the

parties stipulated that if Rosario is unable


to redeem the ring on due date, she will

execute a document in favor of Jennifer


providing that the ring shall automatically

be considered full payment of the loan.

(A). Is the contract valid? Explain. (3%)

SUGGESTED ANSWER:

The contract is valid because Rosario

has to execute a document in favor of

Jennifer to transfer the ownership of the

pledged ring to the latter. The contract

does not amount to pactum

commissorium because it does not

provide for the automatic appropriation


them xxx. Jennifer cannot immediately

(B). Will your answer to [a] be the same if sell by herself the thing pledged. It must

the contract stipulates that upon failure of be foreclosed by selling it at a public

Rosario to redeem the ring on due date, auction in accordance with the

Jennifer may immediately sell the ring and procedure under Art 2112 of the New
appropriate the entire proceeds thereof for Civil Code.
herself as full payment of the loan?
Torts and Damages
Reasons. (3%)

Damages (2012)
SUGGESTED ANSWER:

No, my answer will be different. While


No.I. a) Roberto was in Nikko Hotel when he
the contract of pledge is valid, the
bumped into a friend who was then on her
stipulation authorizing the pledgee to way to a wedding reception being held in
immediately sell the thing pledged is said hotel. Roberto alleged that he was then
void under Art 2088 of the New Civil invited by his friend to join her at the

Code, which provides that the creditor wedding reception and carried the basket

cannot appropriate the things given by full of fruits which she was bringing to the
affair. At the reception, the wedding
way of pledge or mortgage, or dispose of

Never Let The Odds Keep You From Pursuing What You Know In Your Heart You Were Meant To Do.-
Leroy Satchel Paige
Page 109 of 180
Civil Law Q&As (2007-2013) hectorchristopher@yahoo.com dbaratbateladot@gmail.com

that exposed Roberto to unnecessary

coordinator of the hotel noticed him and ridicule or shame, his action will

asked him, allegedly in a loud voice, to leave


as he was not in the guest list. He retorted

that he had been invited to the affair by his


friend, who however denied doing so.

Deeply embarrassed by the incident,


Roberto then sued the hotel for damages

under Articles 19 and 21 of the

Civil Code. Will Robertos action prosper?


Explain. (5%)

SUGGESTED ANSWER:

No. Robertos action will not prosper.

From the facts given in the problem, the

wedding coordinator did not abuse her

right when she asked him to leave the

wedding reception because he was not in

the guest list. Hotel Nikko could not be

held liable for damages as its liable

spring from the liability of its employee

(Nikko Hotel Manila Garden v. Reyes,

G.R. No. 154259, Feb 28, 2005).

ALTERNATIVE ANSWER:

It depends. While the hotel has the right

to exclude an uninvited guest from the

wedding reception, that does not give

the hotel the license to humiliate

Roberto. If the wedding coordinator of

the hotel acted wrongfully e.g. with the

abuse of right, unfairly, or in a matter


finding. In the contrary, the problem

prosper. Otherwise, Robertos action will states that it is a mere allegation.

not prosper.

The hotel is liable for the wrongful acts


Damages; Moral & Exemplary (2009)
of its employees.

No.XIV. Rodolfo, married to Sharon, had an


COMMENT:
illicit affair with his secretary, Nanette, a

The facts of the problem are almost 19-year old girl, and begot a baby girl,

similar to the facts of Nikko Hotel Rona. Nanette sued Rodolfo for damages:

Manila Garden v. Reyes, G.R. No. actual, for hospital and other medical

expenses in delivering the child by


154259, Feb 28, 2005. In the said case,
caesarean section; moral, claiming that
however, there is a categorical finding
Rodolfo promised to marry her,
that the hotel employee did not, exposed
representing that he was single when, in
the complainant to the ridicule, shame
fact, he was not; and exemplary, to teach a
or embarrassment; hence, did not
lesson to like-minded Lotharios.
commit any abuse of right. The present

problem makes no statement of that

Never Let The Odds Keep You From Pursuing What You Know In Your Heart You Were Meant To Do.-
Leroy Satchel Paige
Page 110 of 180
Civil Law Q&As (2007-2013) hectorchristopher@yahoo.com dbaratbateladot@gmail.com

thereby imposed higher taxes), which the


Supreme Court later declared invalid.
(A). If you were the judge, would you award
all the claims of Nanette? Explain. (3%)

SUGGESTED ANSWER:

If Rodolfo's marriage could not have

been possibly known to Nanette or there

is no gross negligence on the part of

Nanette, Rodolfo could be held liable for

moral damages.

If there is gross negligence in a suit for

quasi-delict, exemplary could be

awarded.

Damages; Public Officers acting in the

Performance of their Duties (2012)

No.II.a) Liwayway Vinzons-Chato was then

the Commissioner of Internal Revenue


while Fortune Tobacco Corporation is an

entity engaged in the manufacture of


different brands of cigarettes, among which

are "Champion," "Hope," and "More"


cigarettes.

Fortune filed a complaint against Vinzons-

Chato to recover damages for the alleged


violation of its constitutional rights arising

from Vinzons-Chatos issuance of Revenue

Memorandum Circular No. 37-934 (which


re-classified Fortune cigarettes as locally

manufactured with foreign brands and


The public officer is not automatically

Vinzons-Chato filed a Motion to Dismiss considered to have violated the rights or

arguing that she cannot be held liable for liberties of a person simply because the

damages for acts she performed while in rule the public officer issued was
the discharge of her duties as BIR declared invalid by the court. The
Commissioner. Is she correct? Explain. (5%) complainant must still allege and prove

the particular injury or prejudice he has


SUGGESTED ANSWER:
suffered from the violation of his
Yes. As a general rule, a public officer is constitutional right by the issuance of
not liable for acts performed in the the invalidated rule.
discharge of his duties. The exceptions
The problem does not state any fact from
are when he acted with malice, bad faith,
which any malice, bad faith or gross
or gross negligence in the performance
negligence on the part of Vinzons-Chato
of his duty, or when his act is in
may be inferred, or the particular injury
violation of a Constitutional guaranteed
or prejudice the complainant may have
right and liberties of a person under
suffered as a result of the violation of his
Art32 of the NCC.
constitutional right. Hence, she

Never Let The Odds Keep You From Pursuing What You Know In Your Heart You Were Meant To Do.-
Leroy Satchel Paige
Page 111 of 180
Civil Law Q&As (2007-2013) hectorchristopher@yahoo.com dbaratbateladot@gmail.com

civil liability, Rommel is subsidiarily

cannot be held liable. The facts liable for the damage awarded against

presented are similar to facts of the case

of Vinzons-Chato v. Fortune, G.R. No.

141309, Dec 23, 2008.

Death Indemnity (2009)

No. X. Rommels private car, while being


driven by the regular family driver, Amado,

hits a pedestrian causing the latters death.


Rommel is not in the car when the incident

happened.

(A). Is Rommel liable for damages to the


heirs of the deceased? Explain. (2%)

SUGGESTED ANSWER:

Yes, Rommel may be held liable for

damages if he fails to prove that he

exercised the diligence of a good father

of a family (Art. 2180, par 5, NCC) in

selecting and supervising his family

driver. The owner is presumed liable

unless he proves the defense of

diligence. If the driver was performing

his assigned task when the accident

happened, Rommel shall be solidarily

liable with the driver.

In case the driver is convicted of

reckless imprudence and cannot pay the


Yes, my answer will be the same except

the driver and the defense of diligence is that in such case the liability of the

not available. owner is not presumed. When the owner

is inside the vehicle, he becomes liable


(B).Would your answer be the same if only when it is shown that he could have
Rommel was in the car at the time of the
prevented the misfortune by the use of
accident? Explain. (2%)
due diligence (Art. 2184, NCC). For the

owner to be held liable, the burden of


SUGGESTED ANSWER:

Yes, my answer would be the same. proving that he could have prevented

Rommel, who was in the car, shall be the misfortune rests on the shoulder of

liable for damages if he could have the victim.

prevented the misfortune by the use of

due diligence in supervising his driver

but failed to exercise it (Art. 2184, NCC). Doctrine of Discovered Peril (Last Clear

In such case, his liability is solidary with Chance) (2007)

his driver.
No.III. Explain the following concepts and
ALTERNATIVE ANSWER:
doctrines and give an example of each:

Never Let The Odds Keep You From Pursuing What You Know In Your Heart You Were Meant To Do.-
Leroy Satchel Paige
Page 112 of 180
Civil Law Q&As (2007-2013) hectorchristopher@yahoo.com dbaratbateladot@gmail.com

No.XIV. Primo owns a pet iguana which he


(B). doctrine of discovered peril (last clear keeps in a man-made pond enclosed by a
chance) (5%) fence situated in his residential lot. A

SUGGESTED ANSWER:

The doctrine of last clear chance states

that where the plaintiff was guilty of

prior or antecedent negligence, but the

defendant, who had the ultimate

opportunity to avoid the impending

harm failed to do so, it is the defendant

who is liable for all the consequences of

the accident notwithstanding the prior

negligence of the plaintiff. An example is

where a person was riding a pony on a

bridge and improperly pulled the pony to

the wrong side when he saw a car

coming. The driver of the car did not

stop or change direction, and nearly hit

the horse, and, the frightened animal

jumped to its death. The driver of the

car is guilty of negligence because he

had a fair opportunity to avoid the

accident and failed to avail himself of

that opportunity. He is liable under the

doctrine of last clear chance (Picart v.

Smith, 37 Phil. 809, 1918).

Liability; Owner of a Pet; Fortuitous

Event (2010)
in case the damage should come from

typhoon knocked down the fence of the force majeure or from the fault of the

pond and the iguana crawled out of the gate person who has suffered damage (Art

of Primos residence. N, a neighbor who was 2183, NCC).

passing by, started throwing stones at the


iguana, drawing the iguana to move toward

him. N panicked and ran but tripped on


Liability; Special Parental Authority
something and suffered a broken leg.
(2010)

Is anyone liable for Ns injuries? Explain.


No.XII. On May 5, 1989, 16-year old
(4%)
Rozanno, who was issued a student permit,

SUGGESTED ANSWER: drove to school a car, a gift from his

parents. On even date, as his class was


No one is liable. The possessor of an
scheduled to go on a field trip, his teacher
animal or whoever may make use of the
requested him to accommodate in his car,
same is responsible for the damage it
as he did, four (4) of his classmates
may cause, although it may escape or be

lost. This responsibility shall cease only

Never Let The Odds Keep You From Pursuing What You Know In Your Heart You Were Meant To Do.-
Leroy Satchel Paige
Page 113 of 180
Civil Law Q&As (2007-2013) hectorchristopher@yahoo.com dbaratbateladot@gmail.com

principally liable under Art 221 (FC), if

because the van rented by the school was they are negligent.

too crowded. On the way to a museum


which the students were scheduled to visit,

Rozanno made a wrong maneuver, causing


a collision with a jeepney. One of his

classmates died. He and the three (3) others


were badly injured.

(A). Who is liable for the death of Rozannos

classmate and the injuries suffered by


Rozanno and his 3 other classmates?

Explain. (2%)

SUGGESTED ANSWER:

At the time the incident occurred in May

1989, Rozanno was still a minor. Being a

minor, Art 218 of the Family Code

applies. Pursuant to Art 218, the school,

its administrators and teachers shall be

liable for the acts of minor Rozanno

because of the special parental authority

and responsibility that they exercise

over him. The authority applies to all

authorized activities, whether inside or

outside the premises of the school,

entity or institution. The field trip on

which occasion Rozanno drove the car,

was an authorized activity, and , thus,

covered by the provision. Furthermore,

the parents of Rozanno are subsidiarily

liable pursuant to Art 219 (FC), and


the middle of 1994, Rozanno have been

(B). How about the damage to the jeepney? 21 years old at the time. Hence, he was

Explain. (2%) already of legal age. The law reducing the

age of majority to 18 years took effect in


SUGGESTED ANSWER:
December 1989.

With respect to the damages caused to


Being of legal age, articles 218, 219, and
the jeepney, only Rozanno should be
221 of the Family Code are no longer
held liable because his negligence or
applicable. In such case, only Rozanno
tortuous act was the sole, proximate and
will be personally responsible for all the
immediate cause thereof.
consequences of his act unless his

(C). Under the same facts, except the date of school or his parents were themselves

occurrence of the incident, this time in also negligent and such negligence

mid-1994, what would be your answer? contributed to the happening of the

Explain. (2%) incident. In that event, the school or his

parents are not liable under Art 218, 218


SUGGESTED ANSWER:
or 221 of the Family Code, but will be

Since Rozanno was 16 years old in 1989,

if the incident happened sometime in

Never Let The Odds Keep You From Pursuing What You Know In Your Heart You Were Meant To Do.-
Leroy Satchel Paige
Page 114 of 180
Civil Law Q&As (2007-2013) hectorchristopher@yahoo.com dbaratbateladot@gmail.com

evidence (documentary and testimonial)


and legal arguments will you present in
liable under general provision on the

Civil Code on quasi-delict.

Quasi-Delict; Claims; Requisites (2013)

No.II. A collision occurred at an intersection


involving a bicycle and a taxicab. Both the

bicycle rider (a businessman then doing his


morning exercise) and the taxi driver

claimed that the other was at fault. Based


on the police report, the bicycle crossed the

intersection first but the taxicab, crossing


at a fast clip from the bicycle's left, could

not brake in time and hit the bicycle's rear


wheel, toppling it and throwing the bicycle

rider into the sidewalk 5 meters away.

The bicycle rider suffered a fractured right

knee, sustained when he fell on his right


side on the concrete side walk. He was

hospitalized and was subsequently


operated on, rendering him immobile for 3

weeks and requiring physical rehabilitation


for another 3 months. In his complaint for

damages, the rider prayed for the award


ofP1,000,000 actual damages,P200,000
moral damages, P200,000 exemplary
damages, P1 00,000 nominal damages
and P50,000 attorney's fees.

Assuming the police report to be correct

and as the lawyer for the bicycle rider, what


The case clearly involves quasi-delict

court to justify the damages that your client where my client, the bicycle rider,

claims? (8%) suffered injury as a result of the

negligence of the over-speeding taxi


SUGGESTED ANSWER:
driver, without fault on my clients part.

I will the base the claim of my client on


To prove actual damages aside from the
quasi-delict under Art 2176 of the Civil
testimony of client, I will present his
Code of the Philippines. The requisites
hospital and medical bills. Receipts paid
for a claim under quasi-delict to prosper
on the rehabilitation will also be
are as follows:
presented. [The sentence in red should

be replaced with the following sentence


(1) Act or omission, there being fault or
because he is a businessman and not an
negligence;
employee. Furthermore, I will present
(2) Damage or injury; and income tax returns, contracts and other

documents to prove unrealized profits as


(3) Causal connection between the
a result of this temporary injury.] I will
damage and the act or omission.

Never Let The Odds Keep You From Pursuing What You Know In Your Heart You Were Meant To Do.-
Leroy Satchel Paige
Page 115 of 180
Civil Law Q&As (2007-2013) hectorchristopher@yahoo.com dbaratbateladot@gmail.com

eyewitness accounts, in order to support

also call the attending physician to the award for moral damages.

testify as to the extent of the injuries


Exemplary damages may be granted if
suffered by my client, and to corroborate
the defendant acted in wanton,
the contents of the medical documents.

Based on Art. 2202, in quasi-delicts, the

defendant shall be liable for all damages

which are the natural and probable

consequences of the act or omission

complained of. It is not necessary that

the damages have been foreseen or could

have been foreseen by the defendant.

Unlike actual damages, no proof of

pecuniary loss is necessary in order that

moral, nominal, temperate liquidated or

exemplary damages may be adjudicated.

The assessment is left to the discretion

of the Court (Art. 2216, Civil Code).

There must be proof pecuniary

estimation, however.

Moral damages can be recovered by my

client under Articles 2219 and 2200.

Moral damages may be recovered in case

of a quasi-delict causing physical

injuries. Additionally, it must be proved

that such damages were the proximate

result of the act complained of. Medical

certificates will be presented, along with

the testimony from my client and other


Finally, attorneys fees may be recovered

fraudulent, reckless, oppressive, or when exemplary damages are awarded

(Art 2208, Civil Code).


malevolent manner. While the amount of

exemplary damages may not be proved,

the plaintiff must show that he is

entitled to moral or compensatory Quasi Tort (2010)

damages. In support of this, I will


No.III. Define, Enumerate or Explain. (2%
present the police report showing the
each)
circumstance under which the accident

took place, taking into account the (B). Define quasi tort. Who are the persons

actions of the parties. I will ask the liable under quasi torts and what are the

officials who responded to the accident defenses available to them?

to testify as to the conduct of the


Note: It is recommended that the examiner
parties at the time of the accident in
exercise leniency and liberality in grading the
order to determine whether defendant
answers given to this question. The term quasi-
was guilty of gross negligence.
tort is not part of legal developments in civil

law. In Philippine legal tradition, quasi-

Never Let The Odds Keep You From Pursuing What You Know In Your Heart You Were Meant To Do.-
Leroy Satchel Paige
Page 116 of 180
Civil Law Q&As (2007-2013) hectorchristopher@yahoo.com dbaratbateladot@gmail.com
Quasi -tort is considered as the

delict has been treated as the closest civil equivalent of quasi-delict. Hence the

law equivalent of the common law tort. In rules of the latter pertaining to persons

fact, in a number of Supreme Court

decisions, the two terms have been

considered synonymous. In reality, however,

the common law tort is much broader in

scope than the civil law quasi-delict. In

recent developments in common law, the

concept of quasi-torts can be considered as

the closest common law equivalent of the

civil law concept of quasi-delict. This is

because it is argued that the growing

recognition of quasi-torts as a source of

obligation is hinged on the acceptance at

common law of the civil law principles of

quasi-delict.

SUGGESTED ANSWER:

Quasi -tort is a legal concept upholding

the doctrine that some legal duty exists

that cannot be classified strictly as a

personal duty (thus resulting in a tort),

nor as a contractual duty but rather

some other kind of duty recognizable by

the law. Tort or Quasi-tort is an

Anglo American or Common Law

concept, while Delict or Quasi-Delict

is a Civil Law concept (Wikipedia

encyclopedia).

ALTERNATIVE ANSWER:
proximate cause of the injury ( Article

who can be held liable and their defenses 2179 NCC );

would also apply.


(c) That the plaintiff's own negligence

Those liable for quasi-delict include: was the immediate and proximate cause

of his injury ( Article 2179 NCC );


(1) Those tortfeasor or the person

causing damage to another through fault (d ) That the person vicariously liable

or negligence ( Article 2176 NCC ); and has observed all the diligence of a good

(2) Persons vicariously liable under father of a family to prevent damage

Article 2180 (NCC ). ( Article 2180 NCC ); and

(e) That the cause of action has

The defenses available include: prescribed after the lapse s (Article 2179

NCC ).
(a) That the defendant was not negligent

or that he exercised due diligence The fact that the plaintiff had
( Article 2176 NCC ); committed contributory negligence is a

partial defense (Art 2179, NCC).


(b) That although the defendant is

negligent his negligence is not the

Never Let The Odds Keep You From Pursuing What You Know In Your Heart You Were Meant To Do.-
Leroy Satchel Paige
Page 117 of 180
Civil Law Q&As (2007-2013) hectorchristopher@yahoo.com dbaratbateladot@gmail.com

If all the brothers/sisters are disqualified

to inherit, the nephews/nieces inherit


MULTIPLE CHOICE

QUESTIONS

2013 Taxation Law Exam

MCQ (October 13, 2013)

I. Armand died intestate. His full-blood

brothers, Bobby and Conrad, and half-


blood brothers, Danny, Edward and Floro,

all predeceased him. The following are the


surviving relatives:

1. Benny and Bonnie, legitimate children of


Bobby;
2. Cesar, legitimate child of Conrad;
3. Dante, illegitimate child of Danny;
4. Ernie, adopted child of Edward; and
5. Felix, grandson of Floro.
The net value of Armand's estate is
Pl,200,000.

I. (1) How much do Benny and Bonnie


stand to inherit by right of representation?
(1%)

(A) P200,000
(B) P300,000
(C) P400,000
(D) P150,000

(E) None of the above.

SUGGESTED ANSWER:

(E) None of the above.


relationship is legitimate. Thus, Dante,

per capita, and not by right of an illegitimate child of Danny, is barred

representation (Art 975, Civil Code) from inheriting from Armand pursuant

to the iron curtain rule which

I. (2) How much is Dante's share in the net disqualifies an illegitimate child from
estate? (1%) inheriting ab intestao from the
(A) P150,000. legitimate children and relatives of his
(B) P200,000.
father or mother, and vice versa (Art
(C) P300,000.
992, Civil Code).
(D) P400,000.

(E) None of the above.


I. (3) How much is Ernie's share in the net
estate . (1%)
SUGGESTED ANSWER:
(A) P 0.
E. None of the above.
(B) P400,000.
There is no showing that Danny is an
(C) P150,000.
illegitimate half-blood brother of
(D) P200,000.
Armand. In the absence of proof to the (E) None of the above.
contrary, the law presumes that the SUGGESTED ANSWER:

Never Let The Odds Keep You From Pursuing What You Know In Your Heart You Were Meant To Do.-
Leroy Satchel Paige
Page 118 of 180
Civil Law Q&As (2007-2013) hectorchristopher@yahoo.com dbaratbateladot@gmail.com

obligation became due and demandable, C


turned out to be insolvent.
(A) 0 or (E) None of the above.

The legal relationship created by

adoption is strictly between the adopter

and the adopted. It does not extend to

the relatives of either party (Sayson v.

CA, G.R. Nos. 89224-25, Jan 23, 1992).

(Note: E. None of the above is another

answer because Ernie has no share at

all in the net estate).

I. (4) How much is Felix's share in the net


estate? (1%)

(A) P400,000.
(B) P150,000.
(C) P300,000.

(D) P0.

(E) None of the above.

SUGGESTED ANSWER:

(D). 0. Or (E) None of the above.

In the collateral line, representation is

granted only to children of brother or

sisters, Felix is a grandson of a

predeceased brother.

(Note: E. None of the above: is another

answer because Felix has no share at

all in the net estate)

II. A, B, C and D are the solidary debtors of


X for P40,000. X released D from the

payment of his share of PI 0,000. When the


(D). No, as the release of the share of one

Should the share of insolvent debtor C be debtor would then increase the burden of

divided only between the two other the other debtors without their consent.

remaining debtors, A and B? (1%) When one of the solidary debtors cannot,

(A) Yes. Remission of D's share carries with because of his insolvency, reimburse his
it total extinguishment of his obligation to share to the debtor paying the
the benefit of the solidary debtors. obligation, such share shall be borne by
(B) Yes. The Civil Code recognizes remission
all his co-debtors, in proportion to the
as a mode of extinguishing an obligation.
debt of each (Art 1217, Civil Code).
This clearly applies to D.
Additionally, D was released only from
(C) No. The rule is that gratuitous acts
his share of P10,000.00 not from the
should be restrictively construed, allowing
solidary tie that binds him to A, B and C.
only the least transmission of rights.

(D) No, as the release of the share of one


III. Amador obtained a loan of P300,000
debtor would then increase the burden of
from Basilio payable on March25, 2012. As
the other debtors without their consent.
security for the payment of his loan,

Amador constituted a mortgage on his


SUGGESTED ANSWER:
residential house and lot in Basilio's favor.

Never Let The Odds Keep You From Pursuing What You Know In Your Heart You Were Meant To Do.-
Leroy Satchel Paige
Page 119 of 180
Civil Law Q&As (2007-2013) hectorchristopher@yahoo.com dbaratbateladot@gmail.com

and has resorted to all the legal remedies

Cacho, a good friend of Amador, guaranteed against the debtor (Art. 2058, Civil Code)

and obligated himself to pay Basilio, in case (Note: A is not the correct answer

Amador fails to pay his loan at maturity. because it states that all the assets of

III. (1) If Amador fails to pay Basilio his loan


on March 25, 2012, can Basilio compel
Cacho to pay? (1%)

(A) No, Basilio cannot compel Cacho to pay

because as guarantor, Cacho can invoke the


principle of excussion, i.e., all the assets of

Basilio must first be exhausted.

(B) No, Basilio cannot compel Cacho to

pay because Basilio has not exhausted

the available remedies against Amador.

(C) Yes, Basilio can compel Cacho to pay

because the nature of Cacho's undertaking


indicates that he has bound himself

solidarily with Amador.

(D) Yes, Basilio can compel Cacho who

bound himself to unconditionally pay in


case Amador fails to pay; thus the benefit of

excussion will not apply.

SUGGESTED ANSWER:

(B) No, Basilio cannot compel Cacho to

pay because Basilio has not exhausted

the available remedies against Amador.

The guarantor cannot be compelled to

pay the creditor unless the latter has

exhausted all the property of the debtor


ownership on the buyer, Diego, who must

Basilio (the creditor) must first be therefore consent.

exhausted) (D) No, Basilio cannot foreclose the real

III. (2) If Amador sells his residential house estate mortgage. To deprive the new owner

and lot to Diego, can Basilio foreclose the of ownership and possession is unjust and

real estate mortgage? (1%) inequitable.

(A) Yes, Basilio can foreclose the real estate SUGGESTED ANSWER:

mortgage because real estate mortgage (B) Yes, Basilio can foreclose the real

creates a real right that attaches to the estate mortgage. It is binding upon Diego

property. as the mortgage is embodied in a public


(B) Yes, Basilio can foreclose the real instrument.
estate mortgage. It is binding upon Diego Since the mortgage is in a public
as the mortgage is embodied in a public instrument, there is constructive notice
instrument. to Diego, who is the buyer if the
(C) No, Basilio cannot foreclose the real mortgaged property.
estate mortgage. The sale confers
ALTERNATIVE ANSWER:

Never Let The Odds Keep You From Pursuing What You Know In Your Heart You Were Meant To Do.-
Leroy Satchel Paige
Page 120 of 180
Civil Law Q&As (2007-2013) hectorchristopher@yahoo.com dbaratbateladot@gmail.com

(C) Yes, Jose's refusal is justified. The


principle of unjust enrichment warrants the
(C) No, Basilio cannot foreclose the real
reimbursement of Jose's expenses.
estate mortgage. The sale confers

ownership on the buyer, Diego, who must

therefore consent.

The mortgage is not registered, thus,

cannot be binding against third persons

(Art. 2125, Civil Code)

IV. Cruz lent Jose his car until Jose

finished his Bar exams. Soon after Cruz


delivered the car, Jose brought it to

Mitsubishi Cubao for maintenance check


up and incurred costs of P8,000. Seeing the

car's peeling and faded paint, Jose also had


the car repainted for P10,000. Answer the

two questions below based on these


common facts.

IV. (1) After the bar exams, Cruz asked for

the return of his car. Jose said he would


return it as soon as Cruz has reimbursed

him for the car maintenance and repainting


costs of P 18,000.

Is Jose's refusal justified? (1%)


(A) No, Jose's refusal is not justified. In this

kind of contract, Jose is obliged to pay for


all the expenses incurred for the

preservation of the thing loaned.

(B) Yes, Jose's refusal is justified. He is

obliged to pay for all the ordinary and


extraordinary expenses, but subject to

reimbursement from Cruz.


The bailee, Jose, has no right of

(D) No, Jose's refusal is not justified. The retention on the ground that the bailor

expenses he incurred are useful for the owes him something, even if it may be

preservation of the thing loaned. It is by reason of expenses. He can only

Jose's obligation to shoulder these useful retain it if he suffers damages by reason

expenses. of a flaw or defect in the thing loaned of

which the bailor knows (Art 1951, Civil

SUGGESTED ANSWER: Code).

(D) No, Jose's refusal is not justified. The

expenses he incurred are useful for the IV. (2) During the bar exam month, Jose
lent the car to his girlfriend, Jolie, who
preservation of the thing loaned. It is
parked the car at the Mall of Asia's open
Jose's obligation to shoulder these useful
parking lot, with the ignition key inside the
expenses.
car. Car thieves broke into and took the car.
In commodatum, the bailee is obliged to

pay for the ordinary expenses for the use Is Jose liable to Cruz for the loss of the car
and preservation of the thing loaned (Art due to Jolie's negligence? (1%)
1941, Civil Code).

Never Let The Odds Keep You From Pursuing What You Know In Your Heart You Were Meant To Do.-
Leroy Satchel Paige
Page 121 of 180
Civil Law Q&As (2007-2013) hectorchristopher@yahoo.com dbaratbateladot@gmail.com

000,000. P joined as an industrial partner,


contributing only his services. The Articles
(A) No, Jose is not liable to Cruz as the loss
of Partnership, registered with the
was not due to his fault or negligence.

(B) No, Jose is not liable to Cruz. In the

absence of any prohibition, Jose could lend


the car to Jolie. Since the loss was due to

force majeure, neither Jose nor Jolie is


liable.

(C) Yes, Jose is liable to Cruz. Since Jose

lent the car to Jolie without Cruz's

consent, Jose must bear the consequent

loss of the car.

(D) Yes, Jose is liable to Cruz. The contract


between them is personal in nature. Jose

can neither lend nor lease the car to a third


person.

SUGGESTED ANSWER:

(C) Yes, Jose is liable to Cruz. Since Jose

lent the car to Jolie without Cruz's

consent, Jose must bear the consequent

loss of the car.

The bailee is liable for the loss of the

thing, even if it should be through a

fortuitous event if he lends or leases the

thing to a third person, who is not a

member of his household (Art 1942, Civil

Code).

V. In 2005, L, M, N, 0 and P formed a

partnership. L, M and N were capitalist


partners who contributed P500,000 each,

while 0, a limited partner, contributed P1 ,


V. (l) Assuming that the just and equitable
share of the industrial partner, P, in the
Securities and Exchange Commission,
profit in 2006 amounted to P1 00,000, how
designated L and 0 as managing partners; L
much is the share of 0, a limited partner, in
was liable only to the extent of his capital
the P800,000 net profit? (1%)
contribution; and P was not liable for
(A) P160,000.
losses.
(B) P175,000.
In 2006, the partnership earned a net profit
(C) P280,000.
of P800,000. In the same year, P engaged in
a different business with the consent of all (D) P200,000.

the partners. However, in 2007, the (E) None of the above.

partnership incurred a net loss of


SUGGESTED ANSWER:
P500,000. In 2008,the partners dissolved
(C) P280,000.
the partnership. The proceeds of the sale of

partnership assets were insufficient to First, deduct the share of P from the

settle its obligation. After liquidation, the profits. P800,000 less P100,000 is

partnership had an unpaid liability P700,000. Next, get the share of O by

ofP300,000. following the proportion that the shares

of L, M, N, O is 1:1:1:2, respectively.

Never Let The Odds Keep You From Pursuing What You Know In Your Heart You Were Meant To Do.-
Leroy Satchel Paige
Page 122 of 180
Civil Law Q&As (2007-2013) hectorchristopher@yahoo.com dbaratbateladot@gmail.com

the partnership, 0 is liable as capitalist


partner.

V. (2) In 2007, how much is the share of 0,


a limited partner, in the net loss of
P500,000? (1%)

(A) P 0.

(B) P1 00,000.

(C) P125,000.

(D) P200,000.

(E) None of the above.

SUGGESTED ANSWER:

(D) P200,000

A limited partner shall not become liable

a s a general partner unless, in addition

to the exercise of his rights and powers

as a limited partner, he takes part in the

control of the business (Art 1948, Civil

Code). In the absence of stipulation as to

profits and losses, the share of each

partner in the losses shall be

proportionate to what he may have

contributed (Art 1797).

V. (3) Can the partnership creditors hold L,


0 and Pliable after all the assets of the
partnership are exhausted? (1%)

(A) Yes. The stipulation exempting P from

losses is valid only among the partners. L is


liable because the agreement limiting his

liability to his capital contribution is not


valid insofar as the creditors are concerned.

Having taken part in the management of


Articles of Partnership embodying such
stipulations serves as constructive notice to
(B) No. P is not liable because there is a
the partnership creditors.(E) None of the
valid stipulation exempting him from losses.
above is completely accurate.
Since the other partners allowed him to
(E) None of the above is completely
engage in an outside business activity, the
stipulation absolving P from liability is accurate.

valid. For 0, it is basic that a limited


partner is liable only up to the extent of his SUGGESTED ANSWER:

capital contribution. (E) None of the above is completely

(C) Yes. The stipulations exempting P and L accurate.

from losses are not binding upon the VI. Gary is a tobacco trader and also a

creditors. 0 is likewise liable because the lending investor. He sold tobacco leaves to

partnership was not formed in accordance Homer for delivery within a month,

with the requirements of a limited although the period for delivery was not

partnership. guaranteed. Despite Gary's efforts to deliver

(D) No. The Civil Code allows the partners on time, transportation problems and

to stipulate that a partner shall not be government red tape hindered his efforts

liable for losses. The registration of the and he could only deliver after 30 days.

Never Let The Odds Keep You From Pursuing What You Know In Your Heart You Were Meant To Do.-
Leroy Satchel Paige
Page 123 of 180
Civil Law Q&As (2007-2013) hectorchristopher@yahoo.com dbaratbateladot@gmail.com

contrary to the terms agreed upon, Gary


could not insist that Homer accept the
Homer refused to accept the late delivery
tobacco leaves.
and to pay on the ground that the agreed
term had not been complied with.

As lending investor, Gary granted a


Pl,000,000 loan to Isaac to be paid within

two years from execution of the contract. As


security for the loan, Isaac promised to

deliver to Gary his Toyota Innova within


seven (7) days, but Isaac failed to do so.

Gary was thus compelled to demand


payment for the loan before the end of the

agreed two-year term.

VI. (l) Was Homer justified in refusing to


accept the tobacco leaves? (1%)

(A) Yes. Homer was justified in refusing to

accept the tobacco leaves. The delivery was


to be made within a month. Gary's promise

of delivery on a "best effort" basis made the


delivery uncertain. The term, therefore, was

ambiguous.

(B) No. Homer was not justified in

refusing to accept the tobacco leaves. He

consented to the terms and conditions

of the sale and must abide by it.

Obligations arising from contract have

the force of law between the contracting

parties.

(C) Yes. Homer was justified in his refusal

to accept the delivery. The contract


contemplates an obligation with a term.

Since the delivery was made after 30 days,


It is clear under the facts that the period

(D) No. Homer was not justified in refusing of delivery of the tobacco leaves was not

to accept the tobacco leaves. There was no guaranteed. Gary anticipated other

term in the contract but a mixed condition. factors which may prevent him from
The fulfillment of the condition did not making the delivery within a month.
depend purely on Gary's will but on other True enough, transportation problems
factors, e.g., the shipping company and the
and government red tape did. Such
government. Homer should comply with his
slight delay was, thus, excusable.
obligation.
Obligations arising from contract have

the force of law between the contracting


SUGGESTED ANSWER:
parties and should be complied with in
(B) No. Homer was not justified in
good faith (Art. 1160, Civil Code)
refusing to accept the tobacco leaves. He

consented to the terms and conditions


VI. (2) Can Gary compel Isaac to pay his
of the sale and must abide by it.
loan even before the end of the two-year
Obligations arising from contract have
period? (1%)
the force of law between the contracting

parties.

Never Let The Odds Keep You From Pursuing What You Know In Your Heart You Were Meant To Do.-
Leroy Satchel Paige
Page 124 of 180
Civil Law Q&As (2007-2013) hectorchristopher@yahoo.com dbaratbateladot@gmail.com

or security renders the obligation

(A) Yes, Gary can compel Isaac to immediately demandable. Isaac lost his

immediately pay the loan. Non- right to make use of the period.

compliance with the promised guaranty

or security renders the obligation

immediately demandable. Isaac lost his

right to make use of the period.

(B) Yes, Gary can compel Isaac to


immediately pay the loan. The delivery of

the Toyota Innova is a condition for the


loan. Isaac's failure to deliver the car

violated the condition upon which the loan


was granted. It is but fair for Gary to

demand immediate payment.

(C) No, Gary cannot compel Isaac to


immediately pay the loan. The delivery of

the car as security for the loan is an


accessory contract; the principal contract is

still the P 1,000,000 loan. Thus, Isaac can


still make use of the period.

(D) No, Gary cannot compel Isaac to

immediately pay the loan. Equity dictates


that Gary should have granted a reasonable

extension of time for Isaac to deliver his


Toyota Innova. It would be unfair and

burdensome for Isaac to pay


the P1,000,000 simplybecause the
promised security was not delivered.

SUGGESTED ANSWER:

(A) Yes, Gary can compel Isaac to

immediately pay the loan. Non-

compliance with the promised guaranty


In 2008, Lita met and married Jaime. They
Under Art 1198 (2) of the Civil Code, the
now have a child of their own.
debtor shall lose every right to make use
While on a tour with her former high school
of the period when he does not furnish
classmates in a remote province of China in
to the creditor the guaranties or
2010, Lita was surprised to see Lito or
securities which he has promised. somebody who looked exactly like him, but

she was sure it was Lito because of the


VII. Lito was a commercial pilot who flew for extreme surprise that registered in his face
Pacific-Micronesian Air. In 1998, he was the
when he also saw her. Shocked, she
co-pilot of the airline's Flight MA916 that immediately fled to her hotel and post haste
mysteriously disappeared two hours after
returned to the country the next day. Lita
take-off from Agana, Guam, presumably
now comes to you for legal advice. She asks
over the Pacific Ocean. No trace of the plane
you the following questions:
and its 105 passengers and crew was ever
found despite diligent search; Lito himself VII. (l) If Lito is alive, what is the status of
was never heard of again. Lito left behind his marriage to Lita? (1%)
his wife, Lita, and their two children.

Never Let The Odds Keep You From Pursuing What You Know In Your Heart You Were Meant To Do.-
Leroy Satchel Paige
Page 125 of 180
Civil Law Q&As (2007-2013) hectorchristopher@yahoo.com dbaratbateladot@gmail.com

(A) The marriage is valid because Lita's


marriage to Lito was terminated upon Lito's
(A) The marriage subsists because the
disappearance for more than seven years.
marital bond has not been terminated by
death.

(B) The marriage was terminated when Lita


married Jaime.

(C) The marriage subsists because Lita's

marriage to Jaime is void.

(D) The marriage is terminated because Lito


is presumed dead after his plane has been

missing for more than 4 years.

(E) The marriage can be formally declared


terminated if Lito would not resurface.

SUGGESTED ANSWER:

(C) The marriage subsists because Lita's

marriage to Jaime is void.

For the purpose of contracting the

subsequent marriage under Art 41 of the

Family Code, the spouse present must

institute a summary proceeding as

provided in the Family Code for the

declaration of presumptive death of the

absentee, without prejudice to the effect

of the reappearance of the absent

spouse.

VII. (2) If Lito is alive, what is the status of


Lita's marriage to Jaime? (1%)
he shall be presumed dead doe all

(B) The marriage is valid. After an absence purposes, except for those of succession.

of more than 10 years, Lito is already This provision was not repealed by the

presumed dead for all purposes. present Family Code. Applying this to

(C) The marriage is void. Lito's mere the problem, (A) may be correct. (B) may
absence, however lengthy, is insufficient to also be correct. (C) and
authorize Lita to contract a subsequent
(D) may also be correct under Art 41 of
marriage.
the Family Code.
(D) The marriage is void. If Lito is indeed
alive, his marriage to Lita was never
VIII.Which of the following actions or
dissolved and they can resume their
defenses are meritorious: (1%)
marital relations at any time.

(A) An action for recovery of down


SUGGESTED ANSWER:
payment paid under a rescinded oral sale
Any answer is correct.
of real property.
Under Art 390 of the Civil Code, after an
(B) A defense in an action for ejectment that
absence of seven years, it being unknown the lessor verbally promised to extend or
whether or not the absentee still lives, renew the lease.

Never Let The Odds Keep You From Pursuing What You Know In Your Heart You Were Meant To Do.-
Leroy Satchel Paige
Page 126 of 180
Civil Law Q&As (2007-2013) hectorchristopher@yahoo.com dbaratbateladot@gmail.com

cancel the contract, the vendee likewise

(C) An action for payment of sum of money can file an action for the recovery of the

filed against one who orally promised to down payment on the basis of solution

answer another's debt in case the latter indebiti.

defaults.

(D) A defense in an action for damages that

the debtor has sufficient, but unliquidated


assets to satisfy the credit acquired when it

becomes due.

(E) None of the above.

SUGGESTED ANSWER:

(A) An action for recovery of down

payment paid under a rescinded oral sale

of real property.

An oral sale of real property is an

unenforceable contract under the

Statute of Frauds. Since, in the problem,

the vendee paid down payment, it takes

it out of the ambit of Statute of Frauds.

The rescission here must be in the sense

of resolution of the reciprocal obligation

arising from the contract of sale. If

rescinded (resolved) by the vendee on

account of the vendors failure to deliver

the thing sold, the parties will go back to

their status prior to the contract. If the

vendor refuses to return the down

payment, then the vendee can file an

action to recover the down payment.

If, on the other hand, the vendor and the

vendee mutually agree to rescind i.e.


can be enforced in a court action. (Art

1403, Civil Code)

ALTERNATIVE ANSWER: (d) The fact that a debtor has

(E) None of the above. unliquidated assets does not excuse him
(a) The recovery of the down payments from paying his debt.
should be made in the same action for (e) In the technical meaning of rescission
rescission. Otherwise, it would be a under Art 1191 of the Civil Code will be
ground for dismissal under Rule 2, Sec 4 adhered to, then there is no absolutely
of Rules of Court. correct answer. Hence, letter E is also a
(b) Lease of a real property is covered by possible answer.
the Statute of Frauds. Furthermore, it

also consists of interest in real property. IX. Betty entrusted to her agent, Aida,

Hence, it must be in writing. (Art 1403, several pieces of jewelry to be sold on

commission with the express obligation to


Civil Code)
turn over to Betty the proceeds of the sale,
(c) A contract of guaranty is a promise to
or to return the jewelries if not sold in a
answer for the debt of another and
month's time. Instead of selling the
hence, it is also covered by the Statute
jewelries, Aida pawned them with the
of Frauds. It must be in writing before it

Never Let The Odds Keep You From Pursuing What You Know In Your Heart You Were Meant To Do.-
Leroy Satchel Paige
Page 127 of 180
Civil Law Q&As (2007-2013) hectorchristopher@yahoo.com dbaratbateladot@gmail.com

reimbursed for the pawn value of the


jewelries.
Tambunting Pawnshop, and used the

money for herself. Aida failed to redeem the


pawned jewelries and after a month, Betty

discovered what Aida had done. Betty


brought criminal charges which resulted in

Aida's conviction for estafa.

Betty thereafter filed an action against

Tambunting Pawnshop for the recovery of


the jewelries. Tambunting raised the

defense of ownership, additionally arguing


that it is duly licensed to engage in the

pawnshop and lending business, and that it


accepted the mortgage of the jewelry in

good faith and in the regular course of its


business.

If you were the judge, how will you decide


the case? (1%)

(A) I will rule in favor of Betty. My ruling

is based on the Civil Code provision that

one who has lost any movable or has

been unlawfully deprived thereof may

recover it from the person in possession

of the same. Tam bunting's claim of good

faith is inconsequential.

(B) I will rule in favor of Betty.


Tambunting's claim of good faith pales into

insignificance in light of the unlawful


deprivation of the jewelries. However, equity

dictates that Tambunting must be


Although possession of movable property

(C) I will rule in favor of Tambunting. Its acquired in good faith is equivalent to a

good faith takes precedence over the right title, nevertheless, one who has lost any

of Betty to recover the jewelries. movable or has been unlawfully deprived

(D) I will rule in favor of Tambunting. Good thereof may recover it from the person
faith is always presumed. Tambunting's in possession of the same. Betty has
lawful acquisition in the ordinary course of been deprived unlawfully of her jewelries
business coupled with good faith gives it
by the estafa committed by Aida. The
legal right over the jewelries.
pledge of the said jewelries by Aida to

Tambunting pawnshop is void because


SUGGESTED ANSWER:
the pledgor is not the owner (Art 2085
(A) I will rule in favor of Betty. My ruling
(2), Civil Code). Tambuntings claim of
is based on the Civil Code provision that
good faith is inconsequential, because,
one who has lost any movable or has
aside from good faith, Tambunting must
been unlawfully deprived thereof may
prove also that it acquired the jewelries
recover it from the person in possession
at a public sale in order to be able to
of the same. Tam bunting's claim of good
retain the jewelries until reimbursed by
faith is inconsequential.

Never Let The Odds Keep You From Pursuing What You Know In Your Heart You Were Meant To Do.-
Leroy Satchel Paige
Page 128 of 180
Civil Law Q&As (2007-2013) hectorchristopher@yahoo.com dbaratbateladot@gmail.com

dismiss, claiming that the lease on which


the action is based, is unenforceable.
Betty the amount of loan including

interest (Art 559, Civil Code).

The only exception the law allows is

when there is acquisition in good faith of

the possessor at a public sale, in which

case, the owner cannot obtain its return

without reimbursing the price (Dizon v.

Suntay, 47 SCRA 160, Sept 29, 1972).

X. Arlene owns a row of apartment houses


in Kamuning, Quezon City. She agreed to

lease Apartment No. 1 to Janet for a period


of 18 months at the rate of P10,000 per

month. The lease was not covered by any


contract. Janet promptly gave Arlene two

(2) months deposit and 18 checks covering

the rental payment for 18 months. This


show of good faith prompted Arlene to

promise Janet that should Arlene decide to


sell the property, she would give Janet the

right of first refusal.

X. (1) Not long after Janet moved in, she


received news that her application for a

Master of Laws scholarship at King's


College in London had been approved.

Since her acceptance of the scholarship


entailed a transfer of residence, Janet asked

Arlene to return the advance rental


payments she made. Arlene refused,

prompting Janet to file an action to recover


the payments. Arlene filed a motion to
seek to enforce any right under the

If you were the judge, would you grant contract of lease.

Arlene's motion? (1%)


SUGGESTED ANSWER:

(A) Yes, I will grant the motion because the (D) No. I will not grant the motion

lease contract between Arlene and Janet because the cause of action does not
was not in writing, hence, Janet may not seek to enforce any right under the
enforce any right arising from the same contract of lease.
contract.
Janet is not asking for the continued use
(B) No, I will not grant the motion because
of the leased premises. Moreover, the
to allow Arlene to retain the advance
contract is aside the ambit of the
payments would amount to unjust
Statute of Frauds as the same has
enrichment.
already been partially performed.
(C) Yes, I will grant the motion because the

action for recovery is premature; Janet


X. (2)Assume that Janet decided not to
should first secure a judicial rescission of
accept the scholarship and continued
the contract of lease.
leasing Apartment No. 1. Midway through
(D) No. I will not grant the motion

because the cause of action does not

Never Let The Odds Keep You From Pursuing What You Know In Your Heart You Were Meant To Do.-
Leroy Satchel Paige
Page 129 of 180
Civil Law Q&As (2007-2013) hectorchristopher@yahoo.com dbaratbateladot@gmail.com

The right of first refusal involves a

the lease period, Arlene decided to sell transfer of interest in the real property.

Apartment No. 1 to Jun in breach of her As such, it is covered by the Statute of

promise to Janet to grant her the right of

first refusal. Thus, Janet filed an action


seeking the recognition of her right of first

refusal, the payment of damages for the


violation of this right, and the rescission of

the sale between Arlene and Jun.

Is Janet's action meritorious? (1%)

(A) Yes, under the Civil Code, a promise to


buy and sell a determinate thing is
reciprocally demandable.

(B) No, the promise to buy and sell a


determinate thing was not supported by a

consideration.

(C) Yes, Janet's right of first refusal was


clearly violated when the property was not

offered for sale to her before it was sold to


Jun.

(D) No, a right of first refusal involves an

interest over real property that must be

embodied in a written contract to be

enforceable.

(E) None of the above.

SUGGESTED ANSWER:

(D) No, a right of first refusal involves an

interest over real property that must be

embodied in a written contract to be

enforceable.
d) Juridical capacity cannot exist

Frauds under Art 1403 (2)(e) of the Civil without capacity to act.

Code. It must be in writing in order to be


2. Which of the following is NOT a
enforceable.
restriction on ones capacity to act?
a) Minority
b) Marriage
2012 Taxation Law Exam
c) Deaf-mute

MCQ (October 14, 2012) d) Civil Interdiction

1. Which of the following is NOT included in SUGGESTED ANSWER:


the attributes of juridical capacity? This question should be disregarded.
a) Juridical capacity is inherent in (NOTE: There is no correct answer among
every natural person, and therefore the choices given. All choices are restrictions
it is not acquired. on ones capacity to act. While Marriage is
b) Juridical capacity is lost only the only one not mentioned in Articles 38
through death. and 39 of the NCC as a restriction on
c) Juridical capacity is the fitness to
be the subject of legal relations.

Never Let The Odds Keep You From Pursuing What You Know In Your Heart You Were Meant To Do.-
Leroy Satchel Paige
Page 130 of 180
Civil Law Q&As (2007-2013) hectorchristopher@yahoo.com dbaratbateladot@gmail.com

capacity to act, it restricts the capacity of a b) Yes, the will is not valid under
married person in cases of adoption.) Philippine law.

c) No, attestation clause is not an


3. This attribute or incident of a case
act of the testator.
determine whether it is a conflict-of-laws
d) No, the governing law is
case or one covered by domestic law.
Spanish law.
a) Cause of action

b) Foreign element
Note: The facts do not state the Law
c) Jurisdiction
observed by the testator in executing his
d) Forum non conveniens
will. He could have observed Spanish Law

or Philippine Law (see comment of Tolentino


4. The capacity of an heir to succeed shall
to Art. 815 NCC in 3Tolentino117, 1992). If
be governed by the:
he observed Spanish Law, the opposition is
a) national law of the decedents
not correct because the will is valid under
heirs
Spanish Law, hence choice (d) is the correct
b) law of the country where the
answer. If he observed Philippine Law, the
decedent was a resident at the time
opposition is still not correct because
of his death
Philippine Law does not require the testator
c) national law of the person who
to sign the Attestation Clause of his will,
died
said clause not being his act. In such case,
d) law of the country where the
choice (c) is the correct answer).
properties of the decedent are
located.
6. Ramon, a Filipino, executed a will in
Manila, where he left his house and located
5. Atty. BUKO, a Filipino, executed a will
in BP Homes Paraaque in favor of his
while he was in Spain. The attestation
Filipino son, Ramgen. Ramons other
clause of the said will does not contain
children RJ and Ramona, both Turkish
Bukos signature. It is valid under Spanish
nationals, are disputing the bequest to
law. At its probate in Manila, it is being
Ramgen. They plotted to kill Ramgen.
opposed on the ground that the attestation
Ramon learned of the plot, so he tore his
clause does not contain BUKOs signature.
will in two pieces out of anger. Which
Is the opposition correct? Choose the best
statement is most accurate?
answer..

a) Yes, because it is a fatal defect.


a) The mere act of Ramon Sr. is readable.
immaterial because the will is still

Never Let The Odds Keep You From Pursuing What You Know In Your Heart You Were Meant To Do.-
Leroy Satchel Paige
Page 131 of 180
Civil Law Q&As (2007-2013) hectorchristopher@yahoo.com dbaratbateladot@gmail.com

b) Philippine law

c) Indonesia law
b) The mere act of tearing the will
amounts to revocation. d) Japanese law

c) The tearing of the will may

amount to revocation if coupled

with intent of revoking it.

d) The act of tearing the will is


material.

7. Even if the applicable law is a foreign

law, a count in the Philippines may be


constrained to apply Philippine law under

any of the following instances, except:

a) when the foreign law, judgment or

contract is contrary to a sound and


important public policy of the

forum;

b) when the property subject of

the case is located outside of the

Philippines;

c) when the foreign law or

judgment is penal in nature;

d) when the foreign law is

procedural in nature.

8. If a will is executed by a testator who was


born a Filipino citizen but became

naturalized Japanese citizen at the time of


his death, what law will govern its

testamentary provisions if the will is


executed in China and the property being

disposed is located in Indonesia?

a) Chinese law
a) American law
b) Philippine law

9. A Japanese national and a Filipino c) Canadian law


d) Japanese law
national entered into a contract for services

in Thailand. The services will be rendered


11. A French national revokes his will in
in Singapore. In case of breach, what law
Japan where he is domiciled. He then
will govern?
changed his domicile to the Philippines
a) Thailand law
where he died. The revocation of his will in
b) Philippine law
Japan is valid under Japanese law but
c) Singapore law
invalid under Philippine law. The affected
d) Japanese law
heir is a Malaysian national residing in the

10. Pedro (Filipino) and his wife Jane Philippines. What law will apply?

(American) executed a joint will in Canada, a) Japanese law

where such joint will is valid. In case the b) Philippine law

joint will is probated in Japan, what law c) French law


d) Malaysian law
will govern the formalities of the joint will?

Never Let The Odds Keep You From Pursuing What You Know In Your Heart You Were Meant To Do.-
Leroy Satchel Paige
Page 132 of 180
Civil Law Q&As (2007-2013) hectorchristopher@yahoo.com dbaratbateladot@gmail.com

construction, Bill will transfer and convey


his cattle ranch located in Japan in favor of
12. In the absence of contrary stipulation in
Pedro. In case Pedro performs his
a marriage settlement, property relations of
Filipino spouses shall be governed by ---

a) Philippines laws
b) Law of the place where the
spouses reside

c) Law of the place where the


properties are situated

d) Law of the place where they were


married.

13. The will of a Filipino executed in a


foreign country ---

a) cannot be probated in the


Philippines;

b) may be probated in the

Philippines provided that properties


in the estate are located in the

Philippines;

c) cannot be probated before the


death of the testator;

d) may be probated in the

Philippines provided it was

executed in accordance with the

laws of the place where the will

was executed.

14. Pedro (Filipino and Bill (American)


entered into a contract in Australia,

whereby it was agreed that Pedro will build


a commercial building for Bill in the

Philippines, and in payment for the


Filipino), by whom she had a daughter,
Regine. In 2009, Regine married James
obligation, but Bill fails or refuses to pay,
(son of Justine with Lea) in California,
what law will govern?
where such marriage is valid.
a) American law
b) Philippine law
15. What is the current status of the
c) Australian law
marriage of Charice and Justine under
d) Japanese law
Philippine laws?
(Facts for item numbers 15-18)
a) Valid
In 1989, Charice (Filipina) and Justine
b) Void
(American), were married in the
c) Voidable
Philippines. In 1990, they separated and
d) Dissolved
Justine went to Las Vegas where he
(Note: While Art 26 of the FC does not
obtained a divorce in the same year. He
categorically provide that the first marriage
then married another Filipina, Lea, in
is dissolved by the divorce obtained by the
Canada on January 1, 1992. They had two
foreign spouse abroad, but provides that
(2) sons, James and John (who were both
such divorce merely gives the Filipino spouse
born in 1992). In 1993, after failing to hear
the capacity to contract a second marriage, it
from Justine, Charice married Bugoy (a

Never Let The Odds Keep You From Pursuing What You Know In Your Heart You Were Meant To Do.-
Leroy Satchel Paige
Page 133 of 180
Civil Law Q&As (2007-2013) hectorchristopher@yahoo.com dbaratbateladot@gmail.com
complaint for immorality in the Supreme

Court, thus preventing him from taking


is believed that the dissolution of the first

marriage us the necessary consequence of

the foreign divorce.)

16. What id the status of the marriage


between Charice and Bugoy under
Philippine laws?

a) Valid
b) Void
c) Voidable
d) Unenforceable

17. What is the status of the marriage


between Charice and Bugoy under
Philippine laws?

a) Valid
b) Void
c) Voidable
d) Unenforceable

18. What is the status of the marriage


between Regine and James under
Philippine laws?

a) Valid
b) Void
c) Voidable
d) Unenforceable

19. Ricky and Princess were sweethearts.


Princess became pregnant. Knowing that

Ricky is preparing for the examinations,


Marforth, a lawyer and cousin of Princess,

threatened Ricky with the filing of a


20. Audrey, single, bought a parcel of land
examinations unless he marries Princess.
in Malolos City from Franco for P 1Million.
As a consequence of the threat, Ricky
A contract was executed between them
married Princess. Can the marriage be
which already vested upon Audrey full
annulled on the ground of intimidation
ownership of the property, although payable
under Article 45 of the Family Code?
in monthly installments for a period of four
Choose the best answer.
(4) years. One (1) year after the execution of
a) Yes, because without the threat,
the contract, Audrey got married to Arnel.
Ricky would not have married
They executed a marriage settlement
Princess.
whereby they agreed that their properties
b) Yes, because the threat to enforce
shall be governed by the regime of conjugal
the claim of Princess vitiates the
partnership of gains. Thereafter,
consent of Ricky in contracting the
subsequent installments were paid from the
marriage.
conjugal partnership funds. Is the land
c) No, because the threat made by
conjugal or paraphernal?
Marforth is just and legal.

d) No, because Marforth is not a

party to the contract of marriage


between Princess and Ricky.

Never Let The Odds Keep You From Pursuing What You Know In Your Heart You Were Meant To Do.-
Leroy Satchel Paige
Page 134 of 180
Civil Law Q&As (2007-2013) hectorchristopher@yahoo.com dbaratbateladot@gmail.com

Australia, a marriage solemnized by a


consular official is valid, provided that such
a) The land is conjugal because the
marriage is celebrated in accordance with
installments were paid from the
conjugal partnership funds.

b) The land is paraphernal

because ownership thereof was

acquired before the marriage.

c) The land is both conjugal and

paraphernal funds of installments


were paid from both the personal

funds of Audrey and the conjugal


partnership funds.

d) The land is paraphernal because

it was Audrey who purchased the


same.

21. Ernesto donated a mobile phone

worth P 32,000 to Hubert orally and


delivered the unit to Hubert who accepted.

Which statement is most accurate?

a) The donation is void and

Ernesto may get mobile phone

back.

b) The donation is void but Ernesto

cannot get the mobile phone back.

c) The donation is voidable and may

be anulled.

d) The donation is valid.

22. Agay, a Filipino citizen and Topacio, an


Australian citizen, got married in the

consular office of the Philippines in


Australia. According to the laws of
d) Valid, because such marriage is

recognized as valid in the place


the laws of such consular official. Under
where it was celebrated.
Philippine law, what is the status of the
marriage of Agay and Topacio? Choose the
(Note: The issues in the problem is whether
best answer.
or not the fact that one of the parties to the
a) Void, because the consular
marriage was an alien constituted absence
official only has authority to
of authority or mere irregularity of authority.
solemnize marriages between
The problem only give the choice, letter (a),
Filipinos.
in case it is interpreted as absence of
b) Valid, because according to the authority. The problem does not give a
laws of Australia, such consular choice in case it is interpreted as an
official has authority to celebrate irregularity thereby making all the other
the marriage. answers wrong).
c) Voidable, because there is an

irregularity in the authority of the


consular official to solemnize 23. Separation of property between spouses

marriages. during the marriage may take place only:

a) by agreement of the spouses.

Never Let The Odds Keep You From Pursuing What You Know In Your Heart You Were Meant To Do.-
Leroy Satchel Paige
Page 135 of 180
Civil Law Q&As (2007-2013) hectorchristopher@yahoo.com dbaratbateladot@gmail.com

b) If one of the spouses has given b) No trial shall be held without the
ground for legal separation. 6-month cooling off period being
c) Upon order of the court. observed.
d) If one spouse has abandoned the c) The spouses will be entitled to live
other. separately upon the start of the
trial.
24. The husband may impugn the d) The prosecuting attorney has
legitimacy of his child but not on the to conduct his own investigation.
ground that:

a) the wife is suspected of27. A husband by chance discovered


infidelity. hidden treasure on the paraphernal
b) the husband had a serious illness property of his wife. Who owns the
that prevented him from engaging in discovered treasure?
sexual intercourse. a) The half pertaining to the
c) they were living apart. husband (finder) belongs to the
d) he is physically incapable of conjugal partnership.
sexual intercourse. b) The half pertaining to the wife (as
owner) belongs to the conjugal
25. A marriage is void if: partnership.
a) solemnized with a marriage c) One half shall belong to the
license issued without complying husband as finder and the other
with the required 10-day posting. half shall belong to the wife as
b) solemnized by a minister whom owner of the property.
the parties believe to have the d) a and b
authority.
c) between parties both 23 years of 28. Which of the following marriages is void
age but without parental advice. for reasons of public policy?
d) none of the above a) Between brothers and sisters,
whether of the full or half blood.
26. In legal separation, which is not b) Between step-parents and step
correct? children.
a) The aggrieved spouse may file the c) Between parents-in-law and
action within five (5) years from the children-in-law.
time of the occurrence of the cause. d) b and c

Never Let The Odds Keep You From Pursuing What You Know In Your Heart You Were Meant To Do.-
Leroy Satchel Paige
Page 136 of 180
Civil Law Q&As (2007-2013) hectorchristopher@yahoo.com dbaratbateladot@gmail.com

d) The donation must be made in

favor of one or both of the future


29. The following constitute the different
spouses.
circumstances or case of fraud which will
serves as ground for the annulment of a
31. Who are illegitimate children?
marriage, except?

a) Non-disclosure of the previous

conviction by final judgment of the


other party of a crime involving

moral turpitude.

b) Concealment of a sexually-

transmissible disease, regardless of


its nature, existing at the time of the

marriage.

c) Concealment of drug addiction,

habitual alcoholism, homosexuality


or lesbianism existing at the time of

marriage.

d) Concealment by the wife or the

husband of the fact of sexual

relations prior to the marriage.

30. Which of the following is not a requisite


for a valid donation propter nuptias?

a) The donation must be made

before the celebration of the


marriage.

b) The donation shall be

automatically revoked in case of

non-celebration of the marriage.

c) The donation must be made in

consideration of the marriage.


a) Filiation has been recognized by
the father through the record of
a) Children conceived or born
birth appearing in the civil register
outside a valid marriage.
b) Admission of filiation by the
b) Children born under a valid
father in a public document.
marriage, which was later declared
c) Private handwritten instrument is
void because of the psychological
made by the father acknowledging
incapacity of either or both of the
his filiation.
spouses.
d) Affidavit by the mother stating
c) Children conceived and born
the name of his true father.
outside a valid marriage.

d) Children born under a valid


33. Under RA 8043, an adopter is required
marriage, but the parents later
to be at least ____ years old and ____ years
obtained a legal separation.
older than the child to be adopted at the

time of the application unless the adopter is


32. An illegitimate child may use the
the parent by nature of the child.
surname of his father when his filiation is
a) 30 and 15
established in any of the following
b) 27 and 16
instances, except:

Never Let The Odds Keep You From Pursuing What You Know In Your Heart You Were Meant To Do.-
Leroy Satchel Paige
Page 137 of 180
Civil Law Q&As (2007-2013) hectorchristopher@yahoo.com dbaratbateladot@gmail.com

c) 50 and 10 d) Subjects the child or allows him


d) 18 and 15 to be subjected to acts of
lasciviousness.
34. Under RA 8043, a child qualified to be

adopted is any person below _____ years 37. Which of the following statements
old. is wrong?
a) 18 a) The possessor in bad faith shall
b) 21 reimburse the fruits received and
c) 15 those which the legitimate possessor
d) 16 could have received.

b) The possessor in bad faith has


35. Which of the following DOES NOT
right of reimbursement for
result in permanent termination of parental
necessary expenses and those for
authority?
the production, gathering and
a) Death of the parents.
preservation of the fruits.
b) Death of the child.
c) The possessor in bad faith is not
c) Emancipation of the child.
entitled to a refund of ornamental
d) Conviction of the parents of a
expenses.
crime which carries with it the
d) The possessor in bad faith is
penalty of civil interdiction.
entitled to a refund of useful

expenses.
36. The court, in an action filed for the
purpose, may suspend parental authority if
38. Which phrase most accurately
the parent or the person exercising parental
completes the statement The expenses
authority commits any of the following acts,
incurred in improvements for the luxury or
except:
mere pleasure shall not be refunded to thew
a) Treats the child with excessive
possessor in bad faith:
harshness or cruelty.
a) but he may remove the objects
b) Gives the child corrupting orders,
for which such expenses have
counsel or example.
been incurred, provided that the
c) Compels the child to take up a
thing suffers no injury thereby,
course in college against his/her

will.
and that the lawful possessor does

not prefer to retain them.

Never Let The Odds Keep You From Pursuing What You Know In Your Heart You Were Meant To Do.-
Leroy Satchel Paige
Page 138 of 180
Civil Law Q&As (2007-2013) hectorchristopher@yahoo.com dbaratbateladot@gmail.com

41. Action to recover real property based on


ownership. Here, the object is the recovery
b) and he may not remove the
objects for which such expenses
have been incurred.

c) and he may not remove the


objects for which such expenses

have been incurred, unless he pays


the value they may have at the time

he entered into possession.

d) but he may remove the objects for


which such expenses have been
incurred.

39. The following are the limitations on the


right of ownership imposed by the owner

himself, except:

a) Will/Succession
b) Mortgage
c) Pledge
d) Lease

40. A plenary action for the recovery of the

possession of real estate, upon mere


allegation and proof of a better right

thereto, and without allegation of proof of


title. This action can only be brought after

the expiration of one (1) year. What action is


being referred to?

a) Accion publiciana
b) Accion reinvindicatoria
c) Accion interdictal
d) Quieting of Title
43. The following things are property of
public dominion, except:
of the dominion over the property as owner.
a) ports and bridges constructed by
What action is being referred to?
the State.
a) Accion publiciana
b) vehicles and weapons of the
b) Accion reinvindicatoria
Armed Forces of the Philippines.
c) Accion interdictal
c) rivers.
d) Quieting of Title
d) lands reclaimed by the state from

42. A summary action to recover physical the sea.

or material possession only and must be


SUGGESTED ANSWER:
brought within one (1) year from the time
This question should be disregarded
the cause of action arises. What action is
because there is no correct answer.
being referred to?
(Note: At first glance, one gets the impression
a) Accion publiciana
b) Accion reinvindicatoria that vehicles and weapons of the AFP are

c) Accion interdictal not property of the public domain. But they

d) Quieting of Title are actually property of the public dominion

under the second paragraph of Art 420 of

Never Let The Odds Keep You From Pursuing What You Know In Your Heart You Were Meant To Do.-
Leroy Satchel Paige
Page 139 of 180
Civil Law Q&As (2007-2013) hectorchristopher@yahoo.com dbaratbateladot@gmail.com
46. Donation is perfected from the moment
---
the NCC. Property of the state which are not

for public use but are intended for some

public service are properties of the public

dominion. While the vehicles and weapons

of the AFP are not for public use, they are

used for the defense of the State which is a

public service.)

44. Which of the following statements

is wrong?

a) patrimonial property of the

state, when no longer intended for

public use or for public service,

shall become property of public

dominion.

b) all property of the State, which is

not of public dominion, is


patrimonial property.

c) The property of provinces, cities

and municipalities is divided into

property for public use and


patrimonial property.

d) Property is either of public

dominion or of private ownership.

45. The following cannot ask for the

reduction of inofficious donation, except:

a) Creditors of the deceased


b) Devisees or legatees

c) Compulsory heirs of the donor


d) The surviving spouse of the
donee.
48. It is a conduct that may consist of
a) the donee accepts the donation.
giving, doing, or not doing something.
b) the donor executes the deed of
a) Obligation
donation. b) Juridical necessity
c) the donor knows of the donees c) Prestation
acceptance even if the latter has d) Contract

not received the copy of the deed 49. It is a juridical relation arising
from lawful, voluntary and unilateral acts
of donation.
based on the principle that no one should
d) the donee confirms that the
unjustly enrich himself at the expense of
donor has learned the formers
another.
acceptance.
a) Quasi-contract

47. The following are the elements of an b) Quasi-delict


c) Cotract
obligation, except:
d) Delict
a) Juridical/Legal Tie
b) Active subject
50. The following are the elements of quasi-
c) Passive subject
delict, except:
d) Consideration

Never Let The Odds Keep You From Pursuing What You Know In Your Heart You Were Meant To Do.-
Leroy Satchel Paige
Page 140 of 180
Civil Law Q&As (2007-2013) hectorchristopher@yahoo.com dbaratbateladot@gmail.com

a) Act or omission 54. It is an international evasion of the


b) Fault/negligence faithful performance of the obligation.
c) Damage/injury a) Negligence
d) Pre-existing contract b) Fraud
c) Delay
51. A debtor is liable for damages in case of d) Mistake
delay if he is guilty of any of the following,

except: 55. The following are the requisites of

a) default (mora) fortuitous event, except:

b) mistake a) Cause is independent of the will


c) negligence (culpa) of the debtor.
d) breach through contravention of b) The event
the tenor thereof is unforeseeable/unavoidable.

c) Occurrence renders it absolutely


52. This term refers to a delay on the part impossible for the debtor to fulfill his
of both the debtor and creditor in reciprocal obligation in a normal manner;
obligations. impossibility must be absolute not
a) Mora accipiendi partial, otherwise not force majeure.
b) Mora solvendi
d) Debtor contributed to the
c) Compensation morae
aggravation of the injury to the
d) Solution indibiti
creditor.

53. The following are the requisites of mora


56. A debtor may still be held liable for loss
solvendi, except:
or damages even if it was caused by a
a) Obligation pertains to the debtor
fortuitous event in any of the following
and is determinate, due,
instances, except:
demandable, and liquidated.
a) The debtor is guilty of dolo, malice
b) Obligation was performed on its
or bad faith, has promised the same
maturity date.
thing to two or more persons who do
c) There is judicial or extrajudicial
not have the same interest.
demand by the creditor.
b) The debtor contributed to the
d) Failure of the debtor to comply
loss.
with such demand.
c) The thing to be delivered is

generic.

Never Let The Odds Keep You From Pursuing What You Know In Your Heart You Were Meant To Do.-
Leroy Satchel Paige
Page 141 of 180
Civil Law Q&As (2007-2013) hectorchristopher@yahoo.com dbaratbateladot@gmail.com

b) No, because the creditor may

d) The creditor is guilty of fraud, proceed against any one of the

negligence or delay or if he solidary debtors or some or all of

contravened the tenor of the them simultaneously.

obligation.

57. Buko, Fermin and Toti bound

themselves solidarily to pay Ayee the


amount of P 5,000.00. Suppose Buko paid

the obligation, what is his right as against


his co-debtors?

a) Buko cas ask for

reimbursement from Fermin and

Toti.

b) Buko can sue Fermin and Toti for

damages.

c) Buko can sue for rescission.

d) Buko can claim a refund from

Ayee.

58. Buko, Fermin and Toti bound

themselves solidarily to pay Ayee the sum of


P 10,000.00. When the obligation became

due and demandable, Ayee sued Buko for


the payment of the P 10,000.00. Buko

moved to dismiss on the ground that there


was failure to implead Fermin and Toti who

are indispensable parties. Will the motion


to dismiss prosper? Why?

a) Yes, because Fermin and Toti

should have been impleaded as their

obligation is solidary.
c) No, because in solidary obligation

any one of the solidary debtors can


c) No, because a motion to dismiss
pay the entire debt.
is a prohibited pleading.

d) Yes, because Fermin and Toti d) Yes, because Fermin and Toti will

should also pay their share of the be unduly enriched at the expense

obligation. of Buko.

59. Buko, Fermin and Toti are solidarily 60. Buko, Fermin and Toti are solidary

debtors of Ayee. Twelve (12) years after the debtors under a loan obligation of P

obligation became due and demandable, 300,000.00 which has fallen due. The

Buko paid Ayee and later on asked for creditor has, however, condoned Fermins

reimbursement of Fermins and Totis entire share in the debt. Since Toti has

shares. Is Buko correct? Why? become insolvent, the creditor makes a

demand on Buko to pay the debt. How


a) No, because the obligation has
much, if any, may Buko be compelled to
already prescribed.
pay?
b) Yes, because the obligation is
a) P 200.000.00
solidary.
b) P 300,000.00

c) P 100,000.00

Never Let The Odds Keep You From Pursuing What You Know In Your Heart You Were Meant To Do.-
Leroy Satchel Paige
Page 142 of 180
Civil Law Q&As (2007-2013) hectorchristopher@yahoo.com dbaratbateladot@gmail.com

and also of the same quality if the


latter has been stated.
d) P 150,000.00
c) That the two (2) debts are not

61. Dina bought a car from Jai and yet due.

delivered a check in payment of the same.


Has Dina paid the obligation? Why?

a) No, not yet. The delivery of

promissory notes payable to

order, or bills of exchange or

other mercantile documents shall

produce the effect of payment

only when they have been cashed,

or when through the fault of the

creditor they have been impaired.

b) Yes, because a check is a valid

legal tender of payment.

c) It depends. If the check is a

managers check or cashiers check


it will produce the effect of payment.

If its an ordinary check, no


payment.

d) Yes, because a check is as good

as cash.

62. The following are the requisites of legal

compensation, except:

a) That each of the obligors is bound

principally and that he be the same

time a principal creditor of the


other.

b) That both debts consist in a sum

of money, or if the things due are

consumable, they be the same kind,


also to the natural consequences that flow
out of such agreement.
d) That they be liquidated and
a) Obligatory force of contracts
demandable.
b) Mutuality of contracts

63. Which of the following statements c) Autonomy of contracts

is correct? d) Relativity of contracts

a) All contracts are perfected by


65. It is a principle which holds that
mere consent.
contracts must be binding to both parties
b) All contracts are perfected by
and its validity and effectivity can never be
delivery of the object.
left to the will of one of the parties.
c) All contracts are required to be in
a) Obligatory force of contracts
writing.
b) Mutuality of contracts
d) All contracts are required to
c) Autonomy of contracts
have a valid consideration.
d) Relativity of contracts

64. It is a principle which holds that parties 66. It refers to the rule that a contract
are bound not only by what has been is binding not only between

expressly provided for in the contract but

Never Let The Odds Keep You From Pursuing What You Know In Your Heart You Were Meant To Do.-
Leroy Satchel Paige
Page 143 of 180
Civil Law Q&As (2007-2013) hectorchristopher@yahoo.com dbaratbateladot@gmail.com
a) Death, civil interdiction,
insanity/insolvency of either party
parties but extends to the heirs, successors
before acceptance is conveyed.
in interest, and assignees of the parties,

provided that the contract involved

transmissible rights by their nature, or by

stipulation or by law.

a) Obligatory force of contracts


b) Mutuality of contracts
c) Autonomy of contracts

d) Relativity of contracts

67. It is rule which holds that the freedom


of the parties to contract includes the

freedom to stipulate, provided the


stipulations are not contrary to law, morals,

good customs, public order or public policy.

a) Obligatory force of contracts


b) Mutuality of contracts

c) Autonomy of contracts
d) Relativity of contracts

68. The following are the ways by which


innominate contracts are

regulated, except:

a) By the stipulation of the parties.

b) By the general principles of

quasi-contracts and delicts

c) By the rules governing the most

analogous nominate contracts.

d) By the customs of the place.


69. An offer becomes ineffective on any of

the following grounds, except:


specific acceptance.
d) Advertisements for Bidders
b) Acceptance of the offer by the
are only invitations to make
offeree. proposals and the advertiser is
c) Qualified/conditional acceptance not bound to accept the
of the offer, which becomes counter- highest/lowest bidder, unless it

offer. appears otherwise.


71. The following are solemn
d) Subject matter becomes
illegal/impossible before acceptance
contracts (Contracts which must appear in
is communicated.
writing), except:

70. Which of the following statements a) Donations of real estate or of

is correct? movables if the value

a) Offers in interrelated contracts exceeds P 5,000.00.

are perfected upon consent. b) Stipulation to pay interest in

b) Offers in interrelated contracts loans.

require a single acceptance. c) Sale of land through an agent

c) Business advertisements (authority must be in writing).


are definite offers that require

Never Let The Odds Keep You From Pursuing What You Know In Your Heart You Were Meant To Do.-
Leroy Satchel Paige
Page 144 of 180
Civil Law Q&As (2007-2013) hectorchristopher@yahoo.com dbaratbateladot@gmail.com

d) Construction contract of a74. The following are the characteristics of


building. a voidable contract, except:
a) Effective until set aside.
72. The following are rescissible b) May be assailed/attacked only in
contracts, except: an action for that purpose.
a) Entered into by guardian c) Can be confirmed or ratified.
whenever ward suffers damage more d) Can be assailed only by either
than of value of property. party.
b) Agreed upon in representation of
absentees, if absentee suffers lesion 75. The following are void
by more than of value of property. contracts, except:
c) Contracts where fraud is a) Pactum commissorium
committed on creditor (accion b) Pactum de non alienando
pauliana). c) Pactum leonina
d) Contracts entered into by d) Pacto de retro
minors.
76. The borrower in a contract of loan or
73. The following are the requisites before a mutuum must pay interest to the lender.
contract entered into in fraud of creditors a) If there is an agreement in
may be rescinded, except: writing to the effect.
a) There must be credited existing b) As a matter of course.
prior to the celebration of the c) If the amount borrowed is very
contract. large.
b) There must be fraud, or at least, d) If the lender so demands at the
the intent to commit fraud to the maturity date.
prejudice of the creditor seeking
rescission. 77. The liability of the school, its
c) The creditor cannot in any legal administrators and teachers, or the
manner collect his credit (subsidiary individual, entity or institution engaged in
character of rescission) child care over the minor child or damage
d) The object of the contract must caused by the acts or omissions of the
be legally in the possession of a unemancipated minor while under their
rd supervision, instruction or custody shall be:
3 person in good faith.
a) Joint and subsidiary
b) Principal and solidary

Never Let The Odds Keep You From Pursuing What You Know In Your Heart You Were Meant To Do.-
Leroy Satchel Paige
Page 145 of 180
Civil Law Q&As (2007-2013) hectorchristopher@yahoo.com dbaratbateladot@gmail.com

82. An obligation which is based on equity


and natural law is known as:
c) Principal and joint
a) pure
d) Subsidiary and solidary.

78. The creditor has the right to the fruits


of the thing from the time:

a) the thing is delivered.

b) the obligation to deliver the

things arises.

c) the contract is perfected.

d) the fruits are delivered.

79. If one of the parties to the contract is


without juridical capacity, the contract is:

a) voidable

b) rescissible

c) void

d) unenforceable

80. When both parties to the contract are


minors, the contract is:

a) voidable

b) rescissible

c) void

d) unenforceable

81. When the consent of one of the parties


was vitiated, the contract is:

a) voidable

b) rescissible

c) void

d) unenforceable
b) valid because all of the essential

b) quasi-contract requisites of a contract are present.

c) civil c) unenforceable because Michael

d) natural Fermin had no authority but he

sold the car in the name of Mr.


83. Consent was given by one in
Lacas, the owner.
representation of another but without
d) rescissible because the contract
authority. The contract is:
caused lesion to Atty. Buko.
a) voidable

b) rescissible 85. Which of the following contracts is void?


c) void a) An oral sale of a parcel of land.
d) unenforceable b) A sale of land by an agent in a
84. Michael Fermin, without the authority
public instrument where his
of Pascual Lacas, owner of a car, sold the
authority from the principal is
same car in the name of Mr. Lacas to Atty.
oral.
Buko. The contract between Atty. Buko and
c) A donation of a wrist watch
Mr. Lacas is ---
worth P 4,500.00.
a) void because of the absence of

consent from the owner, Mr. Lacas.

Never Let The Odds Keep You From Pursuing What You Know In Your Heart You Were Meant To Do.-
Leroy Satchel Paige
Page 146 of 180
Civil Law Q&As (2007-2013) hectorchristopher@yahoo.com dbaratbateladot@gmail.com

b) If Aligada refused to deliver the

d) A relatively simulated contract land, Balane may successfully sue

86. Which of the following expresses a

correct principle of law? Choose the best


answer.

a) Failure to disclose facts when

there is a duty to reveal them, does


not constitute fraud.

b) Violence or intimidation does not

render a contract annullable if

employed not by a contracting party


but by a third person.

c) A threat to enforce ones claim

through competent authority, if the

claim is legal or just, does not vitiate


consent.

d) Absolute simulation of a

contract always results in a void

contract.

87. Aligada orally offered to sell his two-


hectare rice land to Balane for P 10Million.

The offer was orally accepted. By


agreement, the land was to be delivered

(through execution of a notarized Deed of


Sale) and the price was to be paid exactly

one-month from their oral agreement.


Which statement is most accurate?

a) If Aligada refuses to deliver the

land on the agreed date despite

payment by Balane, the latter may


not successfully sue Aligada

because the contract is oral.


the contract are not transmissible
by their nature, or by stipulation or
for fulfillment of the obligation even
by provision of law.
if he has not tendered payment of
the purchase price. c) If a contract should contain some

stipulation in favor of a third


c) The contract between the parties
person, he may demand its
is rescissible.
fulfillment provided he
d) The contract between the
communicated his acceptance to the
parties is subject to ratification
obligor before its revocation.
by the parties.
d) In contracts creating real

rights, third persons who come


88. Which of the following statements

is wrong? into possession of the object of

a) Creditors are protected in cases of the contract are not bound

contracts intended to defraud them. thereby.

b) Contracts take effect only

between the parties, their assign 89. Which phrase most accurately

and heirs, except in case where the completes the statement Any third person

rights and obligations arising from who induces another to violate his contract:

Never Let The Odds Keep You From Pursuing What You Know In Your Heart You Were Meant To Do.-
Leroy Satchel Paige
Page 147 of 180
Civil Law Q&As (2007-2013) hectorchristopher@yahoo.com dbaratbateladot@gmail.com

c) Right to inherit

a) shall be liable for damages only if d) Agency

he is a party to the same contract.

b) shall be liable for damages to

the other contracting party.

c) shall not be liable for damages to

the other contracting party.

d) shall not be liable for damages if

the parties are in pari delicto.

90. The requisites of succession are as

follows, except:

a) Death of decedent
b) Transmissible estate
c) Existence and capacity of

successor, designated by decedent


or law

d) Payment of Taxes

91. The characteristics of succession are as

follows, except:

a) It is a legal contract.
b) Only property, rights and

obligations to the extent of the value


of the inheritance are transmitted.

c) The transmission takes place only


at the time of death.

d) The transmission takes place


either by will or by operation of law.

92. The following rights are extinguished by

death, except:

a) Legal support
b) Parental authority
a) Entirely written;

b) Dated;

93. The attestation clause contains the c) Signed by testator himself

following, except: d) Notarized by a notary public.

a) the number of pages used;


95. The following are the grounds for
b) that the testator signed or caused
disallowance of wills, except:
another to sign the will and every
a) The formalities required by law
page thereof in the presence of the
have not been complied with.
instrumental witnesses;

c) notary public; b) The testator was insane or

mentally incapable of making will.


d) the instrumental witnesses

witnessed and signed the will and c) The will was executed through

all the pages thereof in the presence force or under duress, or influence

of the testator and one another. of fear or threats.

d) The will contains an attestation

94. The following are the formalities clause.


required in the execution of holographic

will, except:

Never Let The Odds Keep You From Pursuing What You Know In Your Heart You Were Meant To Do.-
Leroy Satchel Paige
Page 148 of 180
Civil Law Q&As (2007-2013) hectorchristopher@yahoo.com dbaratbateladot@gmail.com

99. A contract granting a privilege to a


96. It is the omission in the testators will of
person, for which he has paid a
one, some or all of the compulsory heirs in
direct line, whether living at the time of

execution of the will or born after the death


of the testator. What principle is being

referred to?

a) reserva troncal

b) preterition

c) fideicommissary

d) disposicion captatoria
97. Any disposition made upon the
condition that the heir shall make some

provision in his will in favor of the testator


or of any other person shall be void. Here,

both the condition and the disposition are


void. What principle is being referred to?

a) reserva troncal

b) preterition

c) fideicommissary

d) disposicion captatoria

98. Which phrase most accurately


completes the statement If at the time the

contract of sale is perfected, the thing


which is the object of the contract has been

entirely lost:

a) the buyer bears the risk of loss.

b) the contract shall be without

any effect.

c) the seller bears the risk of loss.

d) the buyer may withdraw from the

contract.
authority is not reduced into

consideration, which gives him the right to writing.

buy certain merchandise or specified c) Sale of EGMs car by KRP, a

property, from another person, at anytime person stranger to EGM, without

within the agreed period, at a fixed price. EGMs consent or authority.

What contract is being referred to? d) Sale of EGMs piece of land by

a) Option Contract KRP, a person stranger to EGM,

b) Contract to Sell without EGMs consent or authority.

c) Contract of Sale
d) Lease
2011 Taxation Law Exam
100. Which of the following contracts of
MCQ (November 13, 2011)
sale is void?

a) Sale of EGMs car by KRP, EGMs (1)When does a declaration of absence of a


agent, whose authority is not missing person take effect?
reduced into writing. (A) Immediately from the issuance of
b) Sale of EGMs piece of land by the declaration of absence.

KRP, EGMs agent, whose

Never Let The Odds Keep You From Pursuing What You Know In Your Heart You Were Meant To Do.-
Leroy Satchel Paige
Page 149 of 180
Civil Law Q&As (2007-2013) hectorchristopher@yahoo.com dbaratbateladot@gmail.com

(4) Upon the proposal of a third person, a


new debtor substituted the original debtor
(B) 3 months after the publication of
without the latters consent. The creditor
the declaration of absence.

(C) 6 months after the publication

of the declaration of absence.

(D) 15 days from the issuance of the


declaration of absence.

(2) The authority that school administrators

exercise over school children under their


supervision, instruction, or custody is

called

(A) legal parental authority.


(B) substitute parental authority.
(C) ordinary parental authority.

(D) special parental authority.

(3) Can future inheritance be the subject of


a contract of sale?

(A) No, since it will put the


predecessor at the risk of harm from

a tempted buyer, contrary to public


policy.

(B) Yes, since the death of the


decedent is certain to occur.

(C) No, since the seller owns no

inheritance while his predecessor

lives.

(D) Yes, but on the condition that

the amount of the inheritance can


only be ascertained after the

obligations of the estate have been


paid.
avoid unjust enrichment on his
part.
accepted the substitution. Later, however,

the new debtor became insolvent and


(5) Lennie bought a business class ticket
defaulted in his obligation. What is the
from Alta Airlines. As she checked in, the
effect of the new debtors default upon the
manager downgraded her to economy on
original debtor?
the ground that a Congressman had to be
(A) The original debtor is freed of
accommodated in the business class.
liability since novation took place
Lennie suffered the discomfort and
and this relieved him of his
embarrassment of the downgrade. She sued
obligation. the airlines for quasi-delict but Alta Airlines
(B) The original debtor shall pay or countered that, since her travel was
perform the obligation with recourse governed by a contract between them, no
to the new debtor. quasi-delict could arise. Is the airline
(C) The original debtor remains correct?
liable since he gave no consent to (A) No, the breach of contract may
the substitution.
in fact be tortious as when it is
(D) The original debtor shall pay or
tainted as in this case with
perform 50% of the obligation to

Never Let The Odds Keep You From Pursuing What You Know In Your Heart You Were Meant To Do.-
Leroy Satchel Paige
Page 150 of 180
Civil Law Q&As (2007-2013) hectorchristopher@yahoo.com dbaratbateladot@gmail.com

wedding, however, Y suddenly died of heart


attack. Can Ys heirs get the property?
arbitrariness, gross bad faith, and

malice.

(B) No, denying Lennie the comfort


and amenities of the business class

as provided in the ticket is a tortious


act.

(C) Yes, since the facts show a


breach of contract, not a quasi-

delict.

(D) Yes, since quasi-delict


presupposes the absence of a pre-

existing contractual relation


between the parties.

(6) Which of the following is an

indispensable requirement in an action for


"quieting of title" involving real property?

The plaintiff must

(A) be in actual possession of the


property.

(B) be the registered owner of the


property.

(C) have legal or equitable title to

the property.

(D) be the beneficial owner of the


property.

(7) X and Y were to marry in 3 months.


Meantime, to express his affection, X

donated a house and lot to Y, which


donation X wrote in a letter to Y. Y wrote

back, accepting the donation and took


possession of the property. Before the
the company of handsome boys. What legal
remedy does Lily have?
(A) No, since the marriage did not
(A) She can file an action for
take place.

(B) Yes, since all the requisites of a annulment of marriage on ground

donation of an immovable are of fraud.

present. (B) She can seek a declaration of

(C) No, since the donation and its nullity of the marriage based on

acceptance are not in a public Renes psychological incapacity.


(C) She can go abroad and file for
instrument.
divorce in a country that can grant
(D) Yes, since X freely donated the
it.
property to Y who became its owner.
(D) She has none since she had the

(8) Rene and Lily got married after a brief opportunity to examine the goods

courtship. After one month, Lily discovered and freely entered into the marriage.

that while Rene presented himself as a


(9) Lucio executed a simple deed of
macho man he was actually gay. He would
donation of P50 million on time deposit
not go to bed with her. He kept obscene
with a bank in favor of A, B, C, D, and E,
magazines of nude men and always sought

Never Let The Odds Keep You From Pursuing What You Know In Your Heart You Were Meant To Do.-
Leroy Satchel Paige
Page 151 of 180
Civil Law Q&As (2007-2013) hectorchristopher@yahoo.com dbaratbateladot@gmail.com

Raffys right only with his


conformity.
without indicating the share of each donee.
All the donees accepted the donation in
writing. A, one of the donees, died. Will B,

C, D, and E get As share in the money?

(A) Yes, accretion will automatically

apply to the joint-donees in equal


shares.

(B) Yes, since the donors intention

is to give the whole of P50 million to


the jointdonees in equal shares.

(C) No, A"s share will revert to the

donor because accretion applies

only if the joint-donees are spouses.

(D) No, As share goes to his heirs

since the donation did not

provide for reversion to donor.

(10) Raul, Ester, and Rufus inherited a 10-

hectare land from their father. Before the


land could be partitioned, however, Raul

sold his hereditary right to Raffy, a stranger


to the family, for P5 million. Do Ester and

Rufus have a remedy for keeping the land


within their family?

(A) Yes, they may be subrogated to

Raffys right by reimbursing to

him within the required time

what he paid Raul.

(B) Yes, they may be subrogated to

Raffys right provided they buy him


out before he registers the sale.

(C) No, they can be subrogated to


(A) Its cause comes from the guilt

(D) No, since there was no of a spouse in a legal separation

impediment to Raul selling his case, the innocent-spouse having

inheritance to a stranger. died.

(B) The truth of its cause is denied


(11) When one exercises a right recognized and not sufficiently proved by
by law, knowing that he thereby causes an evidence.
injustice to another, the latter is entitled to (C) Its cause is not authorized by
recover damages. This is known as the the law.
principle of (D) Its cause is not specified.
(A) res ipsa loquitur.
(B) damnum absque injuria. (13) Manuel came to Manila and married
(C) vicarious liability. Marianne. Unknown to Marianne, Manuel
(D) abuse of rights. had been previously convicted in Palawan of
theft and served time for it. After Marianne
(12) Which of the following is NOT a basis learned of his previous conviction, she
for rendering a disinheritance defective or stopped living with him. Can Marianne seek
imperfect? the annulment of the

Never Let The Odds Keep You From Pursuing What You Know In Your Heart You Were Meant To Do.-
Leroy Satchel Paige
Page 152 of 180
Civil Law Q&As (2007-2013) hectorchristopher@yahoo.com dbaratbateladot@gmail.com

(C) The children born before and


after the divorce are all legitimate
marriage based on Manuels nondisclosure
of his previous crime?

(A) No, since the assumption is that


marriage forgives all past wrongs.

(B) Yes, since the non-disclosure

of that crime is the equivalent of

fraud, which is a ground for

annulment.

(C) No, in case of doubt, the law


must be construed to preserve the
institution of marriage.

(D) No, since Manuel already served


the penalty for his crime.

(14) Arthur and Helen, both Filipinos, got

married and had 2 children. Arthur later


worked in Rome where he acquired Italian

citizenship. He got a divorce from Helen in


Rome but, on returning to the Philippines,

he realized his mistake, asked forgiveness of


his wife, and resumed living with her. They

had 2 more children. What is the status of


their 4 children?

(A) The children born before the

divorce are legitimate but those

born after it are not since Arthur

got the divorce when he had

ceased to be a Filipino.

(B) The divorce rendered illegitimate


the children born before it since the

marriage that begot them had been


nullified.
(16) The liability of the partners, including
industrial partners for partnership
since Philippine law does not
contracts entered into in its name and for
recognize divorce.
its account, when all partnership assets
(D) All the children are legitimate
have been exhausted is
since they were born of the same
(A) Pro-rata.
father and mother.
(B) Joint.

(15) Who can make a donation? (C) Solidary.

(A) All persons who can enter into (D) Voluntary.

contracts and dispose of their


(17) When can a missing person who left
property.
someone to administer his property be
(B) All persons who are of legal age
declared an absentee by the court? When
and suffer from no civil interdiction.
he has been missing for
(C) All persons who can make a last
(A) 2 years from the receipt of the
will and testament.
last news about him.
(D) All persons, whether natural or
(B) 7 years from the receipt of the
artificial, who own property.
last news about him.

Never Let The Odds Keep You From Pursuing What You Know In Your Heart You Were Meant To Do.-
Leroy Satchel Paige
Page 153 of 180
Civil Law Q&As (2007-2013) hectorchristopher@yahoo.com dbaratbateladot@gmail.com

given the agent. The agent executed the


deed of absolute sale on behalf of his
(C) 10 years from the receipt of the
last news about him.

(D) 5 years from the receipt of the

last news about him.

(18) Which of the following claims against

the debtor enjoys preference over the others


with respect to his specific immovable

property and real rights?

(A) Unpaid price of real property


sold, upon the immovable property.

(B) Mortgage credits recorded in the


registry of property, upon the
mortgaged real estate.

(C) Taxes due, upon the land or

building.

(D) Expenses for the preservation

and improvement of property, when


the law authorizes reimbursement,

upon the preserved or improved


immovable.

(19) When bilateral contracts are vitiated


with vices of consent, they are rendered
(A) rescissible.
(B) void.
(C) unenforceable.

(D) voidable.

(20) An agent, authorized by a special

power of attorney to sell a land belonging to


the principal succeeded in selling the same

to a buyer according to the instructions


claimed that it was exempt from execution,
being a family home. Is this claim correct?
principal two days after the principal died,
(A) Yes, because while Bs parents
an event that neither the agent nor the
own the land, they agreed to have
buyer knew at the time of the sale. What is
their daughter build her family
the standing of the sale?
home on it.
(A) Voidable.
(B) No, because there is no judicial
(B) Valid.
declaration that it is a family home.
(C) Void.
(D) Unenforceable. (C) No, since the land does not

belong to A and B, it cannot


(21) Spouses A and B leased a piece of land qualify as a family home.
belonging to B's parents for 25 years. The (D) Yes, because the A and Bs
spouses built their house on it worth family actually lives in that house.
P300,000.00. Subsequently, in a case that

C filed against A and B, the court found the (22) Solomon sold his coconut plantation to
latter liable to C for P200,000.00. When the Aragon, Inc. for P100 million, payable in
sheriff was attaching their house for the installments of P10 million per month with
satisfaction of the judgment, A and B 6% interest per annum. Solomon married

Never Let The Odds Keep You From Pursuing What You Know In Your Heart You Were Meant To Do.-
Leroy Satchel Paige
Page 154 of 180
Civil Law Q&As (2007-2013) hectorchristopher@yahoo.com dbaratbateladot@gmail.com

Lorna after 5 months and they chose (24) When A and B married, they chose

conjugal partnership of gains to govern conjugal partnership of gains to govern


their property relations. When they their property relations. After 3 years, B

married, Aragon had an unpaid balance of succeeded in getting her marriage to A

P50 million plus interest in Solomons favor. annulled on ground of the latters

To whom will Aragons monthly payments psychological incapacity. What liquidation

go after the marriage? procedure will they follow in disposing of

(A) The principal shall go to the their assets?

conjugal partnership but the (A) They will follow the rule
interests to Solomon. governing the liquidation of a
(B) Both principal and interests conjugal partnership of gains where
shall go to Solomon since they are the party who acted in bad faith
his exclusive properties. forfeits his share in the net profits.
(C) Both principal and interests (B) Since the marriage has been
shall go to the conjugal partnership declared void, the rule for
since these become due after the liquidation of absolute community of
marriage. property shall be followed.
(D) The principal shall go to (C) The liquidation of a co-
Solomon but the interests to the ownership applies since the
conjugal partnership. annulment brought their property

relation under the chapter on


(23) X and Y, although not suffering from
property regimes without
any impediment, cohabited as husband and
marriage.
wife without the benefit of marriage.
(D) The law on liquidation of
Following the birth of their child, the couple
partnerships applies.
got married. A year after, however, the court
(25) X and Y agreed verbally before their
annulled the marriage and issued a decree
marriage (a) on the paternity of the
of annulment. What is the present status of
illegitimate child of Y and (b) on the
the child?
economic regime that will govern X and Ys
(A) Legitimated.
property relations. Is the verbal agreement
(B) Illegitimate.
valid?
(C) Natural child.
(D) Legitimate.
(A) No, because a marriage

settlement to be valid should be

in writing.

Never Let The Odds Keep You From Pursuing What You Know In Your Heart You Were Meant To Do.-
Leroy Satchel Paige
Page 155 of 180
Civil Law Q&As (2007-2013) hectorchristopher@yahoo.com dbaratbateladot@gmail.com

Fidels.

(B) Yes, since ante-nuptial


agreements need not be in writing.

(C) No, because a marriage

settlement cannot include an


agreement on the paternity of an

illegitimate child.

(D) Yes, since even if it is not a valid

marriage settlement, it is a valid


verbal contract.

(26) Spouses X and Y have a minor


daughter, Z, who needs support for her

education. Both X and Y, who are


financially distressed, could not give the

needed support to Z. As it happens, Zs


other relatives are financially capable of

giving that support. From whom may Z first


rightfully demand support? From her

(A) grandfather.
(B) brother.
(C) uncle.
(D) first cousin.

(27) Fidel, a Filipino with fair complexion,

married Gloria. Before the marriage, Gloria


confessed to Fidel that she was two-month

pregnant with the child of a black African


who had left the country for good. When

the child was born, Fidel could not accept it


being too black in complexion. What is the

status of the child?

(A) Illegitimate, because Gloria


confessed that the child is not
(B) Illegitimate, because by the color (29) In his will, the testator designated X as

of its skin, the child could not a legatee to receive P2 million for the

possibly be that of Fidel. purpose of buying an ambulance that the

(C) Legitimate, because the child residents of his Barangay can use. What
kind of institution is this?
was born within a valid marriage.
(A) a fideicomissary institution.
(D) Legitimate, because Fidel agreed
(B) a modal institution.
to treat the child as his own after
(C) a conditional institution.
Gloria told him who the father was.
(D) a collective institution.

(28) The husbands acts of forcibly ejecting


(30) X insured himself for P5 million,
his wife without just cause from the
designating Y, his wife, as his sole
conjugal dwelling and refusing to take her
beneficiary. The designation was
back constitutes
irrevocable. A few years later, X had their
(A) desertion.
marriage annulled in court on the ground
(B) recrimination.
that Y had an existing prior marriage. X
(C) constructive abandonment.
(D) de facto separation.

Never Let The Odds Keep You From Pursuing What You Know In Your Heart You Were Meant To Do.-
Leroy Satchel Paige
Page 156 of 180
Civil Law Q&As (2007-2013) hectorchristopher@yahoo.com dbaratbateladot@gmail.com

(A) Marlon gets 1/4 and Cecilia gets


3/4.
subsequently died, Is Y entitled to the
(B) Marlon gets 2/3 and Cecilia 1/3.
insurance benefits?

(A) Yes, since the insurance was not

dependent on the marriage.

(B) Yes, since her designation as

beneficiary was irrevocable.

(C) No, Xs designation of Y is

revoked by operation of law upon

the annulment of their marriage

based on Ys fault.

(D) Yes, since without judicial

revocation, Xs designation of Y

remains valid and binding.

(31) May a spouse freely donate communal


or conjugal property without the consent of

the other?

(A) Absolutely not, since the spouses


co-own such property.

(B) Yes, for properties that the


family may spare, regardless of

value.

(C) Yes, provided the donation is

moderate and intended for charity

or family rejoicing.

(D) Yes, in a donation mortis causa


that the donor may still revoke in

his lifetime.

(32) The decedent died intestate leaving an

estate of P10 million. He left the following


heirs: a) Marlon, a legitimate child and b)

Cecilia, the legal spouse. Divide the estate.


(34) A buyer ordered 5,000 apples from the
seller at P20 per apple. The seller delivered
(C) Marlon gets 1/2 and Cecilia
6,000 apples. What are the rights and
gets 1/2.
obligations of the buyer?
(D) Marlon gets 3/4 and Cecilia 1/4.
(A) He can accept all 6,000 apples

(33) Contracts take effect only between the and pay the seller at P20 per

parties or their assigns and heirs, except apple.

where the rights and obligations arising (B) He can accept all 6,000 apples

from the contract are not transmissible by and pay a lesser price for the 1,000

their nature, by stipulation, or by provision excess apples.

of law. In the latter case, the assigns or the (C) He can keep the 6,000 apples

heirs are not bound by the contracts. This without paying for the 1,000 excess

is known as the principle of since the seller delivered them

(A) Relativity of contracts. anyway.

(B) Freedom to stipulate. (D) He can cancel the whole

(C) Mutuality of contracts. transaction since the seller violated


(D) Obligatory force of contracts. the terms of their agreement.

Never Let The Odds Keep You From Pursuing What You Know In Your Heart You Were Meant To Do.-
Leroy Satchel Paige
Page 157 of 180
Civil Law Q&As (2007-2013) hectorchristopher@yahoo.com dbaratbateladot@gmail.com

(37) Acme Cannery produced sardines in


cans known as "Sards." Mylene bought a
(35) Lino entered into a contract to sell with
can of Sards from a store, ate it, and
Ramon, undertaking to convey to the latter
one of the five lots he owns, without

specifying which lot it was, for the price of


P1 million. Later, the parties could not

agree which of five lots he owned Lino


undertook to sell to Ramon. What is the

standing of the contract?

(A) Unenforceable.
(B) Voidable.
(C) Rescissible.

(D) Void.

(36) Knowing that the car had a hidden

crack in the engine, X sold it to Y without


informing the latter about it. In any event,

the deed of sale expressly stipulated that X


was not liable for hidden defects. Does Y

have the right to demand from X a


reimbursement of what he spent to repair

the engine plus damages?

(A) Yes. X is liable whether or not he


was aware of the hidden defect.

(B) Yes, since the defect was not

hidden; X knew of it but he acted

in bad faith in not disclosing the

fact to Y.

(C) No, because Y is in estoppel,


having changed engine without prior
demand.

(D) No, because Y waived the


warranty against hidden defects.
and Mylene has not overcome such
presumption.
suffered from poisoning caused by a

noxious substance found in the sardines.


(38) Fernando executed a will, prohibiting
Mylene filed a case for damages against
his wife Marina from remarrying after his
Acme. Which of the following defenses will
death, at the pain of the legacy of P100
hold?
Million in her favor becoming a nullity. But
(A) The expiry date of the "Sards"
a year after Fernandos death, Marina was
was clearly printed on its can,
so overwhelmed with love that she married
still the store sold and Mylene
another man. Is she entitled to the legacy,
bought it. the amount of which is well within the
(B) Mylene must have detected the capacity of the disposable free portion of
noxious substance in the sardines Fernandos estate?
by smell, yet she still ate it. (A) Yes, since the prohibition against
(C) Acme had no transaction with remarrying is absolute, it is deemed
Mylene; she bought the "Sards" from not written.
a store, not directly from Acme. (B) Yes, because the prohibition is
(D) Acme enjoys the presumption of inhuman and oppressive and
safeness of its canning procedure

Never Let The Odds Keep You From Pursuing What You Know In Your Heart You Were Meant To Do.-
Leroy Satchel Paige
Page 158 of 180
Civil Law Q&As (2007-2013) hectorchristopher@yahoo.com dbaratbateladot@gmail.com

violates Marinas rights as a free (A) It is an ordinary donation


woman. since it was not given to the bride
(C) No, because the nullity of the
or groom.
prohibition also nullifies the legacy.
(B) It is donation propter nuptias
(D) No, since such prohibition is
since it was given with the marriage
authorized by law and is not in mind.
repressive; she could remarry but (C) It is an indirect donation propter
must give up the money. nuptias since the bride would
eventually inherit the property from
(39) X, the owner, constituted a 10-year her parents.
usufruct on his land as well as on the (D) It is a remunatory donation.
building standing on it in Ys favor. After
flood totally destroyed the building 5 years (41) X and Y, both Filipinos, were married

later, X told Y that an act of God terminated and resided in Spain although they intend

the usufruct and that he should vacate the to return to the Philippines at some future

land. Is X, the owner of the land, correct? time. They have not executed any marriage

(A) No, since the building was settlements. What law governs their

destroyed through no fault of Y. property relations?

(B) No, since Y still has the right (A) They may choose between

to use the land and the materials Spanish law and Philippine law.

left on it. (B) Philippine law since they are

(C) Yes, since Y cannot use the land both Filipinos.

without the building. (C) No regime of property relations

(D) Yes, since the destruction of the will apply to them.

building without the Xs fault (D) Spanish law since they live in

terminated the usufruct. Spain.

(40) In gratitude, the grooms parents made (42) Birth determines personality. Death

a donation of a property in writing to the extinguishes it. Under what circumstances

brides parents shortly before their may the personality of a deceased person

childrens wedding. The donation was continue to exist?

accepted. What is the nature of the (A) In case of re-appearance of a

donation? missing person presumed dead.


Never Let The Odds Keep You From Pursuing What You Know In Your Heart You Were Meant To Do.-
Leroy Satchel Paige
Page 159 of 180
Civil Law Q&As (2007-2013) hectorchristopher@yahoo.com dbaratbateladot@gmail.com

(A) the brother or sister who needs


support lives in another place.
(B) In protecting the works of a
(B) such brothers and sisters are not
deceased under intellectual property
recognized by their father.
laws.

(C) In case of declaration of


presumptive death of a missing

spouse.

(D) In the settlement of the estate

of a deceased person.

(43) Six tenants sued X, the landowner, for


willfully denying them water for their farms,

which water happened to flow from land


under Xs control, his intention being to

force them to leave his properties. Is X


liable for his act and why?

(A) No, because the tenants must be


content with waiting for rainfall for
their farms.

(B) No, since X owns both the land


and the water.

(C) Yes, because the tenants farms


have the natural right of access to
water wherever it is located.

(D) Yes, since X willfully caused

injury to his tenants contrary to

morals, good customs or public

policy.

(44) Illegitimate brothers and sisters,


whether of full or half-blood, are bound to

support each other, EXCEPT when


(A) No, the child was 7 years old and
knew the dangers that the pool
(C) the brother or sister in need
offered.
stops schooling without valid
(B) Yes, being an attractive
reason.
nuisance, Virgilio had the duty to
(D) the need for support of a
prevent children from coming near
brother or sister, already of age, is
it.
due to the latter's fault.
(C) No, since the pool was bare

and had no enticing or alluring


(45) Virgilio owned a bare and simple
swimming pool in his garden. MB, a 7-year gadgets, floats, or devices in it

old child, surreptitiously entered the garden that would attract a 7-year old

and merrily romped around the ledges of child.

the pool. He accidentally tripped, fell into (D) Yes, since Virgilio did not cover
the pool, and drowned. MBs parents sued the swimming pool while not in use
Virgilio for damages arising from their to prevent children from falling into
childs death, premised on the principle of it.
"attractive nuisance". Is Virgilio liable for
the death of MB?

Never Let The Odds Keep You From Pursuing What You Know In Your Heart You Were Meant To Do.-
Leroy Satchel Paige
Page 160 of 180
Civil Law Q&As (2007-2013) hectorchristopher@yahoo.com dbaratbateladot@gmail.com

(B) No, but Rex is entitled to recover


the value of the land from the
(46) The term of a 5-year lease contract
municipality.
between X the lessor and Y the lessee,
(C) No, the transfer of ownership has
where rents were paid from month to
been completed.
month, came to an end. Still, Y continued
using the property with Xs consent. In

such a case, it is understood that they


impliedly renewed the lease

(A) from month to month under

the same conditions as to the

rest.

(B) under the same terms and


conditions as before.

(C) under the same terms except the


rent which they or the court must
fix.

(D) for only a year, with the rent


raised by 10% pursuant to the
rental control law.

(47) Rex, a philanthropist, donated a

valuable lot to the municipality on the

condition that it will build a public school on

such lot within 2 years from its acceptance of

the donation. The municipality properly

accepted the donation but did not yet build

the public school after 2 years. Can Rex

revoke the donation?

(A) Yes, since the donation is

subject to a resolutory condition

which was not fulfilled.


Without his consent, his friend Boyong paid
the whole loan. Since Asiong benefited from
(D) Yes, the donation is not deemed
the payment, can Boyong compel the bank
made until the suspensive condition
to subrogate him in its right as mortgagee
has been fulfilled.
of Asiong's land?

(48) Illegitimate children, those not (A) No, but the bank can foreclose

recognized by their biological fathers, shall and pay Boyong back.

use the surname of their (B) No, since Boyong paid for

(A) biological father subject to no Asiongs loan without his

condition. approval.

(B) mother or biological father, at (C) Yes, since a change of creditor

the mothers discretion. took place by novation with the

(C) mother. banks consent.

(D) biological father unless he (D) Yes, since it is but right that

judicially opposes it. Boyong be able to get back his


money and, if not, to foreclose the

(49) Asiong borrowed P1 million from a mortgage in the manner of the

bank, secured by a mortgage on his land. bank.

Never Let The Odds Keep You From Pursuing What You Know In Your Heart You Were Meant To Do.-
Leroy Satchel Paige
Page 161 of 180
Civil Law Q&As (2007-2013) hectorchristopher@yahoo.com dbaratbateladot@gmail.com

(C) Yes, since the payment covers


the whole obligation.
(50) Congress passed a law imposing taxes

on income earned out of a particular


activity that was not previously taxed. The

law, however, taxed incomes already earned


within the fiscal year when the law took

effect. Is the law valid?

(A) No, because laws are intended to


be prospective, not retroactive.

(B) No, the law is arbitrary in that it


taxes income that has already been
spent.

(C) Yes, since tax laws are the


lifeblood of the nation.

(D) Yes, tax laws are an exception;

they can be given retroactive

effect.

(51) Rudolf borrowed P1 million from

Rodrigo and Fernando who acted as


solidary creditors. When the loan matured,

Rodrigo wrote a letter to Rudolf, demanding


payment of the loan directly to him. Before

Rudolf could comply, Fernando went to see


him personally to collect and he paid him.

Did Rudolf make a valid payment?

(A) No, since Rudolf should have

split the payment between Rodrigo


and Fernando.

(B) No, since Rodrigo, the other

solidary creditor, already made a

prior demand for payment from

Rudolf.
governed by the regime of absolute
community of property.
(D) Yes, since Fernando was a
(D) They are superseded by the
solidary creditor, payment to him
Family Code which has retroactive
extinguished the obligation.
effect.

(52) What happens to the property regimes (53) The testator executed a will following

that were subsisting under the New Civil the formalities required by the law on

Code when the Family Code took effect? succession without designating any heir.

(A) The original property regimes The only testamentary disposition in the

will is the recognition of the testator's


are immutable and remain
illegitimate child with a popular actress. Is
effective.
the will valid?
(B) Those enjoying specific regimes
(A) Yes, since in recognizing his
under the New Civil Code may adopt
illegitimate child, the testator has
the regime of absolute community of
made him his heir.
property under the Family Code.
(B) No, because the non-designation
(C) Those that married under the
of heirs defeats the purpose of a will.
New Civil Code but did not choose

any of its regimes shall now be

Never Let The Odds Keep You From Pursuing What You Know In Your Heart You Were Meant To Do.-
Leroy Satchel Paige
Page 162 of 180
Civil Law Q&As (2007-2013) hectorchristopher@yahoo.com dbaratbateladot@gmail.com

(B) Void, because the couple did not

get local permit for a beach wedding.


(C) No, the will comes to life only
when the proper heirs are instituted.

(D) Yes, the recognition of an

illegitimate heir is an ample

reason for a will.

(54) A left B, his wife, in the Philippines to


work in Egypt but died in that country after

a years continuous stay. Two months after


As death, B gave birth to a child, claiming

it is As child. Who can assail the legitimacy


of the child?

(A) As other heirs apart from B.

(B) The State which has interest in

the welfare of overseas contract

workers.

(C) Any one who is outraged by Bs

claim.

(D) No one since A died.

(55) QR and TS who had a marriage license


requested a newly appointed Judge in
Manila to marry them on the beach of

Boracay. Since the Judge maintained


Boracay as his residence, he agreed. The

sponsors were all public officials. What is


the status of the marriage.

(A) Valid, since the improper

venue is merely an irregularity;

all the elements of a valid

marriage are present.


Y, came to the Philippines and married Z.
Can X be held liable for bigamy?
(C) Voidable, because the Judge
(A) No since Xs marriage to Y is void
acted beyond his territorial
ab initio or did not exist.
jurisdiction and is administratively

liable for the same. (B) No since X acted in good faith,

(D) Void, because the Judge did not conscious that public policy did not

solemnize the marriage within the approve of marriage between first

premises of his court. cousins.

(C) Yes since he married Z without

(56) X and Y, Filipinos, got married in Los first securing a judicial


Angeles, USA, using a marriage license declaration of nullity of his
issued by the Philippine consul in Los marriage to Y.
Angeles, acting as Civil Registrar. X and Y
(D) Yes since his first marriage to Y
did not know that they were first cousins
in Los Angeles is valid.
because their mothers, who were sisters,
were separated when they were quite (57) Allan bought Billys property through
young. Since X did not want to continue Carlos, an agent empowered with a special
with the relation when he heard of it, he left power of attorney (SPA) to sell the same.

Never Let The Odds Keep You From Pursuing What You Know In Your Heart You Were Meant To Do.-
Leroy Satchel Paige
Page 163 of 180
Civil Law Q&As (2007-2013) hectorchristopher@yahoo.com dbaratbateladot@gmail.com

consumables, rice can easily

When Allan was ready to pay as scheduled, deteriorate.

Billy called, directing Allan to pay directly to


him. On learning of this, Carlos, Billy's

agent, told Allan to pay through him as his


SPA provided and to protect his

commission. Faced with two claimants,


Allan consigned the payment in court. Billy

protested, contending that the consignation


is ineffective since no tender of payment

was made to him. Is he correct?

(A) No, since consignation without

tender of payment is allowed in

the face of the conflicting claims

on the plaintiff.

(B) Yes, as owner of the property


sold, Billy can demand payment

directly to himself.

(C) Yes, since Allan made no


announcement of the tender.

(D) Yes, a tender of payment is


required for a valid consignation.

(58) X sold Y 100 sacks of rice that Y was to


pick up from Xs rice mill on a particular

date. Y did not, however, appear on the


agreed date to take delivery of the rice.

After one week, X automatically rescinded


the sale without notarial notice to Y. Is the

rescission valid?

(A) Yes, automatic rescission is

allowed since, having the

character of movables and


(A) No, efforts at a compromise will
only deepen the wifes anguish.
(B) No, the buyer is entitled to a
(B) No, since legal separation like
customary 30-day extension of his
obligation to take delivery of the validity of marriage is not subject

goods. to compromise agreement for

(C) No, since there was no express purposes of filing.


agreement regarding automatic (C) Yes, to avoid a family feud that is
rescission. hurtful to everyone.
(D) No, the seller should first (D) Yes, since the dispute could have
determine that Y was not justified in been settled with the parties
failing to appear. agreeing to legal separation.
(59) The wife filed a case of legal separation
against her husband on the ground of (60) An Australian living in the Philippines

sexual infidelity acquired shares of stock worth P10 million

without previously exerting earnest efforts in food manufacturing companies. He died

to come to a compromise with him. The in Manila, leaving a legal wife and a child in

judge dismissed the case for having been Australia and a live-in partner with whom

filed without complying with a condition


precedent. Is the dismissal proper?

Never Let The Odds Keep You From Pursuing What You Know In Your Heart You Were Meant To Do.-
Leroy Satchel Paige
Page 164 of 180
Civil Law Q&As (2007-2013) hectorchristopher@yahoo.com dbaratbateladot@gmail.com

(B) No, since the sale cannot under


the Statute of Frauds be enforced.
he had two children in Manila. He also left
(C) Yes, since X bought the land and
a will, done according to Philippine laws,
paid Y for it.
leaving all his properties to his live-in

partner and their children. What law will


govern the validity of the disposition in the

will?

(A) Australia law since his legal wife

and legitimate child are Australians

and domiciled in Australia.

(B) Australian law since the

intrinsic validity of the provisions

of a will is governed by the

decedents national law.

(C) Philippine law since the decedent

died in Manila and he executed his

will according to such law.

(D) Philippine law since the

decedents properties are in the

Philippines.

(61) X bought a land from Y, paying him

cash. Since they were friends, they did not

execute any document of sale. After 7 years,

the heirs of X asked Y to execute a deed of

absolute sale to formalize the verbal sale to

their father. Unwilling to do so, Xs heirs filed

an action for specific performance against Y.

Will their action prosper?

(A) No, after more than 6 years,

the action to enforce the verbal

agreement has already elapsed.


proceedings to a new

(D) Yes, after full payment, the administrator whom it will

action became imprescriptible. appoint.

(C) Cicero automatically becomes


(62) A court declared Ricardo, an old administrator of Ricardos estate
bachelor, an absentee and appointed Cicero until judicially relieved.
administrator of his property. After a year, it (D) Ciceros alienations of Ricardo's
was discovered that Ricardo had died property will be set aside.
abroad. What is the effect of the fact of his
death on the administration of his (63) Baldo, a rejected suitor, intimidated
property? Judy into marrying him. While she wanted
(A) With Ricardo no longer an to question the validity of their marriage
absentee but a deceased person, two years after the intimidation ceased,
Cicero will cease to be administrator Judy decided in the meantime to freely
of his properties. cohabit with Baldo. After more than 5 years

(B) The administration shall be following their wedding, Judy wants to file a

given by the court having case for annulment of marriage against

jurisdiction over the intestate

Never Let The Odds Keep You From Pursuing What You Know In Your Heart You Were Meant To Do.-
Leroy Satchel Paige
Page 165 of 180
Civil Law Q&As (2007-2013) hectorchristopher@yahoo.com dbaratbateladot@gmail.com

(65) In the order of intestate succession


where the decedent is legitimate, who is the
Baldo on ground of lack of consent. Will her
last intestate heirs or heir who will inherit if
action prosper?

(A) Yes, the action for annulment is


imprescriptible.

(B) No, since the marriage was

merely voidable and Judy ratified

it by freely cohabiting with Baldo

after the force and intimidation

had ceased.

(C) No, since the action prescribed 5


years from the date of the
celebration of the marriage.

(D) Yes, because the marriage was


celebrated without Judy's consent
freely given.

(64) Is the wife who leaves her husband


without just cause entitled to support?

(A) No, because the wife must

always be submissive and respectful


to the husband.

(B) Yes. The marriage not having

been dissolved, the husband


continues to have an obligation to

support his wife.

(C) No, because in leaving the

conjugal home without just cause,

she forfeits her right to support.

(D) Yes, since the right to receive

support is not subject to any


condition.
(A) Neither solidary nor joint since

they cannot waive the defense of


all heirs in the higher level are disqualified
fortuitous event to which they are
or unable to inherit?
entitled.
(A) Nephews and nieces.
(B) Brothers and sisters. (B) Solidary or joint upon the

(C) State. discretion of Sam.

(D) Other collateral relatives up to (C) Solidary since Roy and Carlos

the 5th degree of consanguinity. failed to perform their obligation to


deliver the motor boat.

(66) Roy and Carlos both undertook a (D) Joint since the conversion of

contract to deliver to Sam in Manila a boat their liability to one of indemnity


docked in Subic. Before they could deliver for damages made it joint.
it, however, the boat sank in a storm. The

contract provides that fortuitous event shall (67) Joanne married James, a person with
not exempt Roy and Carlos from their no known relatives. Through James' hard
obligation. Owing to the loss of the motor work, he and his wife Joane prospered.
boat, such obligation is deemed converted When James died, his estate alone

into one of indemnity for damages. Is the


liability of Roy and Carlos joint or solidary?

Never Let The Odds Keep You From Pursuing What You Know In Your Heart You Were Meant To Do.-
Leroy Satchel Paige
Page 166 of 180
Civil Law Q&As (2007-2013) hectorchristopher@yahoo.com dbaratbateladot@gmail.com

(D) When the precedent has

amounted to P100 million. If, in his will, ceased to be beneficial and useful.

James designates Joanne as his only heir,


what will be the free portion of his estate.

(A) Joanne gets all; estate has no


free portion left.

(B) Joanne gets 1/2; the other

half is free portion.

(C) Joanne gets 1/3; the remaining


2/3 is free portion.

(D) Joanne gets 1/4; the remaining


3/4 is free portion.

(68) A warranty inherent in a contract of


sale, whether or not mentioned in it, is
known as the

(A) warranty on quality.


(B) warranty against hidden defects.

(C) warranty against eviction.


(D) warranty in merchantability.

(69) The doctrine of stare decisis prescribes

adherence to precedents in order to


promote the stability of the law. But the

doctrine can be abandoned

(A) When adherence to it would


result in the Governments loss of its

case.

(B) When the application of the


doctrine would cause great
prejudice to a foreign national.

(C) When necessary to promote the


passage of a new law.
(C) Yes, since they executed their

joint will out of mutual love and


(70) Ric and Josie, Filipinos, have been
care, values that the generally
sweethearts for 5 years. While working in a
accepted principles of international
European country where the execution of
law accepts.
joint wills are allowed, the two of them
(D) Yes, since it is valid in the
executed a joint holographic will where they
country where it was executed,
named each other as sole heir of the other
applying the principle of "lex loci
in case either of them dies. Unfortunately,
celebrationis."
Ric died a year later. Can Josie have the
joint will successfully probated in the
(71) ML inherited from his father P5 million
Philippines?
in legitime but he waived it in a public
(A) Yes, in the highest interest of
instrument in favor of his sister QY who
comity of nations and to honor the
accepted the waiver in writing. But as it
wishes of the deceased.
happened, ML borrowed P6 million from PF
(B) No, since Philippine law
before the waiver. PF objected to the waiver
prohibits the execution of joint
and filed an action for its rescission on the
wills and such law is binding on
ground that he had the right to MLs P5
Ric and Josie even abroad.

Never Let The Odds Keep You From Pursuing What You Know In Your Heart You Were Meant To Do.-
Leroy Satchel Paige
Page 167 of 180
Civil Law Q&As (2007-2013) hectorchristopher@yahoo.com dbaratbateladot@gmail.com

is essentially void for containing an


unrelated matter.
million legitime as partial settlement of

what ML owed him since ML has proved to


be insolvent. Does PF, as creditor, have the

right to rescind the waiver?

(A) No, because the waiver in favor

of his sister QY amounts to a


donation and she already accepted

it.

(B) Yes, because the waiver is

prejudicial to the interest of a

third person whose interest is

recognized by law.

(C) No, PF must wait for ML to


become solvent and, thereafter, sue
him for the unpaid loan.

(D) Yes, because a legitime cannot

be waived in favor of a specific heir;


it must be divided among all the

other heirs.

(72) While engaged to be married, Arnold


and Josephine agreed in a public

instrument to adopt out the economic


regime of absolute community of property.

Arnold acknowledged in the same


instrument that Josephines daughter

Mary, is his illegitimate child. But


Josephine died before the marriage could

take place. Does the marriage settlement


have any significance?

(A) None, since the instrument


containing the marriage settlement
Philippines and married Leonora. What is
the status of this second marriage?
(B) Yes, insofar as Arnold
(A) Void, because he did not cause
acknowledged Mary as his
the judicial issuance of
illegitimate child.
declaration of the nullity of his
(C) None, since the marriage did not
first marriage to Jenny before
take place.
marrying Leonora.
(D) Yes, if they acquired properties

while living together as husband (B) Valid, because Joseph's marriage

and wife. to Jenny is void, he being only 17

(73) Joseph, a 17-year old Filipino, married years of age when he married her.

Jenny, a 21-year old American in Illinois, (C) Valid, because his marriage to

USA, where the marriage was valid. Their Leonora has all the elements of a

parents gave full consent to the marriage of valid marriage.

their children. After three years, Joseph (D) Void, because Joseph is still

filed a petition in the USA to promptly considered married to Jenny since

divorce Jenny and this was granted. When the Philippines does not recognize

Joseph turned 25 years, he returned to the divorce.

Never Let The Odds Keep You From Pursuing What You Know In Your Heart You Were Meant To Do.-
Leroy Satchel Paige
Page 168 of 180
Civil Law Q&As (2007-2013) hectorchristopher@yahoo.com dbaratbateladot@gmail.com

(C) unless the court first directs

mediation of the parties.


(74) T died intestate, leaving an estate of

P9,000,000. He left as heirs three legitimate (D) without prior investigation

children, namely, A, B, and C. A has two conducted by a public prosecutor.

children, D and E. Before he died, A


irrevocably repudiated his inheritance from

T in a public instrument filed with the


court. How much, if any, will D and E, as

As children, get from Ts estate?

(A) Each of D and E will get

P1,500,000 by right of
representation since their father

repudiated his inheritance.

(B) Each of D and E will get

P2,225,000 because they will inherit


from the estate equally with B and

C.

(C) D and E will get none because

of the repudiation; "B" and "C"

will get As share by right of

accretion.

(D) Each of D and E will get

P2,000,000 because the law gives

them some advantage due to the


demise of "A".

(75) No decree of legal separation can be


issued

(A) unless the childrens welfare is

attended to first.

(B) without prior efforts at

reconciliation shown to be futile.


with the title needed to effect the
sale.
(76) X, who was abroad, phoned his
(D) Valid, since the buyer could file
brother, Y, authorizing him to sell Xs parcel
an action to compel X to execute a
of land in Pasay. X sent the title to Y by
deed of sale.
courier service. Acting for his brother, Y
executed a notarized deed of absolute sale
(77) In a true pacto de retro sale, the title
of the land to Z after receiving payment.
and ownership of the property sold are
What is the status of the sale?
immediately vested in the vendee a retro
(A) Valid, since a notarized deed of
subject only to the resolutory condition of
absolute sale covered the
repurchase by the vendor a retro within the
transaction and full payment was
stipulated period. This is known as
made.
(A) equitable mortgage.
(B) Void, since X should have
(B) conventional redemption.
authorized agent Y in writing to
(C) legal redemption.
sell the land.
(D) equity of redemption.
(C) Valid, since Y was truly his (78) A natural obligation under the New
brother Xs agent and entrusted Civil Code of the Philippines is one which

Never Let The Odds Keep You From Pursuing What You Know In Your Heart You Were Meant To Do.-
Leroy Satchel Paige
Page 169 of 180
Civil Law Q&As (2007-2013) hectorchristopher@yahoo.com dbaratbateladot@gmail.com

only upon the wifes acceptance

(A) the obligor has a moral or the courts authorization.

obligation to do, otherwise entitling


the obligee to damages.

(B) refers to an obligation in writing

to do or not to do.

(C) the obligee may enforce through

the court if violated by the obligor.

(D) cannot be judicially enforced

but authorizes the obligee to

retain the obligors payment or

performance.

(79) The husband assumed sole


administration of the familys mango
plantation since his wife worked abroad.

Subsequently, without his wifes knowledge,

the husband entered into an antichretic


transaction with a company, giving it

possession and management of the


plantation with power to harvest and sell

the fruits and to apply the proceeds to the


payment of a loan he got. What is the

standing of the contract?

(A) It is void in the absence of the

wifes consent.

(B) It is void absent an authorization

from the court.

(C) The transaction is void and can

neither be ratified by the wife nor


authorized by the court.

(D) It is considered a continuing

offer by the parties, perfected


(81) Anne owed Bessy P1 million due on
October 1, 2011 but failed to pay her on
(80) When the donor gives donations
due date. Bessy sent a demand letter to
without reserving sufficient funds for his
Anne giving her 5 days from receipt within
support or for the support of his
which to pay. Two days after receipt of the
dependents, his donations are
letter, Anne personally offered to pay Bessy
(A) Rescissible, since it results in
in manager's check but the latter refused to
economic lesion of more than 25%
accept the same. The 5 days lapsed. May
of the value of his properties.
Annes obligation be considered
(B) Voidable, since his consent to
extinguished?
the donation is vitiated by mindless
(A) Yes, since Bessys refusal of the
kindness.
managers check, which is presumed
(C) Void, since it amounts to wanton
funded, amounts to a satisfaction of
expenditure beyond his means.
the obligation.
(D) Reducible to the extent that
(B) No, since tender of payment
the donations impaired the
even in cash, if refused, will not
support due to himself and his
discharge the obligation without
dependents.
proper consignation in court.

Never Let The Odds Keep You From Pursuing What You Know In Your Heart You Were Meant To Do.-
Leroy Satchel Paige
Page 170 of 180
Civil Law Q&As (2007-2013) hectorchristopher@yahoo.com dbaratbateladot@gmail.com

(C) Yes, since Anne tendered (83) The owner of a thing cannot use it in a
payment of the full amount due. way that will injure the right of a third
(D) No, since a managers check is person. Thus, every building or land is
not considered legal tender in the subject to the easement which prohibits its
Philippines. proprietor or possessor from committing

nuisance like noise, jarring, offensive odor,


(82) The residents of a subdivision have and smoke. This principle is known as
been using an open strip of land as passage (A) Jus vindicandi.
to the highway for over 30 years. The owner (B) Sic utere tuo ut alienum non
of that land decided, however, to close it in laedas.
preparation for building his house on it. (C) Jus dispondendi.
The residents protested, claiming that they (D) Jus abutendi.
became owners of the land through
acquisitive prescription, having been in (84) Janice and Jennifer are sisters. Janice
possession of the same in the concept of sued Jennifer and Laura, Jennifers
owners, publicly, peacefully, and business partner for recovery of property
continuously for more than 30 years. Is this with damages. The complaint did not allege
claim correct? that Janice exerted earnest efforts to come

(A) No, the residents have not to a compromise with the defendants and

been in continuous possession of that such efforts failed. The judge

the land since they merely passed dismissed the complaint outright for failure

to comply with a condition precedent. Is the


through it in going to the
dismissal in order?
highway.
(A) No, since Laura is a stranger to
(B) No, the owner did not abandon
the sisters, Janice has no moral
his right to the property; he merely
tolerated his neighbors use of it for obligation to settle with her.

passage. (B) Yes, since court should promote

(C) Yes, residents of the subdivision amicable settlement among

have become owners by acquisitive relatives.

prescription. (C) Yes, since members of the same

(D) Yes, community ownership by family, as parties to the suit, are

prescription prevails over private required to exert earnest efforts to

claims.
settle their disputes before coming
to court.

Never Let The Odds Keep You From Pursuing What You Know In Your Heart You Were Meant To Do.-
Leroy Satchel Paige
Page 171 of 180
Civil Law Q&As (2007-2013) hectorchristopher@yahoo.com dbaratbateladot@gmail.com

(D) No, the family council, which (87) When does the regime of conjugal

would ordinarily mediate the partnership of gains begin to exist?

dispute, has been eliminated under (A) At the moment the parties
the Family Code. take and declare each other as

husband and wife before


(85) X borrowed money from a bank,
officiating officer.
secured by a mortgage on the land of Y, his
(B) At the time the spouses acquire
close friend. When the loan matured, Y
properties through joint efforts.
offered to pay the bank but it refused since
(C) On the date the future spouses
Y was not the borrower. Is the banks action
executed their marriage settlements
correct?
because this is the starting point of
(A) Yes, since X, the true borrower,
their marital relationship.
did not give his consent to Ys offer
(D) On the date agreed upon by the
to pay.
future spouses in their marriage
(B) No, since anybody can discharge
settlements since their agreement is
Xs obligation to his benefit.
the law between them.
(C) No, since Y, the owner of the

collateral, has an interest in the


(88) Josie, 18, married Dante, 25, without
payment of the obligation.
her parents knowledge and consent, and
(D) Yes, since it was X who has an
lived with him. After a year, Josie returned
obligation to the bank.
to her parents home, complained of the

unbearable battering she was getting from


(86) The right of a mortgagor in a judicial
Dante, and expressed a desire to have her
foreclosure to redeem the mortgaged
marriage with him annulled. Who may
property after his default in the
bring the action?
performance of the conditions of the
(A) Dante.
mortgage but before the sale of the
(B) Her parents.
mortgaged property or confirmation of the
(C) Josie herself.
sale by the court, is known as
(D) The State.
(A) accion publiciana.

(B) equity of redemption.


(89) X, a married man, cohabited with Y, an
(C) pacto de retro.
unmarried woman. Their relation bore them
(D) right of redemption.
BB, a baby boy. Subsequently, after X
became a widower, he married Y. Was BB

legitimated by that marriage?


Never Let The Odds Keep You From Pursuing What You Know In Your Heart You Were Meant To Do.-
Leroy Satchel Paige
Page 172 of 180
Civil Law Q&As (2007-2013) hectorchristopher@yahoo.com dbaratbateladot@gmail.com

(A) Yes, since his parents are now (D) Yes, as long as they leave
lawfully married. sufficient property for themselves
(B) Yes, since he is an innocent and for their dependents.

party and the marriage rectified the


wrong done him. (92) X owed Y P1.5 million. In his will, X

(C) No, since once illegitimate, a gave Y legacy of P1 million but the will

child shall always remain provided that this legacy is to be set off

illegitimate. against the P1.5 million X owed Y. After the

(D) No, since his parents were not set off, X still owed Y P500,000. Can Y still

qualified to marry each other collect this amount?

(A) Yes, because the designation


when he was conceived.
of Y as legatee created a new and

(90) The presence of a vice of consent separate juridical relationship

vitiates the consent of a party in a contract between them, that of testator-


and this renders the contract legatee.
(A) Rescissible. (B) It depends upon the discretion of
(B) Unenforceable. the probate court if a claim is filed
(C) Voidable. in the testate proceedings.
(D) Void.
(C) No, because the intention of the
testator in giving the legacy is to
(91) Can common-law spouses donate
abrogate his entire obligation to Y.
properties of substantial value to one
(D) No, because X had no
another?
instruction in his will to deliver
(A) No, they are only allowed to
more than the legacy of P1 million to
give moderate gifts to each other
Y.
during family rejoicing.

(B) No, they cannot give anything of (93) Josie owned a lot worth P5 million
value to each other to prevent prior to her marriage to Rey. Subsequently,
placing their legitimate relatives at a their conjugal partnership spent P3 million
disadvantage. for the construction of a house on the lot.
(C) Yes, unlike the case of legally The construction resulted in an increase in
married spouses, such donations the value of the house and lot to P9 million.
are not prohibited. Who owns the house and the lot?
Never Let The Odds Keep You From Pursuing What You Know In Your Heart You Were Meant To Do.-
Leroy Satchel Paige
Page 173 of 180
Civil Law Q&As (2007-2013) hectorchristopher@yahoo.com dbaratbateladot@gmail.com

(B) When the title holder concealed


the matter from the plaintiff.
(A) Josie and the conjugal

partnership of gains will own both


on a 50-50 basis.

(B) Josie will own both since the

value of the house and the

increase in the propertys value is

less than her lots value; but she

is to reimburse conjugal

partnership expenses.

(C) Josie still owns the lot, it being

her exclusive property, but the

house belongs to the conjugal


partnership.

(D) The house and lot shall both

belong to the conjugal partnership,

with Josie entitled to reimbursement

for the value of the lot.

(94) An action for reconveyance of a


registered piece of land may be brought

against the owner appearing on the title


based on a claim that the latter merely

holds such title in trust for the plaintiff.


The action prescribes, however, within 10

years from the registration of the deed or


the date of the issuance of the certificate of

title of the property as long as the trust had


not been repudiated. What is the exception

to this 10-year prescriptive period?

(A) When the plaintiff had no notice


of the deed or the issuance of the

certificate of title.
marriage after they became 21 of

(C) When fortuitous circumstances age; their cohabitation also

prevented the plaintiff from filing convalidated the marriage.

the case sooner. (B) No, since Lindas parents made

(D) When the plaintiff is in no allegations that earnest efforts

possession of the property. have been made to come to a


compromise with Conrad and Linda

(95) Conrad and Linda, both 20 years old, and which efforts failed.

applied for a marriage license, making it (C) Yes, since the marriage is

appear that they were over 25. They voidable, the couple being below 21

married without their parents knowledge years of age when they married.

before an unsuspecting judge. After the (D) Yes, since Lindas parents never

couple has been in cohabitation for 6 years, gave their consent to the marriage.

Lindas parents filed an action to annul the


marriage on ground of lack of parental (96) Pepito executed a will that he and 3

consent. Will the case prosper? attesting witnesses signed following the

(A) No, since only the couple can formalities of law, except that the Notary

Public failed to come. Two days later, the


question the validity of their

Never Let The Odds Keep You From Pursuing What You Know In Your Heart You Were Meant To Do.-
Leroy Satchel Paige
Page 174 of 180
Civil Law Q&As (2007-2013) hectorchristopher@yahoo.com dbaratbateladot@gmail.com

(A) Conjugal since they earned the


same while living as husband and
Notary Public notarized the will in his law
wife.
office where all signatories to the will
acknowledged that the testator signed the

will in the presence of the witnesses and


that the latter themselves signed the will in

the presence of the testator and of one


another. Was the will validly notarized?

(A) No, since it was not notarized on

the occasion when the signatories


affixed their signatures on the will.

(B) Yes, since the Notary Public

has to be present only when the

signatories acknowledged the acts

required of them in relation to

the will.

(C) Yes, but the defect in the mere


notarization of the will is not fatal to
its execution.

(D) No, since the notary public did

not require the signatories to sign


their respective attestations again.

(97) Venecio and Ester lived as common-law


spouses since both have been married to

other persons from whom they had been


separated in fact for several years.

Hardworking and bright, each earned


incomes from their respective professions

and enterprises. What is the nature of their


incomes?
(C) 4 years from the perfection of the
donation.
(B) Separate since their property
(D) Such action does not prescribe.
relations with their legal spouses

are still subsisting.


(99) Before Karen married Karl, she
(C) Co-ownership since they agreed inherited P5 million from her deceased
to work for their mutual benefit.
mother which amount she brought into the
(D) Communal since they earned the marriage. She later used part of the money
same as common-law spouses.
to buy a new Mercedes Benz in her name,

which Karen and her husband used as a


(98) What is the prescriptive period for filing
family car. Is the car a conjugal or Karens
an action for revocation of a donation based
exclusive property?
on acts of ingratitude of the donee?
(A) It is conjugal property since the
(A) 5 years from the perfection of the
spouses use it as a family car.
donation.
(B) It is Karens exclusive property
(B) 1 year from the perfection of
since it is in her name.
the donation.
(C) It is conjugal property having
been bought during the marriage.

Never Let The Odds Keep You From Pursuing What You Know In Your Heart You Were Meant To Do.-
Leroy Satchel Paige
Page 175 of 180
Civil Law Q&As (2007-2013) hectorchristopher@yahoo.com dbaratbateladot@gmail.com

stairway eventually gave way and collapsed,


resulting to injuries to some lessees. Who
(D) It is Karens exclusive

property since she bought it with

her own money.

(100) Because of Xs gross negligence, Y


suffered injuries that resulted in the

abortion of the foetus she carried. Y sued X


for, among other damages, P1 million for

the death of a family member. Is Y entitled


to indemnity for the death of the foetus she

carried?

(A) Yes, since the foetus is already


regarded as a child from conception,
though unborn.

(B) No, since Xs would not have

known that the accident would


result in Ys abortion.

(C) No, since birth determines

personality, the accident did not

result in the death of a person.

(D) Yes, since the mother believed in


her heart that she lost a child.

2010 Civil Law Exam MCQ

(September 12, 2010)

No.II. Multiple choice.

(A). A had a 4-storey building which was


constructed by Engineer B. After five years,

the building developed cracks and its


1659 NCC, the proprietor of a building or

should the lessees sue for damages? (1%) structure is responsible for the damages

(1). A, the owner resulting from its total or partial

collapse, if it is due to lack of necessary


(2). B, the engineer
repairs.

(3). both A & B


Under Article 1723, NCC, the engineer

SUGGESTED ANSWER: or architect who drew up the plans and

specifications for a building is liable for


3. Both A & B.
damage if 15 years from the completion

The lessee may proceed against A for of the structure the same should

breach of contract, and against B for tort collapse by a reason of a defect by those

or statutory liability. Under Article 1654 plans and specifications, or due to the

(2) of the New Civil Code, the lessor is defects in the ground. This liability

obliged to make all the necessary repairs maybe enforced against the architect or

in order to keep the leased property engineer even by a third party who has

suitable for the use to which it has been no privity of contract with the architect

devoted. Consequently, under Article or engineer under Article 2192, NCC.

Never Let The Odds Keep You From Pursuing What You Know In Your Heart You Were Meant To Do.-
Leroy Satchel Paige
Page 176 of 180
Civil Law Q&As (2007-2013) hectorchristopher@yahoo.com dbaratbateladot@gmail.com

problem are clearly those resulting from

ALTERNATIVE ANSWER: defects in the construction plans or

specifications.
No.1. A , the owner .

The lessee can sue only the lessor for

breach of contract under Article 1659 in

relation to Article 1654, NCC. The lessee

cannot sue the architect or the engineer

because there was no privity of contracts

between them. When sued, however, the

lessor may file a third party claim

against the architect or the engineer.

ANOTHER ALTERNATIVE ANSWER:

No. 2. B, the Engineer .

Under Article 1723 the engineer or

architect who drew up the plans and

specifications for a building is liable for

damages if within 15 years from the

completion of the structure, the same

should collapse by reason of a defect in

those plans and specifications, or due to

the defects in the ground. Under Article

2192 (NCC), however, if the damages

should be the result of any of the defects

in the construction mentioned in Art

1723, NCC, the third person suffering

damages may proceed only against the

engineer or architect or contractor

within the period fixed therein. The

damages suffered by the lessee in the


No. 4. None of the above.

(B) O, owner of Lot A, learning that


The general rule is that the treasure
Japanese soldiers may have buried gold
shall belong to the spouses X and Y, the
and other treasures at the adjoining vacant
owner of Lot B. Under Article 438 (NCC),
Lot B belonging to spouses X & Y, excavated
the exception is that when the discovery
in Lot B where she succeeded in unearthing

gold and precious stones. How will the of a hidden treasure is made on the

treasures found by O be divided? (1%) property of another and by chance, one-

half thereof shall belong to the owner of


(1). 100% to O as finder
the land and the other one-half is

(2). 50% to O and 50% to the spouses X allowed to the finder. In the problem, the

and Y finding of the treasure was not by

chance because O knew that the treasure


(3). 50% to O and 50% to the state
was in Lot B. While a trespasser is also

(4). None of the above. not entitled to any share, and there is no

indication in the problem whether or not


SUGGESTED ANSWER: O was a trespasser, O is not entitled to a

Never Let The Odds Keep You From Pursuing What You Know In Your Heart You Were Meant To Do.-
Leroy Satchel Paige
Page 177 of 180
Civil Law Q&As (2007-2013) hectorchristopher@yahoo.com dbaratbateladot@gmail.com

No. 2 and No.4 are both false. The

share because the finding was not by acceptance by the donees father alone

chance.

(C) A executed a Deed of Donation in favor

of B, a bachelor, covering a parcel of land


valued at P1 million. B was, however, out of

the country at the time. For the donation to


be valid, (1%)

(1). B may e-mail A accepting the donation.

(2). The donation may be accepted by Bs


father with whom he lives.

(3). B can accept the donation anytime


convenient to him.

(4). Bs mother who has a general power of


attorney may accept the donation for him.

(5). None of the above is sufficient to make


Bs acceptance valid

SUGGESTED ANSWER:

No. 5 None of the above is sufficient to

make B's acceptance valid .

Since the donation covered an

immovable property, the donation and

the acceptance must be in public

document and e-mail is not a public

document. Hence, No.1 is false.


(D) A executed a 5-page notarial will before a

notary public and three witnesses. All of them


or mother alone, even in a public
signed each and every page of the will.
document, is not sufficient because the

father and mother did not have a special One of the witnesses was B, the father of
power of attorney for the purpose. Under one of the legatees to the will. What is the

Article 745 (NCC), the donee must accept effect of B being a witness to the will? (1%)

the donation personally, or through an


(1). The will is invalidated
authorized person with a special power of

attorney for the purpose; otherwise, the (2). The will is valid and effective

donation shall be void.


(3). The legacy given to Bs child is not valid
No.3 is also false. B cannot accept the
SUGGESTED ANSWER:
donation anytime at his convenience.

Under Article 749 NCC, the donee may No. 3. The legacy given to B's child is not
accept the donation only during the valid.
lifetime of the donor.
The validity of the will is not affected by

the legacy in favor of the son of an

Never Let The Odds Keep You From Pursuing What You Know In Your Heart You Were Meant To Do.-
Leroy Satchel Paige
Page 178 of 180
Civil Law Q&As (2007-2013) hectorchristopher@yahoo.com dbaratbateladot@gmail.com

(E). letters a and b

attesting witness to the will. However,

the said legacy is void under Article 823

NCC.

ALTERNATIVE ANSWER:

No. 2 .The will is valid and effective.

Under Article 823 ( NCC ),the legacy

given in favor of the son of an

instrumental witness to a will has no

effect on the validity of the will. Hence,

the will is valid and effective.

2007 Civil Law Exam MCQ

(September 09, 2007)

No.IX. Multiple choice: Choose the right


answer. (2% each)

(1). The parties to a bailment are the:

(A). bailor;

(B). bailee;

(C) comodatario;

(D). all the above;

(E). letters a and b

SUGGESTED ANSWER:
(C). a necessary deposit
ALTERNATIVE ANSWER:

(3). A contract of antichresis is always:


(D). all the above

(A). a written contract;


(2). A deposit made in compliance with a
legal obligation is:
(B). a contract, with a stipulation that the
debt will be paid through receipt of the
(A). an extrajudicial deposit;
fruits of an immovable;

(B). a voluntary deposit;


(C). Involves the payment of interests, if

(C). a necessary deposit; owing;

(D). a deposit with a warehouseman; (D). All of the above;

(E). letters a and b (E). Letters a and b

SUGGESTED ANSWER: SUGGESTED ANSWER:

Never Let The Odds Keep You From Pursuing What You Know In Your Heart You Were Meant To Do.-
Leroy Satchel Paige
Page 179 of 180
Civil Law Q&As (2007-2013) hectorchristopher@yahoo.com dbaratbateladot@gmail.com

(C). 3/5 of the number of creditors

(D). All of the above; should agree to the settlement;

(4). An, assignee in a proceeding under the


Insolvency Law does not have the duty of:

(A). suing to recover the properties of the


state of the insolvent debtor;

(B). selling property of the insolvent debtor;

(C). ensuring that a debtor corporation

operate the business efficiently and

effectively while the proceedings are

pending;

(D). collecting and discharging debts owed


to the insolvent debtor.

SUGGESTED ANSWER:

(C). ensuring that a debtor corporation

operate the business efficiently and

effectively while the proceedings are

pending;

(5). In order to obtain approval of the


proposed settlement of the debtor in an
insolvency proceeding.

(A). the court must initiate the proposal

(B). 2/3 of the number of creditors should


agree to the settlement;
full credit for the two items regardless of

(D). 1/3 of the total debts must be their answers.]

represented by the approving creditors;

(E). Letters a and b


References:

SUGGESTED ANSWER:

Answers to Bar Examination
(C). 3/5 of the number of creditors Questions by the UP LAW COMPLEX
should agree to the settlement; (2007, 2009, 2010)


[Note: Items 4&5 on Insolvency Law are UP LAW REVIEW

not included within the coverage of Civil



Law but Commercial Law. It is therefore PHILIPPINE ASSOCIATION OF LAW
SCHOOLS (2008)
suggested that the examinees be given

lawphil.net

Never Let The Odds Keep You From Pursuing What You Know In Your Heart You Were Meant To Do.-
Leroy Satchel Paige
Page 180 of 180

You might also like